Sei sulla pagina 1di 159

IV.

REMEDIES

1. TAX ADMINISTRATION

Sections 2-3, 9-20, 232-235, 244-246 and 290, Tax Code

2. POWERS OF THE CIR

FITNESS BY DESIGN, INC., G.R. No. 177982


Petitioner,
Present:

QUISUMBING, J., Chairperson,


CARPIO MORALES,
- versus - TINGA,
VELASCO, JR., and
BRION, JJ.

COMMISSIONER ON Promulgated:
INTERNAL REVENUE, October 17, 2008
Respondent.
x-------------------------------------------------x

DECISION

CARPIO MORALES, J.:


On March 17, 2004, the Commissioner on Internal Revenue (respondent) assessed Fitness by
Design, Inc. (petitioner) for deficiency income taxes for the tax year 1995 in the total amount
of P10,647,529.69.[1] Petitioner protested the assessment on the ground that it was issued beyond the
three-year prescriptive period under Section 203 of the Tax Code.[2]Additionally, petitioner claimed
that since it was incorporated only on May 30, 1995, there was no basis to assume that it had already
earned income for the tax year 1995.[3]

On February 1, 2005, respondent issued a warrant of distraint and/or levy against


petitioner,[4] drawing petitioner to file on March 1, 2005 a Petition for Review (with Motion to Suspend
Collection of Income Tax, Value Added Tax, Documentary Stamp Tax and Surcharges and Interests
subject of this Petition)[5] before the Court of Tax Appeals (CTA) before which it reiterated its defense
of prescription. The petition was docketed as CTA Case No. 7160.

1
In his Answer,[6] respondent alleged:

The right of the respondent to assess petitioner for deficiency income tax, VAT
and Documentary Stamp Tax for the year 1995 has not prescribed pursuant to Section
222(a) of the 1997 Tax Code.Petitioners 1995 Income Tax Return (ITR) filed on April
11, 1996 was false and fraudulent for its deliberate failure to declare its true
sales. Petitioner declared in its 1995 Income Tax Return that it was on its pre-
operation stage and has not declared its income. Investigation by the revenue officers
of the respondent, however, disclosed that it has been operating/doing business and
had sales operations for the year 1995 in the total amount of P7,156,336.08 which it
failed to report in its 1995 ITR. Thus, for the year 1995, petitioner filed a fraudulent
annual income return with intent to evade tax. Likewise, petitioner failed to file
Value-Added Tax (VAT) Return and reported the amount of P7,156,336.08 as its
gross sales for the year 1995. Hence, for failure to file a VAT return and for filing a
fraudulent income tax return for the year 1995, the corresponding taxes may be
assessed at any time within ten (10) years after the discovery of such omission or
fraud pursuant to Section 222(a) of the 1997 Tax Code.

The subject deficiency tax assessments have already become final, executory
and demandable for failure of the petitioner to file a protest within the reglementary
period provided for by law. The alleged protest allegedly filed on June 25, 2004 at the
Legal Division, Revenue Region No. 8, Makati City is nowhere to be found in the BIR
Records nor reflected in the Record Book of the Legal Division as normally done by
our receiving clerk when she receive[s] any document. The respondent, therefore, has
legal basis to collect the tax liability either by distraint and levy or civil
action.[7](Emphasis and underscoring supplied)

The aforecited Section 222(a)[8] of the 1997 Tax Code provides:

In the case of a false or fraudulent return with intent to evade tax or of failure to
file a return, the tax may be assessed, or a proceeding in court for the collection of
such tax may be filed without assessment, at any time within ten (10) years after the
discovery of the falsity, fraud, or omission: Provided, That in a fraud assessment
which has become final and executory, the fact of fraud shall be judicially taken
cognizance of in the civil or criminal action for the collection thereof. (Underscoring
supplied)

The Bureau of Internal Revenue (BIR) in fact filed on March 10, 2005 a criminal complaint
before the Department of Justice against the officers and accountant of petitioner for violation of the

2
provisions of The National Internal Revenue Code of 1977, as amended,[9] covering the taxable year
1995. The criminal complaint was docketed as I.S. No. 2005-203.

On motion of petitioner in CTA Case No. 7160, a preliminary hearing on the issue of
prescription[10] was conducted during which petitioners former bookkeeper attested that a former
colleague certified public accountant Leonardo Sablan (Sablan) illegally took custody of petitioners
accounting records, invoices, and official receipts and turned them over to the BIR.[11]

On petitioners request, a subpoena ad testificandum was issued to Sablan for the hearing before
the CTA scheduled on September 4, 2006 but he failed to appear.[12]

Petitioner thus requested for the issuance of another subpoena ad testificandum to Sablan for the
hearing scheduled on October 23, 2006,[13] and of subpoena duces tecum to the chief of the National
Investigation Division of the BIR for the production of the Affidavit of the Informer bearing on the
assessment in question.[14] Petitioners requests were granted.[15]

During the scheduled hearing of the case on October 23, 2006, on respondents counsels
manifestation that he was not furnished a copy of petitioners motion for the issuance of subpoenaes, the
CTA ordered petitioner to file a motion for the issuance of subpoenas and to furnish respondents
counsel a copy thereof.[16] Petitioner complied with the CTA order.[17]

In a related move, petitioner submitted written interrogatories addressed to Sablan and to Henry
Sarmiento and Marinella German, revenue officers of the National Investigation Division of the BIR.[18]

By Resolution[19] of January 15, 2007, the CTA denied petitioners Motion for Issuance of
Subpoenas and disallowed the submission by petitioner of written interrogatories to Sablan, who is not a
party to the case, and the revenue officers,[20] it finding that the testimony, documents, and admissions
sought are not relevant.[21] Besides, the CTA found that to require Sablan to testify would violate
Section 2 of Republic Act No. 2338, as implemented by Section 12 of Finance Department Order No.
46-66, proscribing the revelation of identities of informers of violations of internal revenue laws, except
when the information is proven to be malicious or false.[22]

In any event, the CTA held that there was no need to issue a subpoena duces tecum to obtain the
Affidavit of the Informer as the same formed part of the BIR records of the case, the production of
which had been ordered by it.[23]

3
Petitioners Motion for Reconsideration[24] of the CTA Resolution of January 15, 2007 was
denied,[25] hence, the present Petition for Certiorari[26] which imputes grave abuse of discretion to the
CTA

I.
x x x in holding that the legality of the mode of acquiring the documents which are the
bases of the above discussed deficiency tax assessments, the subject matter of the
Petition for Review now pending in the Honorable Second Division, is not material
and relevant to the issue of prescription.

II.
x x x in holding that Mr. Leonardo Sablans testimony, if allowed, would violate RA
2338 which prohibits the BIR to reveal the identity of the informer since 1) the
purpose of the subpoena is to elicit from him the whereabouts of the original
accounting records, documents and receipts owned by the Petitioner and not to
discover if he is the informer since the identity of the informer is not relevant to the
issues raised; 2) RA 2338 cannot legally justify violation of the Petitioners property
rights by a person, whether he is an informer or not, since such RA cannot allow such
invasion of property rights otherwise RA 2338 would run counter to the constitutional
mandate that no person shall be deprive[d] of life, liberty or property without due
process of law.

III.
x x x in holding that the issuance of subpoena ad testificandum would constitute a
violation of the prohibition to reveal the identity of the informer because compliance
with such prohibition has been rendered moot and academic by the voluntary
admissions of the Respondent himself.

IV.
x x x in holding that the constitutional right of an accused to examine the witness
against him does not exist in this case. The Petitioners liability for tax deficiency
assessment which is the main issue in the Petition for Review is currently pending at
the Honorable Second Division. Therefore, it is a prejudicial question raised in the
criminal case filed by the herein Respondent against the officers of the Petitioner with
the Department of Justice.

V.
x x x in dismissing the request for subpoena ad testificandum because the Opposition
thereto submitted by the Respondent was not promptly filed as provided by the Rules
of Court thus, it is respectfully submitted that, Respondent has waived his right to
object thereto.
4
VI.
x x x when the Honorable Court of Tax Appeals ruled that the purpose of the
Petitioner in requesting for written interrogatories is to annoy, embarrass, or oppress
the witness because such ruling has no factual basis since Respondent never alleged
nor proved that the witnesses to whom the interrogatories are addressed will be
annoyed, embarrassed or oppressed; besides the only obvious purpose of the Petitioner
is to know the whereabouts of accounting records and documents which are in the
possession of the witnesses to whom the interrogatories are directed and to ultimately
get possession thereof.Granting without admitting that there is annoyance,
embarrassment or oppression; the same is not unreasonable.

VII.
x x x when it failed to rule that the BIR officers and employees are not covered by the
prohibition under RA 2338 and do not have the authority to withhold from the
taxpayer documents owned by such taxpayer.

VIII.
x x x when it required the clear and unequivocal proof of relevance of the documents
as a condition precedent for the issuance of subpoena duces tecum.

IX.
x x x when it quashed the subpoena duces tecum as the Honorable Court had issued an
outstanding order to the Respondent to certify and forward to the CTA all the records
of the case because up to the date of this Petition the BIR records have not been
submitted yet to the CTA.[27]

Grave abuse of discretion implies such capricious and whimsical exercise of judgment as
equivalent to lack of jurisdiction or, in other words, when the power is exercised in an arbitrary or
despotic manner by reason of passion or personal hostility, and it must be so patent and gross as to
amount to an evasion of positive duty or a virtual refusal of duty enjoined or to act at all in
contemplation of law.[28]

The Court finds that the issuance by the CTA of the questioned resolutions was not tainted by
arbitrariness.

The fact that Sablan was not a party to the case aside, the testimonies, documents, and
admissions sought by petitioner are not indeed relevant to the issue before the CTA. For in requesting
the issuance of the subpoenas and the submission of written interrogatories, petitioner sought to
establish that its accounting records and related documents, invoices, and receipts which were the bases

5
of the assessment against it were illegally obtained. The only issues, however, which surfaced during
the preliminary hearing before the CTA, were whether respondents issuance of assessment against
petitioner had prescribed and whether petitioners tax return was false or fraudulent.

Besides, as the CTA held, the subpoenas and answers to the written interrogatories would violate
Section 2 of Republic Act No. 2338 as implemented by Section 12 of Finance Department Order No.
46-66.
Petitioner claims, however, that it only intended to elicit information on the whereabouts of the
documents it needs in order to refute the assessment, and not to disclose the identity of the
informer.[29] Petitioners position does not persuade. The interrogatories addressed to Sablan and the
revenue officers show that they were intended to confirm petitioners belief that Sablan was the
informer. Thus the questions for Sablan read:

1. Under what circumstances do you know petitioner corporation? Please


state in what capacity, the date or period you obtained said knowledge.
2. Do you know a Ms. Elnora Carpio, who from 1995 to the early part of
1996 was the book keeper of petitioner? Please state how you came to know of
Ms. Carpio.
3. At the time that Ms. Carpio was book keeper of petitioner did she consult
you or show any accounting documents and records of petitioner?
4. What documents, if any, did you obtain from petitioner?
5. Were these documents that you obtained from petitioner submitted to the
Bureau of Internal Revenue (BIR)? Please describe said documents and under
what circumstances the same were submitted.
6. Was the consent of the petitioner, its officers or employees obtained
when the documents that you obtained were submitted to the BIR? Please state
when and from whom the consent was obtained.
7. Did you execute an affidavit as an informer in the assessment which was
issued by the BIR against petitioner for the tax year 1995 and other
years?[30] (Underscoring supplied)

while the questions for the revenue officers read:

1. Where did you obtain the documents, particularly the invoices and official receipts,
which [were] used by your office as evidence and as basis of the assessment for
deficiency income tax and value added tax for the tax year 1995 issued against
petitioner?

2. Do you know Mr. Leonardo Sablan? Please state under what circumstance you
came to know Mr. Sablan?[31] (Underscoring supplied)

6
Petitioner impugns the manner in which the documents in question reached the BIR, Sablan
having allegedly submitted them to the BIR without its (petitioners) consent. Petitioners lack of consent
does not, however, imply that the BIR obtained them illegally or that the information received is false or
malicious. Nor does the lack of consent preclude the BIR from assessing deficiency taxes on petitioner
based on the documents. Thus Section 5 of the Tax Code provides:

In ascertaining the correctness of any return, or in making a return when none


has been made, or in determining the liability of any person for any internal revenue
tax, or in collecting any such liability, or in evaluating tax compliance, the
Commissioner is authorized:

(A) To examine any book, paper, record or other data which may be
relevant or material to such query;
(B) To obtain on a regular basis from any person other than the
person whose internal revenue tax liability is subject to audit or
investigation, or from any office or officer of the national and local
governments, government agencies and instrumentalities, including
the Bangko Sentral ng Pilipinas and government-owned and controlled
corporations, any information such as, but not limited to, costs and
volume of production, receipts or sales and gross incomes of taxpayers,
and the names, addresses, and financial statements of corporations,
mutual fund companies, insurance companies, regional operating
headquarters of multinational companies, joint accounts, associations,
joint ventures or consortia and registered partnerships and their
members;
(C) To summon the person liable for tax or required to file a return, or
any officer or employee of such person, or any person having
possession, custody, or care of the books of accounts and other
accounting records containing entries relating to the business of the
person liable for tax, or any other person, to appear before the
Commissioner or his duly authorized representatives at a time and place
specified in the summons and to produce such books, papers, records, or
other data, and to give testimony;
(D) To take such testimony of the person concerned, under oath, as may
be relevant or material to such inquiry; and
(E) To cause revenue officers and employees to make a canvass from
time to time of any revenue district or region and inquire after and
concerning all persons therein who may be liable to pay any internal
revenue tax, and all persons owning or having the care, management or
possession of any object with respect to which a tax is imposed.

x x x x (Emphasis and underscoring supplied)

7
The law thus allows the BIR access to all relevant or material records and data in the person of
the taxpayer,[32] and the BIR can accept documents which cannot be admitted in a judicial proceeding
where the Rules of Court are strictly observed.[33] To require the consent of the taxpayer would defeat
the intent of the law to help the BIR assess and collect the correct amount of taxes.

Petitioners invocation of the rights of an accused in a criminal prosecution to cross examine the
witness against him and to have compulsory process issued to secure the attendance of witnesses and
the production of other evidence in his behalf does not lie. CTA Case No. 7160 is not a criminal
prosecution, and even granting that it is related to I.S. No. 2005-203, the respondents in the latter
proceeding are the officers and accountant of petitioner-corporation, not petitioner. From the complaint
and supporting affidavits in I.S. No. 2005-203, Sablan does not even appear to be a witness against the
respondents therein.[34]

AT ALL EVENTS, issuance of subpoena duces tecum for the production of the documents
requested by the petitioner which documents petitioner claims to be crucial to its defense[35] is
unnecessary in view of the CTA order for respondent to certify and forward to it all the records of the
case.[36] If the order has not been complied with, the CTA can enforce it by citing respondent for
indirect contempt.[37]

WHEREFORE, in light of the foregoing disquisition, the petition is DISMISSED.Costs against


petitioner.SO ORDERED.

DIGEST: Fitness by Design v. CIR


G.R. No. 177982 October 17, 2008
CARPIO MORALES, J.

Lessons Applicable: BIR power to gather information without consent

Laws Applicable:

FACTS:
 March 17, 2004: CIR assessed Fitness by Design Inc. for deficiency Income Taxes for the year of
1995 for P 10,647, 529.69
 February 1, 2005: CIR issued a warrant of distraint and levy against petitioner which prompted
petitioner to file a Petition for Review before the CTA where he alleged his defense of
prescription based on Sec. 203 of the Tax Code.
 CIR answer: Tax return was false and fraudulent for deliberately failing to declare its true sales of
P 7,156,336.08 and failure to file a VAT return for it. Since petitioner failed to file a protest, it is

8
subject to either distraint or levy. Moreover, it cited Sec. 222 (a) of 1997 Tax Code where false and
fraudulent return with intent to evade tax or failure to file a return prescribe 10 years after the
discovery of the falsity, fraud or omission.
 March 10, 2005: BIR filed a criminal complaint before the DOJ against the officers and accountant
of petitioner for violation against the 1977 NIRC.
 During the preliminary hearing on the issue of prescription, petitioner's former bookkeeper attested
that his former colleague, CPA Sablan, illegally took custody of accounting records and turned
them over to the BIR.
 Petitioner then requested a subpoena ad testificandum for Sablan who failed to appear.
 CTA: Denied the motion for issuance of subpoena and disallowed the submission of written
interrogatories to Sablan who is NOT a party to the case nor was his testimony relevant. It also
violates Section 2 of Republic Act No. 2338, as implemented by Section 12 of Finance Department
Order No. 46-66, proscribing the revelation of identities of informers of violations of internal
revenue laws, except when the information is proven to be malicious or false. Moreover, the
subpoena is NOT needed to obtain affidavit of the informer.
ISSUE: W/N BIR can use the information without petitioner's consent

HELD: YES.
 Sec. 5 of the tax code provides that the BIR is authorized to obtain from any person other than the
person whose internal revenue tax liability is subject to audit or investigation and can even summon
any person having possession, custody or care of the books of accountants and other accounting
records containing entries relating to the business of the person liable for tax. This includes even
those which cannot be admitted in a judicial proceeding where the Rules of Court are strictly
observed. CTA case is not a criminal prosecution where he can cross examine the witness against
him. CTA can enforce its order by citing them for indirect contempt.

G.R. No. 170389 October 20, 2010

COMMISSION OF INTERNAL REVENUE, Petitioner,


vs.
AQUAFRESH SEAFOODS, INC., Respondent.

DECISION

PERALTA, J.:

Before this Court is a petition for review on certiorari,1 under Rule 45 of the Rules of Court, seeking to
set aside the November 9, 2005 Decision2 of the Court of Tax Appeals (CTA) En Banc in CTA-E.B.
No. 77. The CTA En Bancaffirmed the December 22, 2004 Decision of the CTA First Division.

9
The facts of the case are as follows:

On June 7, 1999, respondent Aquafresh Seafoods Inc. sold to Philips Seafoods, Inc. two parcels of
land, including improvements thereon, located at Barrio Banica, Roxas City, for the consideration of
Three Million One Hundred Thousand Pesos (Php 3,100, 000.00). Said properties were covered under
Transfer Certificate of Titles Nos. T-21799 and T-21804.

Respondent then filed a Capital Gains Tax Return/Application for Certification Authorizing
Registration and paid the amount of Php186,000.00, representing the Capital Gains Tax (CGT) and the
amount of Php46,500.00, representing the Documentary Stamp Tax (DST) due from the said sale.
Subsequently, Revenue District Officer Gil G. Tabanda issued Certificate Authorizing Registration No.
1071477.

The Bureau of Internal Revenue (BIR), however, received a report that the lots sold were undervalued
for taxation purposes. This prompted the Special Investigation Division (SID) of the BIR to conduct an
occular inspection over the properties. After the investigation, the SID concluded that the subject
properties were commercial with a zonal value of Php2,000.00 per square meter.

On September 15, 2000, Regional Director Leonardo Q. Sacamos (Director Sacamos) of the Revenue
Region Iloilo City sent two Assessment Notices apprising respondent of CGT and DST defencies in the
sum of Php1,372,171.46 and Php356,267.62, respectively. Director Sacamos relied on the findings of
the SID that the subject properties were commercial with a zonal valuation of Php2,000.00 per square
meter.

On October 1, 2000, respondent sent a letter protesting the assessments made by Director Sacamos. On
December 1, 2000, Director Sacamos denied respondent's protest for lack of legal basis. Respondent
appealed, but the same was denied with finality on February 13, 2002.

On March 19, 2002, respondent filed a petition for review3 before the CTA seeking the reversal of the
denial of its protest. The main thrust of respondent's petition was that the subject properties were
located in Barrio Banica, Roxas, where the pre-defined zonal value was Php650.00 per square meter
based on the "Revised Zonal Values of Real Properties in the City of Roxas under Revenue District
Office No. 72 – Roxas City" (1995 Revised Zonal Values of Real Properties). Respondent asserted that
the subject properties were classified as "RR" or residential and not commercial. Respondent argued
that since there was already a pre-defined zonal value for properties located in Barrio Banica, the BIR
officials had no business re-classifying the subject properties to commercial.

On December 22, 2004, the CTA promulgated a Decision4 ruling in favor of respondent, the dispositive
portion of which reads:

IN VIEW OF THE FOREGOING, respondent's assessments for deficiency capital against tax and
documentary stamp taxes are hereby CANCELLED and SET ASIDE. x x x

SO ORDERED.5

10
Ruling in favor of respondent, the CTA opined that that the existing Revised Zonal Values in the City
of Roxas should prevail for purposes of determining respondent's tax liabilities, thus:

While respondent is given the authority to determine the fair market value of the subject properties for
the purpose of computing internal revenue taxes, such authority is not without restriction or
limitation. The first sentence of Section 6(E) sets the limitation or condition in the exercise of such
power by requiring respondent to consult with competent appraisers both from private and
public sectors. As there was no re-evaluation and no revision of the zonal values of the subject
properties in Roxas City at the time of the sale, respondent cannot unilaterally determine the zonal
values of the subject properties by invoking his powers of obtaining information and making
assessments under Sections 5 and 6 of the NIRC. The existing Revised Zonal Values of Real
Properties in the City of Roxas shall prevail for the purpose of determining the proper tax
liabilities of petitioner.6

Petitioner Commissioner of Internal Revenue filed a Motion for Reconsideration, which was, however,
denied by the CTA in a Resolution7 dated April 4, 2005.

Petitioner then appealed to the CTA En Banc.

In a Decision dated November 9, 2005, the CTA En Banc dismissed petitioner's appeal, the dispositive
portion of which reads:

WHEREFORE, premises considered, the Petition for Review is DISMISSED for lack of merit.

SO ORDERED.8

The CTA En Banc ruled that the 1995 Revised Zonal Values of Real Properties should prevail. Said
court relied on Section 6 (E) of the National Internal Revenue Code (NIRC) which requires
consultation from appraisers, from both the public and private sectors, in fixing the zonal valuation of
properties. The CTA En Banc held that petitioner failed to prove any amendment effected on the 1995
Revised Zonal Values of Real Properties at the time of the sale of the subject properties.

Hence, herein petition, with petitioner raising the following issues for this Court's resolution, to wit:

I.

WHETHER OR NOT THE REQUIREMENT OF CONSULTATION WITH COMPETENT


APPRAISERS BOTH FROM THE PRIVATE AND PUBLIC SECTORS IN DETERMINING THE
FAIR MARKET VALUE OF THE SUBJECT LOTS IS APPLICABLE IN THE CASE AT BAR.

II.

WHETHER OR NOT THE COURT OF TAX APPEALS EN BANC COMMITTED GRAVE ERROR
IN APPLYING THE FAIR MARKET VALUE BASED ON THE ZONAL VALUATION OF A
RESIDENTIAL LAND AS TAX BASE IN THE COMPUTATION OF CAPITAL GAINS TAX AND
DOCUMENTARY STAMP TAX DEFICIENCIES OF RESPONDENT.9
11
The petition is not meritorious. The issues being interrelated, this Court shall discuss the
same in seriatim.

Under Section 27(D)(5) of the NIRC of 1997, a CGT of six (6%) percent is imposed on the gains
presumed to have been realized in the sale, exchange or disposition of lands and/or buildings which are
not actively used in the business of a corporation and which are treated as capital assets based on the
gross selling price or fair market value as determined in accordance with Section 6(E) of the NIRC,
whichever is higher.

On the other hand, under Section 196 of the NIRC, DST is based on the consideration contracted to be
paid or on its fair market value determined in accordance with Section 6(E) of the NIRC, whichever is
higher.

Thus, in determining the value of CGT and DST arising from the sale of a property, the power of the
CIR to assess is subject to Section 6(E) of the NIRC, which provides:

Section 6. Power of the Commissioner to Make Assessments and Prescribe Additional Requirements
for Tax Administration and Enforcement. -

xxxx

(E) Authority of the Commissioner to Prescribe Real Property Values – The Commissioner is hereby
authorized to divide the Philippines into different zones or area and shall, upon consultation with
competent appraisers both from the private and public sectors, determine the fair market value of
real properties located in each zone or area. For purposes of computing internal revenue tax, the value
of the property shall be, whichever is higher of:

(1) the fair market value as determined by the Commissioner; or

(2) the fair market value as shown in the schedule of values of the Provincial and City
Assessors.

While the CIR has the authority to prescribe real property values and divide the Philippines into zones,
the law is clear that the same has to be done upon consultation with competent appraisers both from the
public and private sectors. It is undisputed that at the time of the sale of the subject properties found
in Barrio Banica, Roxas City, the same were classified as "RR," or residential, based on the 1995
Revised Zonal Value of Real Properties. Petitioner, thus, cannot unilaterally change the zonal valuation
of such properties to "commercial" without first conducting a re-evaluation of the zonal values as
mandated under Section 6(E) of the NIRC.

Petitioner argues, however, that the requirement of consultation with competent appraisers is
mandatory only when it is prescribing real property values – that is when a formulation or change is
made in the schedule of zonal values. Petitioner also contends that what it did in the instant case was
not to prescribe the zonal value, but merely classify the same as commercial and apply the
corresponding zonal value for such classification based on the existing schedule of zonal values in
Roxas City.10
12
We disagree.

To this Court's mind, petitioner's act of re-classifying the subject properties from residential to
commercial cannot be done without first complying with the procedures prescribed by law. It bears to
stress that ALL the properties in Barrio Banica were classified as residential, under the 1995 Revised
Zonal Values of Real Properties. Thus, petitioner's act of classifying the subject properties involves a
re-classification and revision of the prescribed zonal values.

In addition, Revenue Memorandum No. 58-69 provides for the procedures on the establishment of the
zonal values of real properties, viz.:

(1) The submission or review by the Revenue District Offices Sub-Technical Committee of the
schedule of recommended zonal values to the TCRPV;

(2) The evaluation by TCRPV of the submitted schedule of recommended zonal values of real
properties;

(3) Except in cases of correction or adjustment, the TCRPV finalizes the schedule and submits
the same to the Executive Committee on Real Property Valuation (ECRPV);

(3) Upon approval of the schedule of zonal values by the ECRPV, the same is embodied in a
Department Order for implementation and signed by the Secretary of Finance. Thereafter, the
schedule takes effect (15) days after its publication in the Official Gazette or in any newspaper
of general circulation.

Petitioner failed to prove that it had complied with Revenue Memorandum No. 58-69 and that a
revision of the 1995 Revised Zonal Values of Real Properties was made prior to the sale of the subject
properties. Thus, notwithstanding petitioner's disagreement to the classification of the subject
properties, the same must be followed for purposes of computing the CGT and DST. It bears stressing,
and as observed by the CTA En Banc, that the 1995 Revised Zonal Values of Real Properties was
drafted by petitioner, BIR personnel, representatives from the Department of Finance, National Tax
Research Center, Institute of Philippine Real Estate Appraisers and Philippine Association of Realtors
Board, which duly satisfied the requirement of consultation with public and private appraisers.11

Petitioner contends, nevertheless, that its act of classifying the subject properties based on actual use
was in accordance with guidelines number 1-b and 2 as set forth in "Certain Guidelines in the
Implementation of Zonal Valuation of Real Properties for RDO 72 Roxas City" (Zonal Valuation
Guidelines).12

Section 1 (b) of the Zonal Valuation Guidelines reads:

1. No zonal value has been prescribed for a particular classification of real property.

Where in the approved schedule of zonal values for a particular barangay -

xxxx
13
b) No zonal value has been prescribed for a particular classification of real property in one barangay,
the zonal value prescribed for the same classification of real property located in an adjacent barangay
of similar conditions shall be used.

Section 1 (b) does not apply to the case at bar for the simple reason that said proviso operates only
when "no zonal valuation has been prescribed." The properties located in Barrio Banica, Roxas City
were already subject to a zonal valuation, a fact which even petitioner has admitted in its petition, thus:

It must be noted that under the schedule of zonal values, Barangay Banica, where the subject lots are
situated, has a single classification only – that of a residential area. Accordingly, it has a prescribed
zonal value of Php650.00 per square meter.13

Petitioner, however, also relies on Section 2 (a) of the Zonal Valuation Guidelines, to justify its action.
Said section states:

2. Predominant Use of Property.

a) All real properties, regardless of actual use, located in a street/barangay zone, the use of which are
predominantly commercial shall be classified as "Commercial" for purposes of zonal valuation.

In BIR Ruling No. 041-2001, issued on September 18, 2001, the BIR tackled the application of a
provision which is identical to Section 2 (a) of the Zonal Valuation Guidelines. BIR Ruling No. 041-
2001 involved a request by the Iglesia Ni Cristo that the re-computation of CGT and DST based on the
predominant use of the real properties located at Mindanao Avenue, Quezon City, be set aside. In said
case, the Iglesia ni Cristo paid the CGT and DST based on the zonal value of residential lots in Quezon
City. The Revenue District Officer, however, ordered a re-computation of the CGT and DST based on
the ground that the real property is located in a predominantly commercial area and must be classified
as commercial for purposes of zonal valuation. The BIR ruled in favor of Iglesia ni Cristo stating that
"Certain Guidelines in the Implementation of Zonal Valuation of Real Properties for RDO No. 38,
applying the predominant use of property as the basis for the computation of the Capital Gains and
Documentary Stamp Taxes, shall apply only when the real property is located in an area or zone
where the properties are not yet classified and their respective zonal valuation are not yet
determined." The pertinent portion of BIR Ruling No. 041-2001 reads:

In reply, please be informed that this Office finds your request meritorious. The number 2 guideline
laid down in Certain Guidelines in the implementation of Zonal valuation of Real Properties for RDO
No. 38- North Quezon City xxx does not apply to this case.

Number 2 of the CERTAIN GUIDELINES IN THE IMPLEMENTATION OF ZONAL VALUATION


OF REAL PROPERTIES FOR RD NO. 38 – NORTH QUEZON CITY" provides:

"2. PREDOMINANT USE OF PROPERTY:

ALL REAL PROPERTIES REGARDLESS OF ACTUAL USE, LOCATED IN A


STREET/BARANGAY ZONE, THE USE OF WHICH ARE PREDOMINANTLY

14
COMMERCIAL SHALL BE CLASSIFIED AS 'COMMERICIAL'FOR PURPOSES OF
ZONAL VALUATION."

It is the considered opinion of this Office that the guideline applies when the real property is
located in an area or zone where the properties are not yet classified and their respective zonal
valuation are not yet determined.

In the instant case, however, the classification and valuation of the properties located in
Mindanao Avenue, Bagong Bantay, have already been determined. Under Department of Finance
Order No. 6-2000, the properties along Mindanao Avenue had already been classified as residential and
commercial. The zonal valuation thereof had already been determined. x x x Therefore, the Revenue
District Officer of RDO No. 38 has no discretion to determine the classification or valuation of
the properties located in the pertinent area. The computation of the capital gains and documentary
stamp taxes shall be based on the zonal of residential properties located at Mindanao Avenue, Bago
Bantay, Quezon City.141avvphil

Based on the foregoing, this Court need not belabour on the applicability of Section 2 (a), as the BIR
itself has already ruled that the same shall apply only when the real property is located in an area or
zone where the properties are not yet classified and their respective zonal valuation are not yet
determined. As mentioned earlier, the subject properties were already part of the 1995 Revised Zonal
Value of Real Properties which classified the same as residential with a zonal value of Php650.00 per
square meter; thus, Section 2 (a) clearly has no application.

This Court agrees with the observation of the CTA that "zonal valuation was established with the
objective of having an ‘efficient tax administration by minimizing the use of discretion in the
determination of the tax based on the part of the administrator on one hand and the taxpayer on the
other hand.’"15 Zonal value is determined for the purpose of establishing a more realistic basis for real
property valuation. Since internal revenue taxes, such as CGT and DST, are assessed on the basis of
valuation, the zonal valuation existing at the time of the sale should be taken into account.16

If petitioner feels that the properties in Barrio Banica should also be classified as commercial, then
petitioner should work for its revision in accordance with Revenue Memorandum Order No. 58-69.
The burden was on petitioner to prove that the classification and zonal valuation in Barrio Banica have
been revised in accordance with the prevailing memorandum. In the absence of proof to the contrary,
the 1995 Revised Zonal Values of Real Properties must be followed.

Lastly, this Court takes note of the wording of Section 2 (b) of the Zonal Valuation Guidelines, to wit:

2. Predominant Use of Property.

b) The predominant use of other classification of properties located in a street/barangay


zone, regardless of actual use shall be considered for purposes of zonal valuation.

Based thereon, this Court rules that even assuming arguendo that the subject properties were used for
commercial purposes, the same remains to be residential for zonal value purposes. It appears that actual
use is not considered for zonal valuation, but the predominant use of other classification of properties
15
located in the zone. Again, it is undisputed that the entire Barrio Banica has been classified as
residential.

WHEREFORE, premises considered, the petition is denied. The November 9, 2005 Decision of the
Court of Tax Appeals En Banc, in CTA-E.B. No. 77, is hereby AFFIRMED.SO ORDERED.

DIGEST:

FACTS:
Aquafresh Seafoods sold two parcels of located at Barrio Banica in Roxas City and paid the
corresponding CGT and DST due on the sale. However, the BIR assessed Aquafresh Seafoods based
on its conclusion that the lots were classified as commercial and not residential as claimed by the
taxpayer. Aquafresh Seafood’s defense was that there was already a pre-defined zonal value for the
said lots and thus the BIR could not reclassify the same to be commercial lots.

ISSUE:
Is the requirement (under Section 6 of the Tax Code) of consultation with competent appraisers both
from the public and private sectors in determining fair market value applicable in this case?

HELD:
YES. The BIR’s position that the requirement of consultation with appraisers is mandatory only when
formulating or making changes in the schedule of zonal values is wrong. The Court held that the BIR’s
act of classifying the subject properties involved a re-classification and revision of the prescribed zonal
values. It was likewise added that the application of the rule of assigning zonal values based on the
‘predominant use of property’ only applies when the property is located in an area or zone where the
properties are not yet classified and their zonal values are not yet determined. If a determination has
already been made, the BIR has no discretion as regards its classification and/or valuation.

[G.R. No. 136975. March 31, 2005] COMMISSION OF INTERNAL REVENUE, petitioner,
vs. HANTEX TRADING CO., INC., respondent.

DECISION CALLEJO, SR., J.:

Before us is a petition for review of the Decision[ 1 ] of the Court of Appeals (CA) which
reversed the Decision[ 2 ] of the Court of Tax Appeals (CTA) in CTA Case No. 5126, upholding the
deficiency income and sales tax assessments against respondent Hantex Trading Co., Inc.

16
The Antecedents

The respondent is a corporation duly organized and existing under the laws of the Philippines.
Being engaged in the sale of plastic products, it imports synthetic resin and other chemicals for the
manufacture of its products. For this purpose, it is required to file an Import Entry and Internal
Revenue Declaration (Consumption Entry) with the Bureau of Customs under Section 1301 of the
Tariff and Customs Code.
Sometime in October 1989, Lt. Vicente Amoto, Acting Chief of Counter-Intelligence Division of
the Economic Intelligence and Investigation Bureau (EIIB), received confidential information that the
respondent had imported synthetic resin amounting to P115,599,018.00 but only
declared P45,538,694.57.[ 3 ] According to the informer, based on photocopies of 77 Consumption
Entries furnished by another informer, the 1987 importations of the respondent were understated in its
accounting records.[ 4 ] Amoto submitted a report to the EIIB Commissioner recommending that an
inventory audit of the respondent be conducted by the Internal Inquiry and Prosecution Office (IIPO) of
the EIIB.[ 5 ]
Acting on the said report, Jose T. Almonte, then Commissioner of the EIIB, issued Mission Order
No. 398-89[ 6 ] dated November 14, 1989 for the audit and investigation of the importations of Hantex
for 1987. The IIPO issued subpoena duces tecum and ad testificandum for the president and general
manager of the respondent to appear in a hearing and bring the following:

1. Books of Accounts for the year 1987;

2. Record of Importations of Synthetic Resin and Calcium Carbonate for the year 1987;

3. Income tax returns & attachments for 1987; and

4. Record of tax payments.[ 7 ]

However, the respondents president and general manager refused to comply with the subpoena,
contending that its books of accounts and records of importation of synthetic resin and calcium
bicarbonate had been investigated repeatedly by the Bureau of Internal Revenue (BIR) on prior
occasions.[ 8 ] The IIPO explained that despite such previous investigations, the EIIB was still
authorized to conduct an investigation pursuant to Section 26-A of Executive Order No. 127. Still, the
respondent refused to comply with the subpoena issued by the IIPO. The latter forthwith secured
certified copies of the Profit and Loss Statements for 1987 filed by the respondent with the Securities
and Exchange Commission (SEC).[ 9 ] However, the IIPO failed to secure certified copies of the
respondents 1987 Consumption Entries from the Bureau of Customs since, according to the custodian
thereof, the original copies had been eaten by termites.[ 1 0 ]
In a Letter dated June 28, 1990, the IIPO requested the Chief of the Collection Division, Manila
International Container Port, and the Acting Chief of the Collection Division, Port of Manila, to
authenticate the machine copies of the import entries supplied by the informer. However, Chief of the
Collection Division Merlita D. Tomas could not do so because the Collection Division did not have the

17
original copies of the entries. Instead, she wrote the IIPO that, as gleaned from the records, the
following entries had been duly processed and released after the payment of duties and taxes:

IMPORTER HANTEX TRADING CO., INC. SERIES OF 1987

ENTRY NO. DATE RELEASED ENTRY NO. DATE RELEASED

03058-87 1-30-87 50265-87 12-09-87

09120-87 3-20-87 46427-87 11-27-87

18089-87 5-21-87 30764-87 8-21-87

19439-87 6-2-87 30833-87 8-20-87

19441-87 6-3-87 34690-87 9-16-87

11667-87 4-15-87 34722-87 9-11-87

23294-87 7-7-87 43234-87 11-2-87

45478-87 11-16-87 44850-87 11-16-87

45691-87 12-2-87 44851-87 11-16-87

25464-87 7-16-87 46461-87 11-19-87

26483-87 7-23-87 46467-87 11-18-87

29950-87 8-11-87 48091-87 11-27-87[ 1 1 ]

Acting Chief of the Collection Division of the Bureau of Customs Augusto S. Danganan could not
authenticate the machine copies of the import entries as well, since the original copies of the said
entries filed with the Bureau of Customs had apparently been eaten by termites. However, he issued a
certification that the following enumerated entries were filed by the respondent which were processed
and released from the Port of Manila after payment of duties and taxes, to wit:

Hantex Trading Co., Inc.

Entry No. Date Released Entry No. Date Released

03903 1-29-87 22869 4-8-87

04414 1-20-87 19441 3-31-87

10683 2-17-87 24189 4-21-87


18
12611 2-24-87 26431 4-20-87

12989 2-26-87 45478 7-3-87

17050 3-13-87 26796 4-23-87

17169 3-13-87 28827 4-30-87

18089 3-16-87 31617 5-14-87

19439 4-1-87 39068 6-5-87

21189 4-3-87 42581 6-21-87

43451 6-29-87 42793 6-23-87

42795 6-23-87 45477 7-3-87

35582 not received 85830 11-13-87

45691 7-3-87 86650 not received

46187 7-8-87 87647 11-18-87

46427 7-3-87 88829 11-23-87

57669 8-12-87 92293 12-3-87

62471 8-28-87 93292 12-7-87

63187 9-2-87 96357 12-16-87

66859 9-15-87 96822 12-15-87

67890 9-17-87 98823 not received

68115 9-15-87 99428 12-28-87

69974 9-24-87 99429 12-28-87

72213 10-2-87 99441 12-28-87

77688 10-16-87 101406 1-5-87

84253 11-10-87 101407 1-8-87

19
85534 11-11-87 03118 1-19-87[ 1 2 ]

Bienvenido G. Flores, Chief of the Investigation Division, and Lt. Leo Dionela, Lt. Vicente Amoto
and Lt. Rolando Gatmaitan conducted an investigation. They relied on the certified copies of the
respondents Profit and Loss Statement for 1987 and 1988 on file with the SEC, the machine copies of
the Consumption Entries, Series of 1987, submitted by the informer, as well as excerpts from the
entries certified by Tomas and Danganan.
Based on the documents/records on hand, inclusive of the machine copies of the Consumption
Entries, the EIIB found that for 1987, the respondent had importations totaling P105,716,527.00
(inclusive of advance sales tax). Compared with the declared sales based on the Profit and Loss
Statements filed with the SEC, the respondent had unreported sales in the amount of P63,032,989.17,
and its corresponding income tax liability was P41,916,937.78, inclusive of penalty charge and
interests.
EIIB Commissioner Almonte transmitted the entire docket of the case to the BIR and
recommended the collection of the total tax assessment from the respondent.[ 1 3 ]
On February 12, 1991, Deputy Commissioner Deoferio, Jr. issued a Memorandum to the BIR
Assistant Commissioner for Special Operations Service, directing the latter to prepare a conference
letter advising the respondent of its deficiency taxes.[ 1 4 ]
Meanwhile, as ordered by the Regional Director, Revenue Enforcement Officers Saturnino D.
Torres and Wilson Filamor conducted an investigation on the 1987 importations of the respondent, in
the light of the records elevated by the EIIB to the BIR, inclusive of the photocopies of the
Consumption Entries. They were to ascertain the respondents liability for deficiency sales and income
taxes for 1987, if any. Per Torres and Filamors Report dated March 6, 1991 which was based on the
report of the EIIB and the documents/records appended thereto, there was a prima facie case of fraud
against the respondent in filing its 1987 Consumption Entry reports with the Bureau of Customs. They
found that the respondent had unrecorded importation in the total amount of P70,661,694.00, and that
the amount was not declared in its income tax return for 1987. The District Revenue Officer and the
Regional Director of the BIR concurred with the report.[ 1 5 ]
Based on the said report, the Acting Chief of the Special Investigation Branch wrote the respondent
and invited its representative to a conference at 10:00 a.m. of March 14, 1991 to discuss its deficiency
internal revenue taxes and to present whatever documentary and other evidence to refute the
same.[ 1 6 ] Appended to the letter was a computation of the deficiency income and sales tax due from the
respondent, inclusive of increments:

B. Computations:

1. Cost of Sales Ratio A2/A1 85.492923%

2. Undeclared Sales Imported A3/B1 110,079,491.61

3. Undeclared Gross Profit B2-A3 15,969,316.61

C. Deficiency Taxes Due:


20
1. Deficiency Income Tax B3 x 35% 5,589,261.00

50% Surcharge C1 x 50% 2,794,630.50

Interest to 2/28/91 C1 x 57.5% 3,213,825.08

Total 11,597,825.58

2. Deficiency Sales Tax

at 10% 7,290,082.72

at 20% 10,493,312.31

Total Due 17,783,395.03

Less: Advanced Sales Taxes Paid 11,636,352.00

Deficiency Sales Tax 6,147,043.03

50% Surcharge C2 x 50% 3,073,521.52

Interest to 2/28/91 5,532,338.73

Total 14,752,903.28[ 1 7 ]

===========

The invitation was reiterated in a Letter dated March 15, 1991. In his Reply dated March 15, 1991,
Mariano O. Chua, the President and General Manager of the respondent, requested that the report of
Torres and Filamor be set aside on the following claim:

[W]e had already been investigated by RDO No. 23 under Letters of Authority Nos. 0322988 RR dated
Oct. 1, 1987, 0393561 RR dated Aug. 17, 1988 and 0347838 RR dated March 2, 1988, and re-
investigated by the Special Investigation Team on Aug. 17, 1988 under Letter of Authority No.
0357464 RR, and the Intelligence and Investigation Office on Sept. 27, 1988 under Letter of Authority
No. 0020188 NA, all for income and business tax liabilities for 1987. The Economic Intelligence and
Investigation Bureau on Nov. 20, 1989, likewise, confronted us on the same information for the same
year.

In all of these investigations, save your request for an informal conference, we welcomed them and
proved the contrary of the allegation. Now, with your new inquiry, we think that there will be no end to
the problem.

21
Madam, we had been subjected to so many investigations and re-investigations for 1987 and nothing
came out except the payment of deficiency taxes as a result of oversight. Tax evasion through
underdeclaration of income had never been proven.[ 1 8 ]

Invoking Section 235[ 1 9 ] of the 1977 National Internal Revenue Code (NIRC), as amended, Chua
requested that the inquiry be set aside.
The petitioner, the Commissioner of Internal Revenue, through Assistant Commissioner for
Collection Jaime M. Maza, sent a Letter dated April 15, 1991 to the respondent demanding payment of
its deficiency income tax of P13,414,226.40 and deficiency sales tax of P14,752,903.25, inclusive of
surcharge and interest.[ 2 0 ] Appended thereto were the Assessment Notices of Tax Deficiency Nos.
FAS-1-87-91-001654 and FAS-4-87-91-001655.[ 2 1 ]
On February 12, 1992, the Chief of the Accounts Receivables/Billing Division of the BIR sent a
letter to the respondent demanding payment of its tax liability due for 1987 within ten (10) days from
notice, on pain of the collection tax due via a warrant of distraint and levy and/or judicial
action.[ 2 2 ] The Warrant of Distraint and/or Levy[ 2 3 ] was actually served on the respondent on January
21, 1992. On September 7, 1992, it wrote the Commissioner of Internal Revenue protesting the
assessment on the following grounds:
I. THAT THE ASSESSMENT HAS NO FACTUAL AS WELL AS LEGAL BASIS, THE
FACT THAT NO INVESTIGATION OF OUR RECORDS WAS EVER MADE BY THE
EIIB WHICH RECOMMENDED ITS ISSUANCE.[ 2 4 ]
II. THAT GRANTING BUT WITHOUT ADMITTING THAT OUR PURCHASES FOR 1987
AMOUNTED TO P105,716,527.00 AS CLAIMED BY THE EIIB, THE ASSESSMENT
OF A DEFICIENCY INCOME TAX IS STILL DEFECTIVE FOR IT FAILED TO
CONSIDER OUR REAL PURCHASES OF P45,538,694.57.[ 2 5 ]
III. THAT THE ASSESSMENT OF A DEFICIENCY SALES TAX IS ALSO BASELESS
AND UNFOUNDED CONSIDERING THAT WE HAVE DUTIFULLY PAID THE
SALES TAX DUE FROM OUR BUSINESS.[ 2 6 ]
In view of the impasse, administrative hearings were conducted on the respondents protest to the
assessment. During the hearing of August 20, 1993, the IIPO representative presented the photocopies
of the Consumption and Import Entries and the Certifications issued by Tomas and Danganan of the
Bureau of Customs. The IIPO representative testified that the Bureau of Customs failed to furnish the
EIIB with certified copies of the Consumption and Import Entries; hence, the EIIB relied on the
machine copies from their informer.[ 2 7 ]
The respondent wrote the BIR Commissioner on July 12, 1993 questioning the assessment on the
ground that the EIIB representative failed to present the original, or authenticated, or duly certified
copies of the Consumption and Import Entry Accounts, or excerpts thereof if the original copies were
not readily available; or, if the originals were in the official custody of a public officer, certified copies
thereof as provided for in Section 12, Chapter 3, Book VII, Administrative Procedure, Administrative
Order of 1987. It stated that the only copies of the Consumption Entries submitted to the Hearing
Officer were mere machine copies furnished by an informer of the EIIB. It asserted that the letters of
Tomas and Danganan were unreliable because of the following:

22
In the said letters, the two collection officers merely submitted a listing of alleged import entry
numbers and dates released of alleged importations by Hantex Trading Co., Inc. of merchandise in
1987, for which they certified that the corresponding duties and taxes were paid after being processed
in their offices. In said letters, no amounts of the landed costs and advance sales tax and duties were
stated, and no particulars of the duties and taxes paid per import entry document was presented.

The contents of the two letters failed to indicate the particulars of the importations per entry number,
and the said letters do not constitute as evidence of the amounts of importations of Hantex Trading Co.,
Inc. in 1987.[ 2 8 ]

The respondent cited the following findings of the Hearing Officer:

[T]hat the import entry documents do not constitute evidence only indicate that the tax assessments in
question have no factual basis, and must, at this point in time, be withdrawn and cancelled. Any new
findings by the IIPO representative who attended the hearing could not be used as evidence in this
hearing, because all the issues on the tax assessments in question have already been raised by the
herein taxpayer.[ 2 9 ]

The respondent requested anew that the income tax deficiency assessment and the sales tax
deficiency assessment be set aside for lack of factual and legal basis.
The BIR Commissioner[ 3 0 ] wrote the respondent on December 10, 1993, denying its letter-request
for the dismissal of the assessments.[ 3 1 ] The BIR Commissioner admitted, in the said letter, the
possibility that the figures appearing in the photocopies of the Consumption Entries had been tampered
with. She averred, however, that she was not proscribed from relying on other admissible evidence,
namely, the Letters of Torres and Filamor dated August 7 and 22, 1990 on their investigation of the
respondents tax liability. The Commissioner emphasized that her decision was final.[ 3 2 ]
The respondent forthwith filed a petition for review in the CTA of the Commissioners Final
Assessment Letter dated December 10, 1993 on the following grounds:
First. The alleged 1987 deficiency income tax assessment (including increments) and the alleged
1987 deficiency sales tax assessment (including increments) are void ab initio, since under Sections
16(a) and 49(b) of the Tax Code, the Commissioner shall examine a return after it is filed and,
thereafter, assess the correct amount of tax. The following facts obtaining in this case, however, are
indicative of the incorrectness of the tax assessments in question: the deficiency interests imposed in
the income and percentage tax deficiency assessment notices were computed in violation of the
provisions of Section 249(b) of the NIRC of 1977, as amended; the percentage tax deficiency was
computed on an annual basis for the year 1987 in accordance with the provision of Section 193, which
should have been computed in accordance with Section 162 of the 1977 NIRC, as amended by Pres.
Decree No. 1994 on a quarterly basis; and the BIR official who signed the deficiency tax assessments
was the Assistant Commissioner for Collection, who had no authority to sign the same under the NIRC.
Second. Even granting arguendo that the deficiency taxes and increments for 1987 against the
respondent were correctly computed in accordance with the provisions of the Tax Code, the facts
indicate that the above-stated assessments were based on alleged documents which are inadmissible in
either administrative or judicial proceedings. Moreover, the alleged bases of the tax computations were
23
anchored on mere presumptions and not on actual facts. The alleged undeclared purchases for 1987
were based on mere photocopies of alleged import entry documents, not the original ones, and which
had never been duly certified by the public officer charged with the custody of such records in the
Bureau of Customs. According to the respondent, the alleged undeclared sales were computed based on
mere presumptions as to the alleged gross profit contained in its 1987 financial statement. Moreover,
even the alleged financial statement of the respondent was a mere machine copy and not an official
copy of the 1987 income and business tax returns. Finally, the respondent was following the accrual
method of accounting in 1987, yet, the BIR investigator who computed the 1987 income tax deficiency
failed to allow as a deductible item the alleged sales tax deficiency for 1987 as provided for under
Section 30(c) of the NIRC of 1986.[ 3 3 ]
The Commissioner did not adduce in evidence the original or certified true copies of the 1987
Consumption Entries on file with the Commission on Audit. Instead, she offered in evidence as proof
of the contents thereof, the photocopies of the Consumption Entries which the respondent objected to
for being inadmissible in evidence.[ 3 4 ] She also failed to present any witness to prove the correct
amount of tax due from it. Nevertheless, the CTA provisionally admitted the said documents in
evidence, subject to its final evaluation of their relevancy and probative weight to the issues
involved.[ 3 5 ]
On December 11, 1997, the CTA rendered a decision, the dispositive portion of which reads:

IN THE LIGHT OF ALL THE FOREGOING, judgment is hereby rendered DENYING the herein
petition. Petitioner is hereby ORDERED TO PAY the respondent Commissioner of Internal Revenue
its deficiency income and sales taxes for the year 1987 in the amounts of P11,182,350.26
and P12,660,382.46, respectively, plus 20% delinquency interest per annum on both deficiency taxes
from April 15, 1991 until fully paid pursuant to Section 283(c)(3) of the 1987 Tax Code, with costs
against the petitioner.

SO ORDERED.[ 3 6 ]

The CTA ruled that the respondent was burdened to prove not only that the assessment was
erroneous, but also to adduce the correct taxes to be paid by it. The CTA declared that the respondent
failed to prove the correct amount of taxes due to the BIR. It also ruled that the respondent was
burdened to adduce in evidence a certification from the Bureau of Customs that the Consumption
Entries in question did not belong to it.
On appeal, the CA granted the petition and reversed the decision of the CTA. The dispositive
portion of the decision reads:

FOREGOING PREMISES CONSIDERED, the Petition for Review is GRANTED and the December
11, 1997 decision of the CTA in CTA Case No. 5162 affirming the 1987 deficiency income and sales
tax assessments and the increments thereof, issued by the BIR is hereby REVERSED. No costs.[ 3 7 ]

The Ruling of the Court of Appeals

24
The CA held that the income and sales tax deficiency assessments issued by the petitioner were
unlawful and baseless since the copies of the import entries relied upon in computing the deficiency tax
of the respondent were not duly authenticated by the public officer charged with their custody, nor
verified under oath by the EIIB and the BIR investigators.[ 3 8 ] The CA also noted that the public officer
charged with the custody of the import entries was never presented in court to lend credence to the
alleged loss of the originals.[ 3 9 ] The CA pointed out that an import entry is a public document which
falls within the provisions of Section 19, Rule 132 of the Rules of Court, and to be admissible for any
legal purpose, Section 24, Rule 132 of the Rules of Court should apply.[ 4 0 ] Citing the ruling of this
Court in Collector of Internal Revenue v. Benipayo,[ 4 1 ] the CA ruled that the assessments were
unlawful because they were based on hearsay evidence. The CA also ruled that the respondent was
deprived of its right to due process of law.
The CA added that the CTA should not have just brushed aside the legal requisites provided for
under the pertinent provisions of the Rules of Court in the matter of the admissibility of public
documents, considering that substantive rules of evidence should not be disregarded. It also ruled that
the certifications made by the two Customs Collection Chiefs under the guise of supporting the
respondents alleged tax deficiency assessments invoking the best evidence obtainable rule under the
Tax Code should not be permitted to supplant the best evidence rule under Section 7, Rule 130 of the
Rules of Court.
Finally, the CA noted that the tax deficiency assessments were computed without the tax returns.
The CA opined that the use of the tax returns is indispensable in the computation of a tax deficiency;
hence, this essential requirement must be complied with in the preparation and issuance of valid tax
deficiency assessments.[ 4 2 ]

The Present Petition

The Commissioner of Internal Revenue, the petitioner herein, filed the present petition for review
under Rule 45 of the Rules of Court for the reversal of the decision of the CA and for the reinstatement
of the ruling of the CTA.
As gleaned from the pleadings of the parties, the threshold issues for resolution are the following:
(a) whether the petition at bench is proper and complies with Sections 4 and 5, Rule 7 of the Rules of
Court; (b) whether the December 10, 1991 final assessment of the petitioner against the respondent for
deficiency income tax and sales tax for the latters 1987 importation of resins and calcium bicarbonate
is based on competent evidence and the law; and (c) the total amount of deficiency taxes due from the
respondent for 1987, if any.
On the first issue, the respondent points out that the petition raises both questions of facts and law
which cannot be the subject of an appeal by certiorari under Rule 45 of the Rules of Court. The
respondent notes that the petition is defective because the verification and the certification against
forum shopping were not signed by the petitioner herself, but only by the Regional Director of the BIR.
The respondent submits that the petitioner should have filed a motion for reconsideration with the CA
before filing the instant petition for review.[ 4 3 ]

25
We find and so rule that the petition is sufficient in form. A verification and certification against
forum shopping signed by the Regional Director constitutes sufficient compliance with the
requirements of Sections 4 and 5, Rule 7 of the Rules of Court. Under Section 10 of the NIRC of
1997,[ 4 4 ] the Regional Director has the power to administer and enforce internal revenue laws, rules
and regulations, including the assessment and collection of all internal revenue taxes, charges and fees.
Such power is broad enough to vest the Revenue Regional Director with the authority to sign the
verification and certification against forum shopping in behalf of the Commissioner of Internal
Revenue. There is no other person in a better position to know the collection cases filed under his
jurisdiction than the Revenue Regional Director.
Moreover, under Revenue Administrative Order No. 5-83,[ 4 5 ] the Regional Director is authorized
to sign all pleadings filed in connection with cases referred to the Revenue Regions by the National
Office which, otherwise, require the signature of the petitioner.
We do not agree with the contention of the respondent that a motion for reconsideration ought to
have been filed before the filing of the instant petition. A motion for reconsideration of the decision of
the CA is not a condition sine qua non for the filing of a petition for review under Rule 45. As we held
in Almora v. Court of Appeals:[ 4 6 ]

Rule 45, Sec. 1 of the Rules of Court, however, distinctly provides that:

A party may appeal by certiorari from a judgment of the Court of Appeals, by filing with the Supreme
Court a petition for certiorari within fifteen (15) days from notice of judgment, or of the denial of his
motion for reconsideration filed in due time. (Emphasis supplied)

The conjunctive or clearly indicates that the 15-day reglementary period for the filing of a petition for
certiorari under Rule 45 commences either from notice of the questioned judgment or from notice of
denial of the appellants motion for reconsideration. A prior motion for reconsideration is not
indispensable for a petition for review on certiorari under Rule 45 to prosper. [ 4 7 ]

While Rule 45 of the Rules of Court provides that only questions of law may be raised by the
petitioner and resolved by the Court, under exceptional circumstances, the Court may take cognizance
thereof and resolve questions of fact. In this case, the findings and conclusion of the CA are
inconsistent with those of the CTA, not to mention those of the Commissioner of Internal Revenue.
The issues raised in this case relate to the propriety and the correctness of the tax assessments made by
the petitioner against the respondent, as well as the propriety of the application of Section 16,
paragraph (b) of the 1977 NIRC, as amended by Pres. Decree Nos. 1705, 1773, 1994 and Executive
Order No. 273, in relation to Section 3, Rule 132 of the Rules of Evidence. There is also an imperative
need for the Court to resolve the threshold factual issues to give justice to the parties, and to determine
whether the CA capriciously ignored, misunderstood or misinterpreted cogent facts and circumstances
which, if considered, would change the outcome of the case.
On the second issue, the petitioner asserts that since the respondent refused to cooperate and show
its 1987 books of account and other accounting records, it was proper for her to resort to the best
evidence obtainable the photocopies of the import entries in the Bureau of Customs and the
respondents financial statement filed with the SEC.[ 4 8 ] The petitioner maintains that these import
entries were admissible as secondary evidence under the best evidence obtainable rule, since they were
26
duly authenticated by the Bureau of Customs officials who processed the documents and released the
cargoes after payment of the duties and taxes due.[ 4 9 ] Further, the petitioner points out that under the
best evidence obtainable rule, the tax return is not important in computing the tax deficiency. [ 5 0 ]
The petitioner avers that the best evidence obtainable rule under Section 16 of the 1977 NIRC, as
amended, legally cannot be equated to the best evidence rule under the Rules of Court; nor can the best
evidence rule, being procedural law, be made strictly operative in the interpretation of the best evidence
obtainable rule which is substantive in character.[ 5 1 ] The petitioner posits that the CTA is not strictly
bound by technical rules of evidence, the reason being that the quantum of evidence required in the
said court is merely substantial evidence.[ 5 2 ]
Finally, the petitioner avers that the respondent has the burden of proof to show the correct
assessments; otherwise, the presumption in favor of the correctness of the assessments made by it
stands.[ 5 3 ] Since the respondent was allowed to explain its side, there was no violation of due
process.[ 5 4 ]
The respondent, for its part, maintains that the resort to the best evidence obtainable method was
illegal. In the first place, the respondent argues, the EIIB agents are not duly authorized to undertake
examination of the taxpayers accounting records for internal revenue tax purposes. Hence, the
respondents failure to accede to their demands to show its books of accounts and other accounting
records cannot justify resort to the use of the best evidence obtainable method.[ 5 5 ] Secondly, when a
taxpayer fails to submit its tax records upon demand by the BIR officer, the remedy is not to assess him
and resort to the best evidence obtainable rule, but to punish the taxpayer according to the provisions of
the Tax Code.[ 5 6 ]
In any case, the respondent argues that the photocopies of import entries cannot be used in making
the assessment because they were not properly authenticated, pursuant to the provisions of Sections
24[ 5 7 ] and 25[ 5 8 ] of Rule 132 of the Rules of Court. It avers that while the CTA is not bound by the
technical rules of evidence, it is bound by substantial rules.[ 5 9 ] The respondent points out that the
petitioner did not even secure a certification of the fact of loss of the original documents from the
custodian of the import entries. It simply relied on the report of the EIIB agents that the import entry
documents were no longer available because they were eaten by termites. The respondent posits that
the two collectors of the Bureau of Customs never authenticated the xerox copies of the import entries;
instead, they only issued certifications stating therein the import entry numbers which were processed
by their office and the date the same were released.[ 6 0 ]
The respondent argues that it was not necessary for it to show the correct assessment, considering
that it is questioning the assessments not only because they are erroneous, but because they were issued
without factual basis and in patent violation of the assessment procedures laid down in the NIRC of
1977, as amended.[ 6 1 ] It is also pointed out that the petitioner failed to use the tax returns filed by the
respondent in computing the deficiency taxes which is contrary to law;[ 6 2 ] as such, the deficiency
assessments constituted deprivation of property without due process of law.[ 6 3 ]
Central to the second issue is Section 16 of the NIRC of 1977, as amended,[ 6 4 ] which provides that
the Commissioner of Internal Revenue has the power to make assessments and prescribe additional
requirements for tax administration and enforcement. Among such powers are those provided in
paragraph (b) thereof, which we quote:

27
(b) Failure to submit required returns, statements, reports and other documents. When a report required
by law as a basis for the assessment of any national internal revenue tax shall not be forthcoming
within the time fixed by law or regulation or when there is reason to believe that any such report is
false, incomplete or erroneous, the Commissioner shall assess the proper tax on the best evidence
obtainable.

In case a person fails to file a required return or other document at the time prescribed by law,
or willfully or otherwise files a false or fraudulent return or other document, the Commissioner shall
make or amend the return from his own knowledge and from such information as he can obtain
through testimony or otherwise, which shall be prima facie correct and sufficient for all legal
purposes.[ 6 5 ]

This provision applies when the Commissioner of Internal Revenue undertakes to perform her
administrative duty of assessing the proper tax against a taxpayer, to make a return in case of a
taxpayers failure to file one, or to amend a return already filed in the BIR.
The petitioner may avail herself of the best evidence or other information or testimony by
exercising her power or authority under paragraphs (1) to (4) of Section 7 of the NIRC:

(1) To examine any book, paper, record or other data which may be relevant or material to such
inquiry;

(2) To obtain information from any office or officer of the national and local governments, government
agencies or its instrumentalities, including the Central Bank of the Philippines and government owned
or controlled corporations;

(3) To summon the person liable for tax or required to file a return, or any officer or employee of such
person, or any person having possession, custody, or care of the books of accounts and other
accounting records containing entries relating to the business of the person liable for tax, or any other
person, to appear before the Commissioner or his duly authorized representative at a time and place
specified in the summons and to produce such books, papers, records, or other data, and to give
testimony;

(4) To take such testimony of the person concerned, under oath, as may be relevant or material to such
inquiry; [ 6 6 ]

The best evidence envisaged in Section 16 of the 1977 NIRC, as amended, includes the corporate
and accounting records of the taxpayer who is the subject of the assessment process, the accounting
records of other taxpayers engaged in the same line of business, including their gross profit and net
profit sales.[ 6 7 ] Such evidence also includes data, record, paper, document or any evidence gathered by
internal revenue officers from other taxpayers who had personal transactions or from whom the subject
taxpayer received any income; and record, data, document and information secured from government
offices or agencies, such as the SEC, the Central Bank of the Philippines, the Bureau of Customs, and
the Tariff and Customs Commission.

28
The law allows the BIR access to all relevant or material records and data in the person of the
taxpayer. It places no limit or condition on the type or form of the medium by which the record subject
to the order of the BIR is kept. The purpose of the law is to enable the BIR to get at the taxpayers
records in whatever form they may be kept. Such records include computer tapes of the said records
prepared by the taxpayer in the course of business.[ 6 8 ] In this era of developing information-storage
technology, there is no valid reason to immunize companies with computer-based, record-keeping
capabilities from BIR scrutiny. The standard is not the form of the record but where it might shed light
on the accuracy of the taxpayers return.
In Campbell, Jr. v. Guetersloh,[ 6 9 ] the United States (U.S.) Court of Appeals (5th Circuit) declared
that it is the duty of the Commissioner of Internal Revenue to investigate any circumstance which led
him to believe that the taxpayer had taxable income larger than reported. Necessarily, this inquiry
would have to be outside of the books because they supported the return as filed. He may take the
sworn testimony of the taxpayer; he may take the testimony of third parties; he may examine and
subpoena, if necessary, traders and brokers accounts and books and the taxpayers book accounts. The
Commissioner is not bound to follow any set of patterns. The existence of unreported income may be
shown by any practicable proof that is available in the circumstances of the particular situation. Citing
its ruling in Kenney v. Commissioner,[ 7 0 ] the U.S. appellate court declared that where the records of the
taxpayer are manifestly inaccurate and incomplete, the Commissioner may look to other sources of
information to establish income made by the taxpayer during the years in question.[ 7 1 ]
We agree with the contention of the petitioner that the best evidence obtainable may consist of
hearsay evidence, such as the testimony of third parties or accounts or other records of other taxpayers
similarly circumstanced as the taxpayer subject of the investigation, hence, inadmissible in a regular
proceeding in the regular courts.[ 7 2 ] Moreover, the general rule is that administrative agencies such as
the BIR are not bound by the technical rules of evidence. It can accept documents which cannot be
admitted in a judicial proceeding where the Rules of Court are strictly observed. It can choose to give
weight or disregard such evidence, depending on its trustworthiness.
However, the best evidence obtainable under Section 16 of the 1977 NIRC, as amended, does not
include mere photocopies of records/documents. The petitioner, in making a preliminary and final tax
deficiency assessment against a taxpayer, cannot anchor the said assessment on mere machine copies of
records/documents. Mere photocopies of the Consumption Entries have no probative weight if offered
as proof of the contents thereof. The reason for this is that such copies are mere scraps of paper and are
of no probative value as basis for any deficiency income or business taxes against a taxpayer. Indeed,
in United States v. Davey,[ 7 3 ] the U.S. Court of Appeals (2nd Circuit) ruled that where the accuracy of
a taxpayers return is being checked, the government is entitled to use the original records rather than be
forced to accept purported copies which present the risk of error or tampering.[ 7 4 ]
In Collector of Internal Revenue v. Benipayo,[ 7 5 ] the Court ruled that the assessment must be based
on actual facts. The rule assumes more importance in this case since the xerox copies of the
Consumption Entries furnished by the informer of the EIIB were furnished by yet another informer.
While the EIIB tried to secure certified copies of the said entries from the Bureau of Customs, it was
unable to do so because the said entries were allegedly eaten by termites. The Court can only surmise
why the EIIB or the BIR, for that matter, failed to secure certified copies of the said entries from the
Tariff and Customs Commission or from the National Statistics Office which also had copies thereof. It
bears stressing that under Section 1306 of the Tariff and Customs Code, the Consumption Entries shall
29
be the required number of copies as prescribed by regulations.[ 7 6 ] The Consumption Entry is
accomplished in sextuplicate copies and quadruplicate copies in other places. In Manila, the six copies
are distributed to the Bureau of Customs, the Tariff and Customs Commission, the Declarant
(Importer), the Terminal Operator, and the Bureau of Internal Revenue. Inexplicably, the
Commissioner and the BIR personnel ignored the copy of the Consumption Entries filed with the BIR
and relied on the photocopies supplied by the informer of the EIIB who secured the same from another
informer. The BIR, in preparing and issuing its preliminary and final assessments against the
respondent, even ignored the records on the investigation made by the District Revenue officers on the
respondents importations for 1987.
The original copies of the Consumption Entries were of prime importance to the BIR. This is so
because such entries are under oath and are presumed to be true and correct under penalty of
falsification or perjury. Admissions in the said entries of the importers documents are admissions
against interest and presumptively correct.[ 7 7 ]
In fine, then, the petitioner acted arbitrarily and capriciously in relying on and giving weight to the
machine copies of the Consumption Entries in fixing the tax deficiency assessments against the
respondent.
The rule is that in the absence of the accounting records of a taxpayer, his tax liability may be
determined by estimation. The petitioner is not required to compute such tax liabilities with
mathematical exactness. Approximation in the calculation of the taxes due is justified. To hold
otherwise would be tantamount to holding that skillful concealment is an invincible barrier to
proof.[ 7 8 ] However, the rule does not apply where the estimation is arrived at arbitrarily and
capriciously.[ 7 9 ]
We agree with the contention of the petitioner that, as a general rule, tax assessments by tax
examiners are presumed correct and made in good faith. All presumptions are in favor of the
correctness of a tax assessment. It is to be presumed, however, that such assessment was based on
sufficient evidence. Upon the introduction of the assessment in evidence, a prima facie case of liability
on the part of the taxpayer is made.[ 8 0 ] If a taxpayer files a petition for review in the CTA and assails
the assessment, the prima facie presumption is that the assessment made by the BIR is correct, and that
in preparing the same, the BIR personnel regularly performed their duties. This rule for tax initiated
suits is premised on several factors other than the normal evidentiary rule imposing proof obligation on
the petitioner-taxpayer: the presumption of administrative regularity; the likelihood that the taxpayer
will have access to the relevant information; and the desirability of bolstering the record-keeping
requirements of the NIRC.[ 8 1 ]
However, the prima facie correctness of a tax assessment does not apply upon proof that an
assessment is utterly without foundation, meaning it is arbitrary and capricious. Where the BIR has
come out with a naked assessment, i.e., without any foundation character, the determination of the tax
due is without rational basis.[ 8 2 ] In such a situation, the U.S. Court of Appeals ruled[ 8 3 ] that the
determination of the Commissioner contained in a deficiency notice disappears. Hence, the
determination by the CTA must rest on all the evidence introduced and its ultimate determination must
find support in credible evidence.
The issue that now comes to fore is whether the tax deficiency assessment against the respondent
based on the certified copies of the Profit and Loss Statement submitted by the respondent to the SEC
30
in 1987 and 1988, as well as certifications of Tomas and Danganan, is arbitrary, capricious and illegal.
The CTA ruled that the respondent failed to overcome the prima facie correctness of the tax deficiency
assessment issued by the petitioner, to wit:

The issue should be ruled in the affirmative as petitioner has failed to rebut the validity or correctness
of the aforementioned tax assessments. It is incongruous for petitioner to prove its cause by simply
drawing an inference unfavorable to the respondent by attacking the source documents (Consumption
Entries) which were the bases of the assessment and which were certified by the Chiefs of the
Collection Division, Manila International Container Port and the Port of Manila, as having been
processed and released in the name of the petitioner after payment of duties and taxes and the duly
certified copies of Financial Statements secured from the Securities and Exchange Commission. Any
such inference cannot operate to relieve petitioner from bearing its burden of proof and this Court has
no warrant of absolution. The Court should have been persuaded to grant the reliefs sought by the
petitioner should it have presented any evidence of relevance and competence required, like that of a
certification from the Bureau of Customs or from any other agencies, attesting to the fact that those
consumption entries did not really belong to them.

The burden of proof is on the taxpayer contesting the validity or correctness of an assessment to prove
not only that the Commissioner of Internal Revenue is wrong but the taxpayer is right (Tan Guan v.
CTA, 19 SCRA 903), otherwise, the presumption in favor of the correctness of tax assessment stands
(Sy Po v. CTA, 164 SCRA 524). The burden of proving the illegality of the assessment lies upon the
petitioner alleging it to be so. In the case at bar, petitioner miserably failed to discharge this duty.[ 8 4 ]

We are not in full accord with the findings and ratiocination of the CTA. Based on the letter of the
petitioner to the respondent dated December 10, 1993, the tax deficiency assessment in question was
based on (a) the findings of the agents of the EIIB which was based, in turn, on the photocopies of the
Consumption Entries; (b) the Profit and Loss Statements of the respondent for 1987 and 1988; and (c)
the certifications of Tomas and Danganan dated August 7, 1990 and August 22, 1990:

In reply, please be informed that after a thorough evaluation of the attending facts, as well as the laws
and jurisprudence involved, this Office holds that you are liable to the assessed deficiency taxes. The
conclusion was arrived at based on the findings of agents of the Economic Intelligence & Investigation
Bureau (EIIB) and of our own examiners who have painstakingly examined the records furnished by
the Bureau of Customs and the Securities & Exchange Commission (SEC). The examination conducted
disclosed that while your actual sales for 1987 amounted to P110,731,559.00, you declared for taxation
purposes, as shown in the Profit and Loss Statements, the sum of P47,698,569.83 only. The difference,
therefore, of P63,032,989.17 constitutes as undeclared or unrecorded sales which must be subjected to
the income and sales taxes.

You also argued that our assessment has no basis since the alleged amount of underdeclared
importations were lifted from uncertified or unauthenticated xerox copies of consumption entries which
are not admissible in evidence. On this issue, it must be considered that in letters dated August 7 and
22, 1990, the Chief and Acting Chief of the Collection Division of the Manila International Container
Port and Port of Manila, respectively, certified that the enumerated consumption entries were filed,
processed and released from the port after payment of duties and taxes. It is noted that the certification
31
does not touch on the genuineness, authenticity and correctness of the consumption entries which are
all xerox copies, wherein the figures therein appearing may have been tampered which may render said
documents inadmissible in evidence, but for tax purposes, it has been held that the Commissioner is not
required to make his determination (assessment) on the basis of evidence legally admissible in a formal
proceeding in Court (Mertens, Vol. 9, p. 214, citing Cohen v. Commissioner). A statutory notice may
be based in whole or in part upon admissible evidence (Llorente v. Commissioner, 74 TC 260
(1980); Weimerskirch v. Commissioner, 67 TC 672 (1977); and Rosano v. Commissioner, 46 TC 681
(1966). In the case also of Weimerskirch v. Commissioner (1977), the assessment was given due course
in the presence of admissible evidence as to how the Commissioner arrived at his determination,
although there was no admissible evidence with respect to the substantial issue of whether the taxpayer
had unreported or undeclared income from narcotics sale. [ 8 5 ]

Based on a Memorandum dated October 23, 1990 of the IIPO, the source documents for the actual
cost of importation of the respondent are the machine copies of the Consumption Entries from the
informer which the IIPO claimed to have been certified by Tomas and Danganan:

The source documents for the total actual cost of importations, abovementioned, were the different
copies of Consumption Entries, Series of 1987, filed by subject with the Bureau of Customs, marked
Annexes F-1 to F-68. The total cost of importations is the sum of the Landed Costs and the Advance
Sales Tax as shown in the annexed entries. These entries were duly authenticated as having been
processed and released, after payment of the duties and taxes due thereon, by the Chief, Collection
Division, Manila International Container Port, dated August 7, 1990, Annex-G, and the Port of Manila,
dated August 22, 1990, Annex-H. So, it was established that subject-importations, mostly resins, really
belong to HANTEX TRADING CO., INC.[ 8 6 ]

It also appears on the worksheet of the IIPO, as culled from the photocopies of the Consumption
Entries from its informer, that the total cost of the respondents importation for 1987
was P105,761,527.00. Per the report of Torres and Filamor, they also relied on the photocopies of the
said Consumption Entries:

The importations made by taxpayer verified by us from the records of the Bureau of Customs and
xerox copies of which are hereto attached shows the big volume of importations made and not declared
in the income tax return filed by taxpayer.

Based on the above findings, it clearly shows that a prima facie case of fraud exists in the herein
transaction of the taxpayer, as a consequence of which, said transaction has not been possibly entered
into the books of accounts of the subject taxpayer.[ 8 7 ]

In fine, the petitioner based her finding that the 1987 importation of the respondent was
underdeclared in the amount of P105,761,527.00 on the worthless machine copies of the Consumption
Entries. Aside from such copies, the petitioner has no other evidence to prove that the respondent
imported goods costing P105,761,527.00. The petitioner cannot find solace on the certifications of
Tomas and Danganan because they did not authenticate the machine copies of the Consumption
Entries, and merely indicated therein the entry numbers of Consumption Entries and the dates when the
Bureau of Customs released the same. The certifications of Tomas and Danganan do not even contain

32
the landed costs and the advance sales taxes paid by the importer, if any. Comparing the certifications
of Tomas and Danganan and the machine copies of the Consumption Entries, only 36 of the entry
numbers of such copies are included in the said certifications; the entry numbers of the rest of the
machine copies of the Consumption Entries are not found therein.
Even if the Court would concede to the petitioners contention that the certification of Tomas and
Danganan authenticated the machine copies of the Consumption Entries referred to in the certification,
it appears that the total cost of importations inclusive of advance sales tax is only P64,324,953.00 far
from the amount of P105,716,527.00 arrived at by the EIIB and the BIR,[ 8 8 ] or even the amount
of P110,079,491.61 arrived at by Deputy Commissioner Deoferio, Jr.[ 8 9 ] As gleaned from the
certifications of Tomas and Danganan, the goods covered by the Consumption Entries were released by
the Bureau of Customs, from which it can be presumed that the respondent must have paid the taxes
due on the said importation. The petitioner did not adduce any documentary evidence to prove
otherwise.
Thus, the computations of the EIIB and the BIR on the quantity and costs of the importations of the
respondent in the amount of P105,761,527.00 for 1987 have no factual basis, hence, arbitrary and
capricious. The petitioner cannot rely on the presumption that she and the other employees of the BIR
had regularly performed their duties. As the Court held in Collector of Internal Revenue v.
Benipayo,[ 9 0 ] in order to stand judicial scrutiny, the assessment must be based on facts. The
presumption of the correctness of an assessment, being a mere presumption, cannot be made to rest on
another presumption.
Moreover, the uncontroverted fact is that the BIR District Revenue Office had repeatedly examined
the 1987 books of accounts of the respondent showing its importations, and found that the latter had
minimal business tax liability. In this case, the presumption that the District Revenue officers
performed their duties in accordance with law shall apply. There is no evidence on record that the said
officers neglected to perform their duties as mandated by law; neither is there evidence aliunde that the
contents of the 1987 and 1988 Profit and Loss Statements submitted by the respondent with the SEC
are incorrect.
Admittedly, the respondent did not adduce evidence to prove its correct tax liability. However,
considering that it has been established that the petitioners assessment is barren of factual basis,
arbitrary and illegal, such failure on the part of the respondent cannot serve as a basis for a finding by
the Court that it is liable for the amount contained in the said assessment; otherwise, the Court would
thereby be committing a travesty.
On the disposition of the case, the Court has two options, namely, to deny the petition for lack of
merit and affirm the decision of the CA, without prejudice to the petitioners issuance of a new
assessment against the respondent based on credible evidence; or, to remand the case to the CTA for
further proceedings, to enable the petitioner to adduce in evidence certified true copies or duplicate
original copies of the Consumption Entries for the respondents 1987 importations, if there be any, and
the correct tax deficiency assessment thereon, without prejudice to the right of the respondent to
adduce controverting evidence, so that the matter may be resolved once and for all by the CTA. In the
higher interest of justice to both the parties, the Court has chosen the latter option. After all, as the Tax
Court of the United States emphasized in Harbin v. Commissioner of Internal Revenue,[ 9 1 ] taxation is
not only practical; it is vital. The obligation of good faith and fair dealing in carrying out its provision

33
is reciprocal and, as the government should never be over-reaching or tyrannical, neither should a
taxpayer be permitted to escape payment by the concealment of material facts.
IN LIGHT OF ALL THE FOREGOING, the petition is GRANTED. The Decision of the Court
of Appeals is SET ASIDE. The records are REMANDED to the Court of Tax Appeals for further
proceedings, conformably with the decision of this Court. No costs. SO ORDERED.

DIGEST:
CIR vs. HANTEX TRADING CO., INC.
G.R. No. 136975; March 31, 2005

Facts: Hantex Trading Co is a company organized under the Philippines. It is engaged in the sale of
plastic products, it imports synthetic resin and other chemicals for the manufacture of its products. For
this purpose, it is required to file an Import Entry and Internal Revenue Declaration (Consumption
Entry) with the Bureau of Customs under Section 1301 of the Tariff and Customs Code. Sometime in
October 1989, Lt. Vicente Amoto, Acting Chief of Counter-Intelligence Division of the Economic
Intelligence and Investigation Bureau (EIIB), received confidential information that the respondent had
imported synthetic resin amounting to P115,599,018.00 but only declared P45,538,694.57. Thus,
Hentex receive a subpoena to present its books of account which it failed to do. The bureau cannot find
any original copies of the products Hentex imported since the originals were eaten by termites. Thus,
the Bureau relied on the certified copies of the respondent’s Profit and Loss Statement for 1987 and
1988 on file with the SEC, the machine copies of the Consumption Entries, Series of 1987, submitted
by the informer, as well as excerpts from the entries certified by Tomas and Danganan. The case was
submitted to the CTA which ruled that Hentex have tax deficiency and is ordered to pay, per
investigation of the Bureau. The CA ruled that the income and sales tax deficiency assessments issued
by the petitioner were unlawful and baseless since the copies of the import entries relied upon in
computing the deficiency tax of the respondent were not duly authenticated by the public officer
charged with their custody, nor verified under oath by the EIIB and the BIR investigators.

Issue: Whether or not the final assessment of the petitioner against the respondent for deficiency
income tax and sales tax for the latter’s 1987 importation of resins and calcium bicarbonate is based on
competent evidence and the law.

Held: Central to the second issue is Section 16 of the NIRC of 1977, as amended which provides that
the Commissioner of Internal Revenue has the power to make assessments and prescribe additional
requirements for tax administration and enforcement. Among such powers are those provided in
paragraph (b), which provides that “Failure to submit required returns, statements, reports and other
documents. – When a report required by law as a basis for the assessment of any national internal
revenue tax shall not be forthcoming within the time fixed by law or regulation or when there is reason
to believe that any such report is false, incomplete or erroneous, the Commissioner shall assess the
proper tax on the best evidence obtainable.” This provision applies when the Commissioner of Internal
Revenue undertakes to perform her administrative duty of assessing the proper tax against a taxpayer,
to make a return in case of a taxpayer’s failure to file one, or to amend a return already filed in the BIR.
The “best evidence” envisaged in Section 16 of the 1977 NIRC, as amended, includes the corporate
34
and accounting records of the taxpayer who is the subject of the assessment process, the accounting
records of other taxpayers engaged in the same line of business, including their gross profit and net
profit sales. Such evidence also includes data, record, paper, document or any evidence gathered by
internal revenue officers from other taxpayers who had personal transactions or from whom the subject
taxpayer received any income; and record, data, document and information secured from government
offices or agencies, such as the SEC, the Central Bank of the Philippines, the Bureau of Customs, and
the Tariff and Customs Commission. However, the best evidence obtainable under Section 16 of the
1977 NIRC, as amended, does not include mere photocopies of records/documents. The petitioner, in
making a preliminary and final tax deficiency assessment against a taxpayer, cannot anchor the said
assessment on mere machine copies of records/documents. Mere photocopies of the Consumption
Entries have no probative weight if offered as proof of the contents thereof. The reason for this is that
such copies are mere scraps of paper and are of no probative value as basis for any deficiency income
or business taxes against a taxpayer.

3. POWER/REMEDY OF ASSESSMENT

G.R. No. 81446 August 18, 1988

BONIFACIA SY PO, petitioner,


vs.
HONORABLE COURT OF TAX APPEALS AND HONORABLE COMMISSIONER OF
INTERNAL REVENUE, respondents.

SARMIENTO, J.:

This is an appeal from the decision 1 of the respondent Court of Tax Appeals, dated September
30,1987, which affirmed an earlier decision of the correspondent Commissioner of Internal Revenue in
assessment letters dated August 16, 1972 and September 26, 1972, which ordered the payment by the
petitioner of deficiency income tax for 1966 to 1970 in the amount of P7,154,685.16 and deficiency
specific tax for January 2, 1964 to January 19, 1972, in the amount of P5,595,003.68.

We adopt the respondent court's finding of facts, to wit:

Petitioner is the widow of the late Mr. Po Bien Sing who died on September 7, 1980. In
the taxable years 1964 to 1972, the deceased Po Bien Sing was the sole proprietor of
Silver Cup Wine Factory (Silver Cup for brevity), Talisay, Cebu. He was engaged in the
business of manufacture and sale of compounded liquors, using alcohol and other
ingredients as raw materials.

On the basis of a denunciation against Silver Cup allegedly "for tax evasion amounting to
millions of pesos" the then Secretary of Finance Cesar Virata directed the Finance-BIR--
NBI team constituted under Finance Department Order No. 13-70 dated February 19,
1971 (Exh- 3, pp. 532-553, Folder II, BIR rec.) to conduct the corresponding
investigation in a memorandum dated April 2, 1971 (p. 528, Folder II, BIR rec.).

35
Accordingly, a letter and a subpoena duces tecum dated April 13,1971 and May 3,1971,
respectively, were issued against Silver Cup requesting production of the accounting
records and other related documents for the examination of the team. (Exh. 11, pp. 525-
526, Folder II, BIR rec.). Mr. Po Bien Sing did not produce his books of accounts as
requested (Affidavit dated December 24, 1971 of Mr. Generoso. Quinain of the team, p.
525, Folder H, BIR rec.). This prompted the team with the assistance of the PC Company,
Cebu City, to enter the factory bodega of Silver Cup and seized different brands,
consisting of 1,555 cases of alcohol products. (Exh. 22, Memorandum Report of the
Team dated June 5, 1971, pp. 491-492, Folder II, BIR rec.). The inventory lists of the
seized alcohol products are contained in Volumes I, II, III, IV and V (Exhibits 14, 15, 16,
17, and 18, respectively, BIR rec.). On the basis of the team's report of investigation, the
respondent Commissioner of Internal Revenue assessed Mr. Po Bien Sing deficiency
income tax for 1966 to 1970 in the amount of P7,154,685.16 (Exh. 6 pp. 17-19, Folder I,
BIR rec.) and for deficiency specific tax for January 2,1964 to January 19, 1972 in the
amount of P5,595,003.68 (Exh. 8, p. 107, Folder I, BIR rec.).

Petitioner protested the deficiency assessments through letters dated October 9 and
October 30, 1972 (Exhs. 7 and 9, pp. 27-28; pp. 152-159, respectively, BIR rec.), which
protests were referred for reinvestigation. The corresponding report dated August 13,
1981 (Exh. 1 0, pp. 355, Folder I, BIR rec.) recommended the reiteration of the
assessments in view of the taxpayer's persistent failure to present the books of accounts
for examination (Exh. 8, p. 107, Folder I, BIR rec.), compelling respondent to issue
warrants of distraint and levy on September 10, 1981 (Exh. 11, p. 361, Folder I, BIR
rec.).

The warrants were admittedly received by petitioner on October 14, 1981 (Par. IX,
Petition; admitted par. 2, Answer), which petitioner deemed respondent's decision
denying her protest on the subject assessments. Hence, petitioner's appeal on October
29,1981. 2

The petitioner assigns the following errors:

RESPONDENT INTENTIONALLY ERRED IN HOLDING THAT PETITIONER HAS NOT


PRESENTED ANY EVIDENCE OF RELEVANCE AND COMPETENCE REQUIRED TO BASH
THE TROUBLING DISCREPANCIES AND SQUARE THE ISSUE OF ILLEGALITY POSITED
ON THE SUBJECT ASSESSMENTS.

II

RESPONDENT COURT OF TAX APPEALS PALPABLY ERRED IN DECIDING THE CASE IN A


WAY CONTRARY TO THE DOCTRINES ALREADY LAID DOWN BY THIS COURT.

III

36
RESPONDENT COURT OF TAX APPEALS GRAVELY ERRED IN FINDING PO BEEN SING TO
HAVE INCURRED THE ALLEGED DEFICIENCY TAXES IN QUESTION. 3

We affirm.

Settled is the rule that the factual findings of the Court of Tax Appeals are binding upon this Honorable
Court and can only be disturbed on appeal if not supported by substantial evidence.4

The assignments of errors boils down to a single issue previously raised before the respondent
Court, i.e., whether or not the assessments have valid and legal bases.

The applicable legal provision is Section 16(b) of the National Internal Revenue Code of 1977 as
amended. It reads:

Sec. 16. Power of the Commissioner of Internal Revenue to make assessments.—

xxx xxx xxx

(b) Failure to submit required returns, statements, reports and other documents. - When a
report required by law as a basis for the assessment of an national internal revenue tax
shall not be forthcoming within the time fixed by law or regulation or when there is
reason to believe that any such report is false, incomplete, or erroneous, the
Commissioner of Internal Revenue shall assess the proper tax on the best evidence
obtainable.

In case a person fails to file a required return or other document at the time prescribed by
law, or willfully or otherwise, files a false or fraudulent return or other documents, the
Commissioner shall make or amend the return from his own knowledge and from such
information as he can obtain through testimony or otherwise, which shall be prima
facie correct and sufficient for all legal purposes.

The law is specific and clear. The rule on the "best evidence obtainable" applies when a tax report
required by law for the purpose of assessment is not available or when the tax report is incomplete or
fraudulent.

In the instant case, the persistent failure of the late Po Bien Sing and the herein petitioner to present
their books of accounts for examination for the taxable years involved left the Commissioner of
Internal Revenue no other legal option except to resort to the power conferred upon him under Section
16 of the Tax Code.

The tax figures arrived at by the Commissioner of Internal Revenue are by no means arbitrary. We
reproduce the respondent court's findings, to wit:

As thus shown, on the basis of the quantity of bottles of wines seized during the raid and
the sworn statements of former employees Messrs. Nelson S. Po and Alfonso Po taken on
May 26, and 27,1971, respectively, by the investigating team in Cebu City (Exhs. 4 and
37
5, pp. 514-517, pp. 511-513, Folder 11, BIR rec.), it was ascertained that the Silver Cup
for the years 1964 to 1970, inclusive, utilized and consumed in the manufacture of
compounded liquours and other products 20,105 drums of alcohol as raw materials
81,288,787 proof liters of alcohol. As determined, the total specific tax liability of the
taxpayer for 1964 to 1971 amounted to P5,593,003.68 (Exh. E, petition, p. 10, CTA rec.)

Likewise, the team found due from Silver Cup deficiency income taxes for the years
1966 to 1970 inclusive in the aggregate sum of P7,154,685.16, as follows:

1966 P207,636.24

1967 645,335.04

1968 1,683,588.48

1969 1,589,622.48

1970 3,028,502.92

Total amount due.

and collectible P7,154,685.16

The 50% surcharge has been imposed, pursuant to Section 72 * of the Tax Code and tax 1/2% monthly
interest has likewise been imposed pursuant to the provision of Section 51(d) ** of the Tax Code (Exh.
O, petition). 5

The petitioner assails these assessments as wrong.

In the case of Collector of Internal Revenue vs. Reyes, 6 we ruled:

Where the taxpayer is appealing to the tax court on the ground that the Collector's
assessment is erroneous, it is incumbent upon him to prove there what is the correct and
just liability by a full and fair disclosure of all pertinent data in his possession. Otherwise,
if the taxpayer confines himself to proving that the tax assessment is wrong, the tax court
proceedings would settle nothing, and the way would be left open for subsequent
assessments and appeals in interminable succession.

Tax assessments by tax examiners are presumed correct and made in good faith. The taxpayer has the
duty to prove otherwise. 7 In the absence of proof of any irregularities in the performance of duties, an
assessment duly made by a Bureau of Internal Revenue examiner and approved by his superior officers
will not be disturbed. 8 All presumptions are in favor of the correctness of tax assessments. 9

38
On the whole, we find that the fraudulent acts detailed in the decision under review had not been
satisfactorily rebutted by the petitioner. There are indeed clear indications on the part of the taxpayer to
deprive the Government of the taxes due. The Assistant Factory Superintendent of Silver Cup, Nelson
Po gave the following testimony:

Annexes "A", "A-1 " to "A-17" show that from January to December 1970, Silver Cup
had used in production 189 drums of untaxed distilled alcohol and 3,722 drums of
untaxed distilled alcohol. Can you tell us how could this be possible with the presence of
a revenue inspector in the premises of Silver Cup during working hours?

Actually, the revenue inspector or storekeeper comes around once a week


on the average. Sometimes, when the storekeeper is around in the morning
and Po Bein Sing wants to operate with untaxed alcohol as raw materials,
Po Bien Sing tells the storekeeper to go home because the factory is not
going to operate for the day. After the storekeeper leaves, the illegal
operation then begins. Untaxed alcohol is brought in from Cebu Alcohol
Plant into the compound of Silver Cup sometimes at about 6:00 A.M. or at
12:00 noon or in the evening or even at mid-night when the storekeeper is
not around. When the storekeeper comes, he sees nothing because untaxed
alcohol is brought directly to, and stored at, a secret tunnel within the
bodega itself inside the compound of Silver Cup.

In the same vein, the factory personnel manager testified that false entries were entered in
the official register book: thus,

A — As factory personnel manager and all-around handy man of Po Bien


Sing, owner of Silver Cup, these labels were entrusted to me to make the
false entries in the official register book of Silver Cup, which I did under
the direction of Po Bien Sing. (Sworn statement, p. 512, Folder II, BIR
rec.) 10 (Emphasis ours)

The existence of fraud as found by the respondents can not be lightly set aside absent substantial
evidence presented by the petitioner to counteract such finding. The findings of fact of the respondent
Court of Tax Appeals are entitled to the highest respect.11 We do not find anything in the questioned
decision that should disturb this long-established doctrine.

WHEREFORE, the Petition is DENIED. The Decision of the respondent Court of Tax Appeals is
hereby AFFIRMED. Costs against the petitioner. SO ORDERED

DIGEST: SY PO vs. CTA


G.R. No. 81446; August 18, 1988

Facts: Po Bien Sing, the sole proprietor of Silver Cup Wine Factory (SCWF), engaged in the business
of manufacture and sale of compounded liquors. On the basis of a denunciation against SCWF

39
allegedly "for tax evasion amounting to millions of pesos, Secretary of Finance directed the Finance-
BIR--NBI team to investigate.

On the basis of the team's report of investigation, the respondent Commissioner of Internal Revenue
assessed Mr. Po Bien Sing deficiency income tax for 1966 to 1970 in the amount of P7,154,685.16 and
for deficiency specific tax for January 2,1964 to January 19, 1972 in the amount of P5,595,003.68

Petitioner protested the deficiency assessments. The BIR recommended the reiteration of the
assessments in view of the taxpayer's persistent failure to present the books of accounts for
examination.

Issue: WON the assessments have valid and legal basis.

Held: The law is specific and clear. The rule on “The Best Evidence Obtainable” applies when a tax
report required by law for the purpose of assessment is not available or when tax report is incomplete
or fraudulent.

The tax assessment by tax examiners are presumed correct and made in good faith. The taxpayer has
the duty to prove otherwise. In the absence of proof of irregularities in the performance of duties, an
assessment duly made by the BIR examiner and approved by his superior officers will not be disturbed.
All presumptions are in favour of the correctness of tax assessments.

G.R. No. L-13656 January 31, 1962

COLLECTOR OF INTERNAL REVENUE, (now Commissioner), petitioner,


vs.
ALBERTO D. BENIPAYO, respondent.

DIZON, J.:

This is an appeal taken by the Collector of Internal Revenue from the decision of the Court of Tax
Appeals dated January 23, 1948, reversing the one rendered by the former, thereby relieving
respondent Alberto D. Benipayo from the payment of the deficiency amusement tax assessed against
him in the total amount of P12,093.45.

Respondent is the owner and operator of the Lucena Theater located in the municipality of Lucena,
Quezon. On October 3, 1953 Internal Revenue Agent Romeo de Guia investigated respondent's
amusement tax liability in connection with the operation of said theater during the period from August,
1952 to September, 1953. On October 15, 1953 De Guia submitted his report to the Provincial Revenue
Agent to the effect that respondent had disproportionately issued tax-free 20-centavo children's tickets.
His finding was that during the years 1949 to 1951 the average ratio of adults and children patronizing
the Lucena Theater was 3 to 1, i.e., for every three adults entering the theater, one child was also
admitted, while during the period in question, the proportion is reversed - three children to one adult.

40
From this he concluded that respondent must have fraudulently sold two tax-free 20-centavo tickets, in
order to avoid payment of the amusement tax prescribed in Section 260 of the National Internal
Revenue Code. Based on the average ratio between adult and children attendance in the past years,
Examiner de Guia recommended a deficiency amusement tax assessment against respondent in the sum
of P11,193.45, inclusive of 25% surcharge, plus a suggested compromise penalty of P900.00 for
violation of section 260 of the National Internal Revenue Code, or a total sum of P12,093.45 covering
the period from August, 1952 to September, 1953 inclusive. On July 14, 1954, petitioner issued a
deficiency amusement tax assessment against respondent, demanding from the latter the payment of the
total sum of P12,152.93 within thirty days from receipt thereof. On August 16, 1954, respondent filed
the corresponding protest with the Conference Staff of the Bureau of Internal Revenue. After due
hearing, the Conference Staff submitted to petitioner Collector of Internal Revenue its finding to the
effect that the "meager reports of these fieldmen (Examiner de Guia and the Provincial Revenue Agent
of Quezon) are mere presumptions and conclusions, devoid of findings of the fact of the alleged
fraudulent practices of the herein taxpayer". In view thereof, and as recommended by the Conference
Staff, petitioner referred the case back to the Provincial Revenue Agent of Quezon for further
investigation. The report submitted by Provincial Revenue Officer H.I. Bernardo after this last
investigation partly reads as follows:.

The returns from July 1 to July 11, showed that 31.43% of the entire audience of 12,754
consisted of adults, the remaining 68.57% of children. During this said period due, perhaps, to
the absence of agents in the premises, subject taxpayer was able to manipulate the issuance of
tickets in the way and manner alleged in Asst. De Guia's indorsement report mentioned above.
But during the period from July 14 to July 24, 1955, when agents of this Office supervised in the
sales of admission tickets the sales for adults soared upwards to 76% while that for children
dropped correspondingly to 24%.

It is opined without fear of contradiction that the ratio of three (3) adults to every one (1) child
in the audience or a proportion of 75:25 as reckoned in Asst. De Guia's indorsement report to
this Office's new findings of a proportion of 76:24, represents and conveys the true picture of the
situation under the law of averages, provided that the film being shown is not a children's show.
There is no hard and fast rule in this regard, but this findings would seem to admit no
contradiction.

Please note that the new findings of this Office is not a direct proof of what has transpired
during the period investigated by Asst. De Guia and now pending before the Conference Staff", .
. (Exh. 3, BIR Record, p. 137-138).

After considering said report, the Conference Staff of the Bureau of Internal Revenue recommended to
the Collector of Internal Revenue the issuance of the deficiency amusement tax assessment in question.

The only issue in this appeal is whether or not there is sufficient evidence in the record showing that
respondent, during the period under review, sold and issued to his adult customers two tax-free 20-
centavo children's tickets, instead of one 40-centavo ticket for each adult customer; to cheat or defraud
the Government. On this question the Court of Tax Appeals said the following in the appealed
decision:.
41
To our mind, the appealed decision has no factual basis and must be reversed. An assessment
fixes and determines the tax liability of a taxpayer. As soon as it is served, an obligation arises
on the part of the taxpayer concerned to pay the amount assessed and demanded. Hence,
assessments should not be based on mere presumptions no matter how reasonable or logical said
presumptions may be. Assuming arguendothat the average ratio of adults and children
patronizing the Lucena Theater from 1949 to 1951 was 3 to 1, the same does not give rise to the
inference that the same conditions existed during the years in question (1952 and 1953). The fact
that almost the same ratio existed during the month of July, 1955 does not provide a sufficient
inference on the conditions in 1952 and 1953. . .

In order to stand the test of judicial scrutiny, the assessment must be based on actual facts. The
presumption of correctness of assessment being a mere presumption cannot be made to rest on
another presumption that the circumstances in 1952 and 1953 are presumed to be the same as
those existing in 1949 to 1951 and July 1955. In the case under consideration there are no
substantial facts to support the assessment in question. ...

A review of the records has not disclosed anything sufficient to justify a reversal of the above finding
made by the Court of Tax Appeals. It should be borne in mind that to sustain the deficiency tax
assessed against respondent would amount, in effect, to a finding that he had, for a considerable period
of time, cheated and defrauded the government by selling to each adult patron two children's tax-free
tickets instead of one ticket subject to the amusement tax provided for in Section 260 of the National
Internal Revenue Code. Fraud is a serious charge and, to be sustained, it must be supported by clear
and convincing proof which, in the present case, is lacking.

The claim that respondent admitted having resorted to the anomalous practice already mentioned is not
entirely correct. What respondent appears to have admitted was that during a certain limited period he
had adopted a sort of rebate system applicable to cases where adults and children came in groups and
were al anomalous practice already mentioned is not entirely correct. What respondent appears to have
admitted was that during a certain limited period he had adopted a sort of rebate ystem applicable to
cases where adults and children came in group and were all charged 20 centavo admission tickets. This
practice was, however, discontinued when he was informed by the Bureau of Internal Revenue that it
was not in accordance with law.

WHEREFORE, the appealed judgment is hereby affirmed with costs.

NO DIGEST

G.R. No. L-36181 October 23, 1982

MERALCO SECURITIES CORPORATION (now FIRST PHILIPPINE HOLDINGS


CORPORATION), petitioner,
vs.

42
HON. VICTORINO SAVELLANO and ASUNCION BARON VDA. DE MANIAGO, et al., as
heirs of the late Juan G. Maniago, respondents.

G.R. No. L-36748 October 23, 1982

COMMISSIONER OF INTERNAL REVENUE, petitioner,


vs.
HON. VICTORINO SAVELLANO and ASUNCION BARON VDA. DE MANIAGO, et al., as
heirs of the late Juan G. Maniago, respondents.

G.R. No. L-36181

TEEHANKEE, J.:

These are original actions for certiorari to set aside and annul the writ of mandamus issued by Judge
Victorino A. Savellano of the Court of First Instance of Manila in Civil Case No. 80830 ordering
petitioner Meralco Securities Corporation (now First Philippine Holdings Corporation) to pay, and
petitioner Commissioner of Internal Revenue to collect from the former, the amount of
P51,840,612.00, by way of alleged deficiency corporate income tax, plus interests and surcharges due
thereon and to pay private respondents 25% of the total amount collectible as informer's reward.

On May 22, 1967, the late Juan G. Maniago (substituted in these proceedings by his wife and children)
submitted to petitioner Commissioner of Internal Revenue confidential denunciation against the
Meralco Securities Corporation for tax evasion for having paid income tax only on 25 % of the
dividends it received from the Manila Electric Co. for the years 1962-1966, thereby allegedly
shortchanging the government of income tax due from 75% of the said dividends.

Petitioner Commissioner of Internal Revenue caused the investigation of the denunciation after which
he found and held that no deficiency corporate income tax was due from the Meralco Securities
Corporation on the dividends it received from the Manila Electric Co., since under the law then
prevailing (section 24[a] of the National Internal Revenue Code) "in the case of dividends received by
a domestic or foreign resident corporation liable to (corporate income) tax under this Chapter . . . .only
twenty-five per centum thereof shall be returnable for the purposes of the tax imposed under this
section." The Commissioner accordingly rejected Maniago's contention that the Meralco from whom
the dividends were received is "not a domestic corporation liable to tax under this Chapter." In a letter
dated April 5, 1968, the Commissioner informed Maniago of his findings and ruling and therefore
denied Maniago's claim for informer's reward on a non-existent deficiency. This action of the
Commissioner was sustained by the Secretary of Finance in a 4th Indorsement dated May 11, 1971.

On August 28, 1970, Maniago filed a petition for mandamus, and subsequently an amended petition for
mandamus, in the Court of First Instance of Manila, docketed therein as Civil Case No. 80830, against
the Commissioner of Internal Revenue and the Meralco Securities Corporation to compel the
Commissioner to impose the alleged deficiency tax assessment on the Meralco Securities Corporation
and to award to him the corresponding informer's reward under the provisions of R.A. 2338.

43
On October 28, 1978, the Commissioner filed a motion to dismiss, arguing that since in matters of
issuance and non-issuance of assessments, he is clothed under the National Internal Revenue Code and
existing rules and regulations with discretionary power in evaluating the facts of a case and since
mandamus win not lie to compel the performance of a discretionary power, he cannot be compelled to
impose the alleged tax deficiency assessment against the Meralco Securities Corporation. He further
argued that mandamus may not lie against him for that would be tantamount to a usurpation of
executive powers, since the Office of the Commissioner of Internal Revenue is undeniably under the
control of the executive department.

On the other hand, the Meralco Securities Corporation filed its answer, dated January 15, 1971,
interposing as special and/or affirmative defenses that the petition states no cause of action, that the
action is premature, that mandamus win not lie to compel the Commissioner of Internal Revenue to
make an assessment and/or effect the collection of taxes upon a taxpayer, that since no taxes have
actually been recovered and/or collected, Maniago has no right to recover the reward prayed for, that
the action of petitioner had already prescribed and that respondent court has no jurisdiction over the
subject matter as set forth in the petition, the same being cognizable only by the Court of Tax Appeals.

On January 10, 1973, the respondent judge rendered a decision granting the writ prayed for and
ordering the Commissioner of Internal Revenue to assess and collect from the Meralco Securities
Corporation the sum of P51,840,612.00 as deficiency corporate income tax for the period 1962 to 1969
plus interests and surcharges due thereon and to pay 25% thereof to Maniago as informer's reward.

All parties filed motions for reconsideration of the decision but the same were denied by respondent
judge in his order dated April 6, 1973, with respondent judge denying respondents' claim for attorneys
fees and for execution of the decision pending appeal.

Hence, the Commissioner filed a separate petition with this Court, docketed as G.R. No. L-36748
praying that the decision of respondent judge dated January 10, 1973 and his order dated April 6, 1973
be reconsidered for respondent judge has no jurisdiction over the subject matter of the case and that the
issuance or non-issuance of a deficiency assessment is a prerogative of the Commissioner of Internal
Revenue not reviewable by mandamus.

The Meralco Securities Corporation (now First Philippine Holdings Corporation) likewise appealed the
same decision of respondent judge in G.R. No. L-36181 and in the Court's resolution dated June 13,
1973, the two cases were ordered consolidated.

We grant the petitions.

Respondent judge has no jurisdiction to take cognizance of the case because the subject matter thereof
clearly falls within the scope of cases now exclusively within the jurisdiction of the Court of Tax
Appeals. Section 7 of Republic Act No. 1125, enacted June 16, 1954, granted to the Court of Tax
Appeals exclusive appellate jurisdiction to review by appeal, among others, decisions of the
Commissioner of Internal Revenue in cases involving disputed assessments, refunds of internal revenue
taxes, fees or other charges, penalties imposed in relation thereto, or other matters arising under the
National Internal Revenue Code or other law or part of law administered by the Bureau of Internal

44
Revenue. The law transferred to the Court of Tax Appeals jurisdiction over all cases involving said
assessments previously cognizable by courts of first instance, and even those already pending in said
courts. 1 The question of whether or not to impose a deficiency tax assessment on Meralco Securities
Corporation undoubtedly comes within the purview of the words "disputed assessments" or of "other
matters arising under the National Internal Revenue Code . . . .In the case of Blaquera vs. Rodriguez, et
al, 2 this Court ruled that "the determination of the correctness or incorrectness of a tax assessment to
which the taxpayer is not agreeable, falls within the jurisdiction of the Court of Tax Appeals and not of
the Court of First Instance, for under the provisions of Section 7 of Republic Act No. 1125, the Court
of Tax Appeals has exclusive appellate jurisdiction to review, on appeal, any decision of the Collector
of Internal Revenue in cases involving disputed assessments and other matters arising under the
National Internal Revenue Code or other law or part of law administered by the Bureau of Internal
Revenue."

Thus, even assuming arguendo that the right granted the taxpayers affected to question and appeal
disputed assessments, under section 7 of Republic Act No. 1125, may be availed of by strangers or
informers like the late Maniago, the most that he could have done was to appeal to the Court of Tax
Appeals the ruling of petitioner Commissioner of Internal Revenue within thirty (30) days from receipt
thereof pursuant to section 11 of Republic Act No. 1125. 3 He failed to take such an appeal to the tax
court. The ruling is clearly final and no longer subject to review by the courts. 4

It is furthermore a well-recognized rule that mandamus only lies to enforce the performance of a
ministerial act or duty 5 and not to control the performance of a discretionary power. 6 Purely
administrative and discretionary functions may not be interfered with by the courts. 7 Discretion, as thus
intended, means the power or right conferred upon the office by law of acting officially under certain
circumstances according to the dictates of his own judgment and conscience and not controlled by the
judgment or conscience of others. 8 mandamus may not be resorted to so as to interfere with the
manner in which the discretion shall be exercised or to influence or coerce a particular determination. 9

In an analogous case, where a petitioner sought to compel the Rehabilitation Finance Corporation to
accept payment of the balance of his indebtedness with his backpay certificates, the Court ruled that
"mandamus does not compel the Rehabilitation Finance Corporation to accept backpay certificates in
payment of outstanding loans. Although there is no provision expressly authorizing such acceptance,
nor is there one prohibiting it, yet the duty imposed by the Backpay Law upon said corporation as to
the acceptance or discount of backpay certificates is neither clear nor ministerial, but discretionary
merely, and such special civil action does not issue to control the exercise of discretion of a public
officer." 10 Likewise, we have held that courts have no power to order the Commissioner of Customs to
confiscate goods imported in violation of the Import Control Law, R.A. 426, as said forfeiture is
subject to the discretion of the said official, 11 nor may courts control the determination of whether or
not an applicant for a visa has a non-immigrant status or whether his entry into this country would be
contrary to public safety for it is not a simple ministerial function but an exercise of discretion. 12

Moreover, since the office of the Commissioner of Internal Revenue is charged with the administration
of revenue laws, which is the primary responsibility of the executive branch of the government,
mandamus may not he against the Commissioner to compel him to impose a tax assessment not found
by him to be due or proper for that would be tantamount to a usurpation of executive functions. As we
45
held in the case of Commissioner of Immigration vs. Arca 13 anent this principle, "the administration of
immigration laws is the primary responsibility of the executive branch of the government. Extensions
of stay of aliens are discretionary on the part of immigration authorities, and neither a petition for
mandamus nor one for certiorari can compel the Commissioner of Immigration to extend the stay of an
alien whose period to stay has expired.

Such discretionary power vested in the proper executive official, in the absence of arbitrariness or
grave abuse so as to go beyond the statutory authority, is not subject to the contrary judgment or
control of others. " "Discretion," when applied to public functionaries, means a power or right
conferred upon them by law of acting officially, under certain circumstances, uncontrolled by the
judgment or consciences of others. A purely ministerial act or duty in contradiction to a discretional act
is one which an officer or tribunal performs in a given state of facts, in a prescribed manner, in
obedience to the mandate of a legal authority, without regard to or the exercise of his own judgment
upon the propriety or impropriety of the act done. If the law imposes a duty upon a public officer and
gives him the right to decide how or when the duty shall be performed, such duty is discretionary and
not ministerial. The duty is ministerial only when the discharge of the same requires neither the
exercise of official discretion or judgment." 14

Thus, after the Commissioner who is specifically charged by law with the task of enforcing and
implementing the tax laws and the collection of taxes had after a mature and thorough study rendered
his decision or ruling that no tax is due or collectible, and his decision is sustained by the Secretary,
now Minister of Finance (whose act is that of the President unless reprobated), such decision or ruling
is a valid exercise of discretion in the performance of official duty and cannot be controlled much less
reversed by mandamus. A contrary view, whereby any stranger or informer would be allowed to usurp
and control the official functions of the Commissioner of Internal Revenue would create disorder and
confusion, if not chaos and total disruption of the operations of the government.

Considering then that respondent judge may not order by mandamus the Commissioner to issue the
assessment against Meralco Securities Corporation when no such assessment has been found to be due,
no deficiency taxes may therefore be assessed and collected against the said corporation. Since no taxes
are to be collected, no informer's reward is due to private respondents as the informer's heirs. Informer's
reward is contingent upon the payment and collection of unpaid or deficiency taxes. An informer is
entitled by way of reward only to a percentage of the taxes actually assessed and collected. Since no
assessment, much less any collection, has been made in the instant case, respondent judge's writ for the
Commissioner to pay respondents 25% informer's reward is gross error and without factual nor legal
basis.

WHEREFORE, the petitions are hereby granted and the questioned decision of respondent judge dated
January 10, 1973 and order dated April 6, 1973 are hereby reversed and set aside. With costs against
private respondents.

DIGEST:

Meralco Securities Corporation vs. Savellano


GR No. L-36181 October 23, 1982
46
Facts: On May 22, 1967, the late Juan G. Maniago (substituted in these proceedings by his wife and
children) submitted to petitioner Commissioner of Internal Revenue confidential denunciation against
the Meralco Securities Corporation for tax evasion for having paid income tax only on 25 % of the
dividends it received from the Manila Electric Co. for the years 1962-1966, thereby allegedly
shortchanging the government of income tax due from 75% of the said dividends.

Petitioner Commissioner of Internal Revenue caused the investigation of the denunciation after which
he found and held that no deficiency corporate income tax was due from the Meralco Securities
Corporation on the dividends it received from the Manila Electric Co. and accordingly denied
Maniago's claim for informer's reward on a non-existent deficiency.

On August 28, 1970, Maniago filed a petition for mandamus, and subsequently an amended petition for
mandamus, in the Court of First Instance of Manila, docketed therein as Civil Case No. 80830, against
the Commissioner of Internal Revenue and the Meralco Securities Corporation to compel the
Commissioner to impose the alleged deficiency tax assessment on the Meralco Securities Corporation
and to award to him the corresponding informer's reward under the provisions of R.A. 2338.
Respondent judge granted the said petition and thereafter, denied the motions for reconsideration filed
by all the parties.

Issues: (1) Whether or not respondent judge has jurisdiction over the subject matter of the case; (2)
Whether or not respondent heirs of Maniago are entitled to informer’s reward.

Held: (1) Respondent judge has no jurisdiction to take cognizance of the case because the subject
matter thereof clearly falls within the scope of cases now exclusively within the jurisdiction of the
Court of Tax Appeals. Section 7 of Republic Act No. 1125, enacted June 16, 1954, granted to the Court
of Tax Appeals exclusive appellate jurisdiction to review by appeal, among others, decisions of the
Commissioner of Internal Revenue in cases involving disputed assessments, refunds of internal revenue
taxes, fees or other charges, penalties imposed in relation thereto, or other matters arising under the
National Internal Revenue Code or other law or part of law administered by the Bureau of Internal
Revenue. The law transferred to the Court of Tax Appeals jurisdiction over all cases involving said
assessments previously cognizable by courts of first instance, and even those already pending in said
courts. The question of whether or not to impose a deficiency tax assessment on Meralco Securities
Corporation undoubtedly comes within the purview of the words "disputed assessments" or of "other
matters arising under the National Internal Revenue Code . . . .In the case of Blaquera vs. Rodriguez, et
al, this Court ruled that "the determination of the correctness or incorrectness of a tax assessment to
47
which the taxpayer is not agreeable, falls within the jurisdiction of the Court of Tax Appeals and not of
the Court of First Instance, for under the provisions of Section 7 of Republic Act No. 1125, the Court
of Tax Appeals has exclusive appellate jurisdiction to review, on appeal, any decision of the Collector
of Internal Revenue in cases involving disputed assessments and other matters arising under the
National Internal Revenue Code or other law or part of law administered by the Bureau of Internal
Revenue."

(2) Considering then that respondent judge may not order by mandamus the Commissioner to issue the
assessment against Meralco Securities Corporation when no such assessment has been found to be due,
no deficiency taxes may therefore be assessed and collected against the said corporation. Since no taxes
are to be collected, no informer's reward is due to private respondents as the informer's heirs. Informer's
reward is contingent upon the payment and collection of unpaid or deficiency taxes. An informer is
entitled by way of reward only to a percentage of the taxes actually assessed and collected. Since no
assessment, much less any collection, has been made in the instant case, respondent judge's writ for the
Commissioner to pay respondents 25% informer's reward is gross error and without factual nor legal
basis.

Petitions granted and the questioned decision of respondent judge and order reversed and set aside.

G.R. No. 130430 December 13, 1999

REPUBLIC OF THE PHILIPPINES, represented by the Commissioner of the Bureau of


Internal Revenue (BIR), petitioner,
vs.
SALUD V. HIZON, respondent.

MENDOZA, J.:

This is a petition for review of the decision 1 of the Regional Trial Court, Branch 44, San Fernando,
Pampanga, dismissing the suit filed by the Bureau of Internal Revenue for collection of tax.

The facts are as follows:

On July 18, 1986, the BIR issued to respondent Salud V. Hizon a deficiency income tax assessment of
P1,113,359.68 covering the fiscal year 1981-1982. Respondent not having contested the assessment,

48
petitioner, on January 12, 1989, served warrants of distraint and levy to collect the tax deficiency.
However, for reasons not known, it did not proceed to dispose of the attached properties.

More than three years later, or on November 3, 1992, respondent wrote the BIR requesting a
reconsideration of her tax deficiency assessment. The BIR, in a letter dated August 11, 1994, denied
the request. On January 1, 1997, it filed a case with the Regional Trial Court, Branch 44, San Fernando,
Pampanga to collect the tax deficiency. The complaint was signed by Norberto Salud, Chief of the
Legal Division, BIR Region 4, and verified by Amancio Saga, the Bureau's Regional Director in
Pampanga.

Respondent moved to dismiss the case on two grounds: (1) that the complaint was not filed upon
authority of the BIR Commissioner as required by §221 2 of the National Internal Revenue Code, and
(2) that the action had already prescribed. Over petitioner's objection, the trial court, on August 28,
1997, granted the motion and dismissed the complaint. Hence, this petition. Petitioner raises the
following issues: 3

I. WHETHER OR NOT THE INSTITUTION OF THE CIVIL CASE FOR


COLLECTION OF TAXES WAS WITHOUT THE APPROVAL OF THE
COMMISSIONER IN VIOLATION OF SECTION 221 OF THE NATIONAL
INTERNAL REVENUE CODE.

II. WHETHER OR NOT THE ACTION FOR COLLECTION OF TAXES FILED


AGAINST RESPONDENT HAD ALREADY BEEN BARRED BY THE STATUTE OF
LIMITATIONS.

First. In sustaining respondent's contention that petitioner's complaint was filed without the authority of
the BIR Commissioner, the trial court stated: 4

There is no question that the National Internal Revenue Code explicitly provides that in
the matter of filing cases in Court, civil or criminal, for the collection of taxes, etc., the
approval of the commissioner must first be secured. . . . [A]n action will not prosper in
the absence of the commissioner's approval. Thus, in the instant case, the absence of the
approval of the commissioner in the institution of the action is fatal to the cause of the
plaintiff . . . .

The trial court arrived at this conclusion because the complaint filed by the BIR was not signed
by then Commissioner Liwayway Chato.

Sec. 221 of the NIRC provides:

Form and mode of proceeding in actions arising under this Code. — Civil and criminal
actions and proceedings instituted in behalf of the Government under the authority of this
Code or other law enforced by the Bureau of Internal Revenue shall be brought in the
name of the Government of the Philippines and shall be conducted by the provincial or
city fiscal, or the Solicitor General, or by the legal officers of the Bureau of Internal
Revenue deputized by the Secretary of Justice, but no civil and criminal actions for the
49
recovery of taxes or the enforcement of any fine, penalty or forfeiture under this Code
shall begun without the approval of the Commissioner. (Emphasis supplied)

To implement this provision Revenue Administrative Order No. 5-83 of the BIR provides in
pertinent portions:

The following civil and criminal cases are to be handled by Special Attorneys and Special
Counsels assigned in the Legal Branches of Revenues Regions:

xxx xxx xxx

II. Civil Cases

1. Complaints for collection on cases falling within the jurisdiction of the


Region . . . .

In all the abovementioned cases, the Regional Director is authorized to sign


all pleadings filed in connection therewith which, otherwise, requires the
signature of the Commissioner.

xxx xxx xxx

Revenue Administrative Order No. 10-95 specifically authorizes the Litigation and Prosecution Section
of the Legal Division of regional district offices to institute the necessary civil and criminal actions for
tax collection. As the complaint filed in this case was signed by the BIR's Chief of Legal Division for
Region 4 and verified by the Regional Director, there was, therefore, compliance with the law.

However, the lower court refused to recognize RAO No. 10-95 and, by implication, RAO No. 5-83. It
held:

[M]emorand[a], circulars and orders emanating from bureaus and agencies whether in the
purely public or quasi-public corporations are mere guidelines for the internal functioning
of the said offices. They are not laws which courts can take judicial notice of. As such,
they have no binding effect upon the courts for such memorand[a] and circulars are not
the official acts of the legislative, executive and judicial departments of the Philippines. .
..5

This is erroneous. The rule is that as long as administrative issuances relate solely to carrying into
effect the provisions of the law, they are valid and have the force of law. 6 The governing statutory
provision in this case is §4(d) of the NIRC which provides:

Specific provisions to be contained in regulations. — The regulations of the Bureau of


Internal Revenue shall, among other things, contain provisions specifying, prescribing, or
defining:

xxx xxx xxx


50
(d) The conditions to be observed by revenue officers, provincial fiscals and other
officials respecting the institution and conduct of legal actions and proceedings.

RAO Nos. 5-83 and 10-95 are in harmony with this statutory mandate.

As amended by R.A. No. 8424, the NIRC is now even more categorical. Sec. 7 of the present Code
authorizes the BIR Commissioner to delegate the powers vested in him under the pertinent provisions
of the Code to any subordinate official with the rank equivalent to a division chief or higher, except the
following:

(a) The power to recommend the promulgation of rules and regulations by the Secretary
of Finance;

(b) The power to issue rulings of first impression or to reverse, revoke or modify any
existing ruling of the Bureau;

(c) The power to compromise or abate under §204 (A) and (B) of this Code, any tax
deficiency: Provided, however, that assessment issued by the Regional Offices involving
basic deficiency taxes of five hundred thousand pesos (P500,000.00) or less, and minor
criminal violations as may be determined by rules and regulations to be promulgated by
the Secretary of Finance, upon the recommendation of the Commissioner, discovered by
regional and district officials, may be compromised by a regional evaluation board which
shall be composed of the Regional Director as Chairman, the Assistant Regional Director,
heads of the Legal, Assessment and Collection Divisions and the Revenue District
Officer having jurisdiction over the taxpayer, as members; and

(d) The power to assign or reassign internal revenue officers to establishments where
articles subject to excise tax are produced or kept.

None of the exceptions relates to the Commissioner's power to approve the filing of tax
collection cases.

Second. With regard to the issue that the case filed by petitioner for the collection of respondent's tax
deficiency is barred by prescription, §223(c) of the NIRC provides:

Any internal revenue tax which has been assessed within the period of limitation above-
prescribed may be collected by distraint or levy or by a proceeding in court within three
years 7 following the assessment of the tax.

The running of the three-year prescriptive period is suspended 8 —

for the period during which the Commissioner is prohibited from making the assessment
or beginning distraint or levy or a proceeding in court and for sixty days thereafter; when
the taxpayer requests for a reinvestigation which is granted by the Commissioner; when
the taxpayer cannot be located in the address given by him in the return filed upon which
the tax is being assessed or collected; provided, that, if the taxpayer informs the
51
Commissioner of any change in address, the running of the statute of limitations will not
be suspended; when the warrant of distraint or levy is duly served upon the taxpayer, his
authorized representative or a member of his household with sufficient discretion, and no
property could be located; and when the taxpayer is out of the Philippines.

Petitioner argues that, in accordance with this provision, respondent's request for reinvestigation
of her tax deficiency assessment on November 3, 1992 effectively suspended the running of the
period of prescription such that the government could still file a case for tax collection. 9

The contention has no merit. Sec. 229 10 of the Code mandates that a request for reconsideration must
be made within 30 days from the taxpayer's receipt of the tax deficiency assessment, otherwise the
assessment becomes final, unappealable and, therefore, demandable. 11 The notice of assessment for
respondent's tax deficiency was issued by petitioner on July 18, 1986. On the other hand, respondent
made her request for reconsideration thereof only on November 3, 1992, without stating when she
received the notice of tax assessment. She explained that she was constrained to ask for a
reconsideration in order to avoid the harassment of BIR collectors. 12 In all likelihood, she must have
been referring to the distraint and levy of her properties by petitioner's agents which took place on
January 12, 1989. Even assuming that she first learned of the deficiency assessment on this date, her
request for reconsideration was nonetheless filed late since she made it more than 30 days thereafter.
Hence, her request for reconsideration did not suspend the running of the prescriptive period provided
under §223(c). Although the Commissioner acted on her request by eventually denying it on August
11, 1994, this is of no moment and does not detract from the fact that the assessment had long become
demandable.

Nonetheless, it is contended that the running of the prescriptive period under §223(c) was suspended
when the BIR timely served the warrants of distraint and levy on respondent on January 12,
1989. 13 Petitioner cites for this purpose our ruling in Advertising Associates Inc., v. Court of
Appeals. 14 Because of the suspension, it is argued that the BIR could still avail of the other remedy
under §223(c) of filing a case in court for collection of the tax deficiency, as the BIR in fact did on
January 1, 1997.

Petitioner's reliance on the Court's ruling in Advertising Associates Inc. v. Court of Appeals is
misplaced. What the Court stated in that case and, indeed, in the earlier case of Palanca
v. Commissioner of Internal Revenue, 15 is that the timely service of a warrant of distraint or levy
suspends the running of the period to collect the tax deficiency in the sense that the disposition of the
attached properties might well take time to accomplish, extending even after the lapse of the statutory
period for collection. In those cases, the BIR did not file any collection case but merely relied on the
summary remedy of distraint and levy to collect the tax deficiency. The importance of this fact was not
lost on the Court. Thus, in Advertising Associates, it was held: 16 "It should be noted that the
Commissioner did not institute any judicial proceeding to collect the tax. He relied on the warrants of
distraint and levy to interrupt the running of the statute of limitations.

Moreover, if, as petitioner in effect says, the prescriptive period was suspended twice, i.e., when the
warrants of distraint and levy were served on respondent on January 12, 1989 and then when
respondent made her request for reinvestigation of the tax deficiency assessment on November 3, 1992,
52
the three-year prescriptive period must have commenced running again sometime after the service of
the warrants of distraint and levy. Petitioner, however, does not state when or why this took place and,
indeed, there appears to be no reason for such. It is noteworthy that petitioner raised this point before
the lower court apparently as an alternative theory, which, however, is untenable.

For the foregoing reasons, we hold that petitioner's contention that the action in this case had not
prescribed when filed has no merit. Our holding, however, is without prejudice to the disposition of the
properties covered by the warrants of distraint and levy which petitioner served on respondent, as such
would be a mere continuation of the summary remedy it had timely begun. Although considerable time
has passed since then, as held in Advertising Associates Inc. v. Court of Appeals 17 and Palanca
v. Commissioner of Internal Revenue, 18 the enforcement of tax collection through summary
proceedings may be carried out beyond the statutory period considering that such remedy was
seasonably availed of.

WHEREFORE, the petition is DENIED.

DIGEST:

REPUBLIC vs. HIZON


320 SCRA 574
GR No. 130430, December 13, 1999

"A request for reconsideration of the tax assessment does not effectively suspend the running of the
precriptive period if the same is filed after the assessment had become final and unappealable."

FACTS: On July 18, 1986, the BIR issued to respondent Salud V. Hizon a deficiency income tax
assessment covering the fiscal year 1981-1982. Respondent not having contested the assessment,
petitioner BIR, on January 12, 1989, served warrants of distraint and levy to collect the tax deficiency.
However, for reasons not known, it did not proceed to dispose of the attached properties.
More than three years later, the respondent wrote the BIR requesting a reconsideration of her tax
deficiency assessment. The BIR, in a letter dated August 11, 1994, denied the request. On January 1,
1997, it filed a case with the RTC to collect the tax deficiency. Hizon moved to dismiss the case on two
grounds: (1) that the complaint was not filed upon authority of the BIR Commissioner as required by
Sec. 221 of the NIRC, and (2) that the action had already prescribed. Over petitioner's objection, the
trial court granted the motion and dismissed the complaint.
BIR on the other hand contends that respondent's request for reinvestigation of her tax deficiency
assessment on November 1992 effectively suspended the running of the period of prescription.

ISSUE: Has the action for collection of the tax prescribed?

HELD: Yes. Sec. 229 of the NIRC mandates that a request for reconsideration must be made within 30
days from the taxpayer's receipt of the tax deficiency assessment, otherwise the assessment becomes
final, unappealable and, therefore, demandable. The notice of assessment for respondent's tax
deficiency was issued by petitioner on July 18, 1986. On the other hand, respondent made her request
53
for reconsideration thereof only on November 3, 1992, without stating when she received the notice of
tax assessment. Hence, her request for reconsideration did not suspend the running of the prescriptive
period provided under Sec. 223(c). Although the Commissioner acted on her request by eventually
denying it on August 11, 1994, this is of no moment and does not detract from the fact that the
assessment had long become demandable.

G.R. No. 177279 October 13, 2010

COMMISSIONER OF INTERNAL REVENUE, Petitioner,


vs.
HON. RAUL M. GONZALEZ, Secretary of Justice, L. M. CAMUS ENGINEERING
CORPORATION (represented by LUIS M. CAMUS and LINO D. MENDOZA), Respondents.

DECISION

VILLARAMA, JR., J.:

This is a petition for review on certiorari under Rule 45 of the 1997 Rules of Civil Procedure, as
amended, assailing the Decision1 dated October 31, 2006 and Resolution2 dated March 6, 2007 of the
Court of Appeals (CA) in CA-G.R. SP No. 93387 which affirmed the Resolution3 dated December 13,
2005 of respondent Secretary of Justice in I.S. No. 2003-774 for violation of Sections 254 and 255 of
the National Internal Revenue Code of 1997 (NIRC).

The facts as culled from the records:

Pursuant to Letter of Authority (LA) No. 00009361 dated August 25, 2000 issued by then
Commissioner of Internal Revenue (petitioner) Dakila B. Fonacier, Revenue Officers Remedios C.
Advincula, Jr., Simplicio V. Cabantac, Jr., Ricardo L. Suba, Jr. and Aurelio Agustin T. Zamora
supervised by Section Chief Sixto C. Dy, Jr. of the Tax Fraud Division (TFD), National Office,
conducted a fraud investigation for all internal revenue taxes to ascertain/determine the tax liabilities of
respondent L. M. Camus Engineering Corporation (LMCEC) for the taxable years 1997, 1998 and
1999.4 The audit and investigation against LMCEC was precipitated by the information provided by an
"informer" that LMCEC had substantial underdeclared income for the said period. For failure to
comply with the subpoena duces tecum issued in connection with the tax fraud investigation, a criminal
complaint was instituted by the Bureau of Internal Revenue (BIR) against LMCEC on January 19,
2001 for violation of Section 266 of the NIRC (I.S. No. 00-956 of the Office of the City Prosecutor of
Quezon City).5

Based on data obtained from an "informer" and various clients of LMCEC,6 it was discovered that
LMCEC filed fraudulent tax returns with substantial underdeclarations of taxable income for the years
1997, 1998 and 1999. Petitioner thus assessed the company of total deficiency taxes amounting to
₱430,958,005.90 (income tax - ₱318,606,380.19 and value-added tax [VAT] - ₱112,351,625.71)
covering the said period. The Preliminary Assessment Notice (PAN) was received by LMCEC on
February 22, 2001.7
54
LMCEC’s alleged underdeclared income was summarized by petitioner as follows:

Year Income Income Undeclared Percentage of


Per ITR Per Income Underdeclaration
Investigation
1997 96,638,540.00 283,412,140.84 186,733,600.84 193.30%
1998 86,793,913.00 236,863,236.81 150,069,323.81 172.90%
1999 88,287,792.00 251,507,903.13 163,220,111.13 184.90%8

In view of the above findings, assessment notices together with a formal letter of demand dated August
7, 2002 were sent to LMCEC through personal service on October 1, 2002.9 Since the company and its
representatives refused to receive the said notices and demand letter, the revenue officers resorted to
constructive service10 in accordance with Section 3, Revenue Regulations (RR) No. 12-9911.

On May 21, 2003, petitioner, through then Commissioner Guillermo L. Parayno, Jr., referred to the
Secretary of Justice for preliminary investigation its complaint against LMCEC, Luis M. Camus and
Lino D. Mendoza, the latter two were sued in their capacities as President and Comptroller,
respectively. The case was docketed as I.S. No. 2003-774. In the Joint Affidavit executed by the
revenue officers who conducted the tax fraud investigation, it was alleged that despite the receipt of the
final assessment notice and formal demand letter on October 1, 2002, LMCEC failed and refused to
pay the deficiency tax assessment in the total amount of ₱630,164,631.61, inclusive of increments,
which had become final and executory as a result of the said taxpayer’s failure to file a protest thereon
within the thirty (30)-day reglementary period.12

Camus and Mendoza filed a Joint Counter-Affidavit contending that LMCEC cannot be held liable
whatsoever for the alleged tax deficiency which had become due and demandable. Considering that the
complaint and its annexes all showed that the suit is a simple civil action for collection and not a tax
evasion case, the Department of Justice (DOJ) is not the proper forum for BIR’s complaint. They also
assail as invalid the assessment notices which bear no serial numbers and should be shown to have
been validly served by an Affidavit of Constructive Service executed and sworn to by the revenue
officers who served the same. As stated in LMCEC’s letter-protest dated December 12, 2002 addressed
to Revenue District Officer (RDO) Clavelina S. Nacar of RD No. 40, Cubao, Quezon City, the
company had already undergone a series of routine examinations for the years 1997, 1998 and 1999;
under the NIRC, only one examination of the books of accounts is allowed per taxable year.13

LMCEC further averred that it had availed of the Bureau’s Tax Amnesty Programs (Economic
Recovery Assistance Payment [ERAP] Program and the Voluntary Assessment Program [VAP]) for
1998 and 1999; for 1997, its tax liability was terminated and closed under Letter of Termination14 dated
June 1, 1999 issued by petitioner and signed by the Chief of the Assessment Division.15 LMCEC
claimed it made payments of income tax, VAT and expanded withholding tax (EWT), as follows:

YEAR AMOUNT OF
TAXES
55
PAID
1997 Termination Letter EWT - P 6,000.00
Under Letter of VAT - 540,605.02
Authority No. 174600 IT - 3,000.00
Dated November 4,
1998
1998 ERAP Program WC - 38,404.55
pursuant VAT - 61,635.40
to RR #2-99
1999 VAP Program pursuant IT - 878,495.28
to RR #8-2001 VAT - 1,324,317.0016

LMCEC argued that petitioner is now estopped from further taking any action against it and its
corporate officers concerning the taxable years 1997 to 1999. With the grant of immunity from audit
from the company’s availment of ERAP and VAP, which have a feature of a tax amnesty, the element
of fraud is negated the moment the Bureau accepts the offer of compromise or payment of taxes by the
taxpayer. The act of the revenue officers in finding justification under Section 6(B) of the NIRC (Best
Evidence Obtainable) is misplaced and unavailing because they were not able to open the books of the
company for the second time, after the routine examination, issuance of termination letter and the
availment of ERAP and VAP. LMCEC thus maintained that unless there is a prior determination of
fraud supported by documents not yet incorporated in the docket of the case, petitioner cannot just
issue LAs without first terminating those previously issued. It emphasized the fact that the BIR officers
who filed and signed the Affidavit-Complaint in this case were the same ones who appeared as
complainants in an earlier case filed against Camus for his alleged "failure to obey summons in
violation of Section 5 punishable under Section 266 of the NIRC of 1997" (I.S. No. 00-956 of the
Office of the City Prosecutor of Quezon City). After preliminary investigation, said case was dismissed
for lack of probable cause in a Resolution issued by the Investigating Prosecutor on May 2, 2001.17

LMCEC further asserted that it filed on April 20, 2001 a protest on the PAN issued by petitioner for
having no basis in fact and law. However, until now the said protest remains unresolved. As to the
alleged informant who purportedly supplied the "confidential information," LMCEC believes that such
person is fictitious and his true identity and personality could not be produced. Hence, this case is
another form of harassment against the company as what had been found by the Office of the City
Prosecutor of Quezon City in I.S. No. 00-956. Said case and the present case both have something to
do with the audit/examination of LMCEC for taxable years 1997, 1998 and 1999 pursuant to LA No.
00009361.18

In the Joint Reply-Affidavit executed by the Bureau’s revenue officers, petitioner disagreed with the
contention of LMCEC that the complaint filed is not criminal in nature, pointing out that LMCEC and
its officers Camus and Mendoza were being charged for the criminal offenses defined and penalized
under Sections 254 (Attempt to Evade or Defeat Tax) and 255 (Willful Failure to Pay Tax) of the
NIRC. This finds support in Section 205 of the same Code which provides for administrative (distraint,
levy, fine, forfeiture, lien, etc.) and judicial (criminal or civil action) remedies in order to enforce
56
collection of taxes. Both remedies may be pursued either independently or simultaneously. In this case,
the BIR decided to simultaneously pursue both remedies and thus aside from this criminal action, the
Bureau also initiated administrative proceedings against LMCEC.19

On the lack of control number in the assessment notice, petitioner explained that such is a mere office
requirement in the Assessment Service for the purpose of internal control and monitoring; hence, the
unnumbered assessment notices should not be interpreted as irregular or anomalous. Petitioner stressed
that LMCEC already lost its right to file a protest letter after the lapse of the thirty (30)-day
reglementary period. LMCEC’s protest-letter dated December 12, 2002 to RDO Clavelina S. Nacar,
RD No. 40, Cubao, Quezon City was actually filed only on December 16, 2002, which was disregarded
by the petitioner for being filed out of time. Even assuming for the sake of argument that the
assessment notices were invalid, petitioner contended that such could not affect the present criminal
action,20 citing the ruling in the landmark case of Ungab v. Cusi, Jr.21

As to the Letter of Termination signed by Ruth Vivian G. Gandia of the Assessment Division, Revenue
Region No. 7, Quezon City, petitioner pointed out that LMCEC failed to mention that the undated
Certification issued by RDO Pablo C. Cabreros, Jr. of RD No. 40, Cubao, Quezon City stated that the
report of the 1997 Internal Revenue taxes of LMCEC had already been submitted for review and
approval of higher authorities. LMCEC also cannot claim as excuse from the reopening of its books of
accounts the previous investigations and examinations. Under Section 235 (a), an exception was
provided in the rule on once a year audit examination in case of "fraud, irregularity or mistakes, as
determined by the Commissioner". Petitioner explained that the distinction between a Regular Audit
Examination and Tax Fraud Audit Examination lies in the fact that the former is conducted by the
district offices of the Bureau’s Regional Offices, the authority emanating from the Regional Director,
while the latter is conducted by the TFD of the National Office only when instances of fraud had been
determined by the petitioner.22

Petitioner further asserted that LMCEC’s claim that it was granted immunity from audit when it availed
of the VAP and ERAP programs is misleading. LMCEC failed to state that its availment of ERAP
under RR No. 2-99 is not a grant of absolute immunity from audit and investigation, aside from the fact
that said program was only for income tax and did not cover VAT and withholding tax for the taxable
year 1998. As for LMCEC’S availment of VAP in 1999 under RR No. 8-2001 dated August 1, 2001 as
amended by RR No. 10-2001 dated September 3, 2001, the company failed to state that it covers only
income tax and VAT, and did not include withholding tax. However, LMCEC is not actually entitled to
the benefits of VAP under Section 1 (1.1 and 1.2) of RR No. 10-2001. As to the principle of estoppel
invoked by LMCEC, estoppel clearly does not lie against the BIR as this involved the exercise of an
inherent power by the government to collect taxes.23

Petitioner also pointed out that LMCEC’s assertion correlating this case with I.S. No. 00-956 is
misleading because said case involves another violation and offense (Sections 5 and 266 of the NIRC).
Said case was filed by petitioner due to the failure of LMCEC to submit or present its books of
accounts and other accounting records for examination despite the issuance of subpoena duces tecum
against Camus in his capacity as President of LMCEC. While indeed a Resolution was issued by Asst.
City Prosecutor Titus C. Borlas on May 2, 2001 dismissing the complaint, the same is still on appeal
and pending resolution by the DOJ. The determination of probable cause in said case is confined to the
57
issue of whether there was already a violation of the NIRC by Camus in not complying with the
subpoena duces tecum issued by the BIR.24

Petitioner contended that precisely the reason for the issuance to the TFD of LA No. 00009361 by the
Commissioner is because the latter agreed with the findings of the investigating revenue officers that
fraud exists in this case. In the conduct of their investigation, the revenue officers observed the proper
procedure under Revenue Memorandum Order (RMO) No. 49-2000 wherein it is required that before
the issuance of a Letter of Authority against a particular taxpayer, a preliminary investigation should
first be conducted to determine if a prima facie case for tax fraud exists. As to the allegedly unresolved
protest filed on April 20, 2001 by LMCEC over the PAN, this has been disregarded by the Bureau for
being pro forma and having been filed beyond the 15-day reglementary period. A subsequent letter
dated April 20, 2001 was filed with the TFD and signed by a certain Juan Ventigan. However, this was
disregarded and considered a mere scrap of paper since the said signatory had not shown any prior
authorization to represent LMCEC. Even assuming said protest letter was validly filed on behalf of the
company, the issuance of a Formal Demand Letter and Assessment Notice through constructive service
on October 1, 2002 is deemed an implied denial of the said protest. Lastly, the details regarding the
"informer" being confidential, such information is entitled to some degree of protection, including the
identity of the informant against LMCEC.25

In their Joint Rejoinder-Affidavit,26 Camus and Mendoza reiterated their argument that the identity of
the alleged informant is crucial to determine if he/she is qualified under Section 282 of the NIRC.
Moreover, there was no assessment that has already become final, the validity of its issuance and
service has been put in issue being anomalous, irregular and oppressive. It is contended that for
criminal prosecution to proceed before assessment, there must be a prima facie showing of a willful
attempt to evade taxes. As to LMCEC’s availment of the VAP and ERAP programs, the certificate of
immunity from audit issued to it by the BIR is plain and simple, but petitioner is now saying it has the
right to renege with impunity from its undertaking. Though petitioner deems LMCEC not qualified to
avail of the benefits of VAP, it must be noted that if it is true that at the time the petitioner filed I.S. No.
00-956 sometime in January 2001 it had already in its custody that "Confidential Information No. 29-
2000 dated July 7, 2000", these revenue officers could have rightly filed the instant case and would not
resort to filing said criminal complaint for refusal to comply with a subpoena duces tecum.

On September 22, 2003, the Chief State Prosecutor issued a Resolution27 finding no sufficient evidence
to establish probable cause against respondents LMCEC, Camus and Mendoza. It was held that since
the payments were made by LMCEC under ERAP and VAP pursuant to the provisions of RR Nos. 2-
99 and 8-2001 which were offered to taxpayers by the BIR itself, the latter is now in estoppel to insist
on the criminal prosecution of the respondent taxpayer. The voluntary payments made thereunder are in
the nature of a tax amnesty. The unnumbered assessment notices were found highly irregular and thus
their validity is suspect; if the amounts indicated therein were collected, it is uncertain how these will
be accounted for and if it would go to the coffers of the government or elsewhere. On the required prior
determination of fraud, the Chief State Prosecutor declared that the Office of the City Prosecutor in I.S.
No. 00-956 has already squarely ruled that (1) there was no prior determination of fraud, (2) there was
indiscriminate issuance of LAs, and (3) the complaint was more of harassment. In view of such
findings, any ensuing LA is thus defective and allowing the collection on the assailed assessment
notices would already be in the context of a "fishing expedition" or "witch-hunting." Consequently,
58
there is nothing to speak of regarding the finality of assessment notices in the aggregate amount of
₱630,164,631.61.

Petitioner filed a motion for reconsideration which was denied by the Chief State Prosecutor. 28

Petitioner appealed to respondent Secretary of Justice but the latter denied its petition for review under
Resolution dated December 13, 2005.29

The Secretary of Justice found that petitioner’s claim that there is yet no finality as to LMCEC’s
payment of its 1997 taxes since the audit report was still pending review by higher authorities, is
unsubstantiated and misplaced. It was noted that the Termination Letter issued by the Commissioner on
June 1, 1999 is explicit that the matter is considered closed. As for taxable year 1998, respondent
Secretary stated that the record shows that LMCEC paid VAT and withholding tax in the amount of
₱61,635.40 and ₱38,404.55, respectively. This eventually gave rise to the issuance of a certificate of
immunity from audit for 1998 by the Office of the Commissioner of Internal Revenue. For taxable year
1999, respondent Secretary found that pursuant to earlier LA No. 38633 dated July 4, 2000, LMCEC’s
1999 tax liabilities were still pending investigation for which reason LMCEC assailed the subsequent
issuance of LA No. 00009361 dated August 25, 2000 calling for a similar investigation of its alleged
1999 tax deficiencies when no final determination has yet been arrived on the earlier LA No. 38633.30

On the allegation of fraud, respondent Secretary ruled that petitioner failed to establish the existence of
the following circumstances indicating fraud in the settlement of LMCEC’s tax liabilities: (1) there
must be intentional and substantial understatement of tax liability by the taxpayer; (2) there must be
intentional and substantial overstatement of deductions or exemptions; and (3) recurrence of the
foregoing circumstances. First, petitioner miserably failed to explain why the assessment notices were
unnumbered; second, the claim that the tax fraud investigation was precipitated by an alleged
"informant" has not been corroborated nor was it clearly established, hence there is no other conclusion
but that the Bureau engaged in a "fishing expedition"; and furthermore, petitioner’s course of action is
contrary to Section 235 of the NIRC allowing only once in a given taxable year such examination and
inspection of the taxpayer’s books of accounts and other accounting records. There was no convincing
proof presented by petitioner to show that the case of LMCEC falls under the exceptions provided in
Section 235. Respondent Secretary duly considered the issuance of Certificate of Immunity from Audit
and Letter of Termination dated June 1, 1999 issued to LMCEC.31

Anent the earlier case filed against the same taxpayer (I.S. No. 00-956), the Secretary of Justice found
petitioner to have engaged in forum shopping in view of the fact that while there is still pending an
appeal from the Resolution of the City Prosecutor of Quezon City in said case, petitioner hurriedly filed
the instant case, which not only involved the same parties but also similar substantial issues (the joint
complaint-affidavit also alleged the issuance of LA No. 00009361 dated August 25, 2000). Clearly, the
evidence of litis pendentia is present. Finally, respondent Secretary noted that if indeed LMCEC
committed fraud in the settlement of its tax liabilities, then at the outset, it should have been discovered
by the agents of petitioner, and consequently petitioner should not have issued the Letter of
Termination and the Certificate of Immunity From Audit. Petitioner thus should have been more
circumspect in the issuance of said documents.32

59
Its motion for reconsideration having been denied, petitioner challenged the ruling of respondent
Secretary via a certiorari petition in the CA.

On October 31, 2006, the CA rendered the assailed decision33 denying the petition and concurred with
the findings and conclusions of respondent Secretary. Petitioner’s motion for reconsideration was
likewise denied by the appellate court.34 It appears that entry of judgment was issued by the CA stating
that its October 31, 2006 Decision attained finality on March 25, 2007.35 However, the said entry of
judgment was set aside upon manifestation by the petitioner that it has filed a petition for review before
this Court subsequent to its receipt of the Resolution dated March 6, 2007 denying petitioner’s motion
for reconsideration on March 20, 2007.36

The petition is anchored on the following grounds:

I.

The Honorable Court of Appeals erroneously sustained the findings of the Secretary of Justice who
gravely abused his discretion by dismissing the complaint based on grounds which are not even
elements of the offenses charged.

II.

The Honorable Court of Appeals erroneously sustained the findings of the Secretary of Justice who
gravely abused his discretion by dismissing petitioner’s evidence, contrary to law.

III.

The Honorable Court of Appeals erroneously sustained the findings of the Secretary of Justice who
gravely abused his discretion by inquiring into the validity of a Final Assessment Notice which has
become final, executory and demandable pursuant to Section 228 of the Tax Code of 1997 for failure
of private respondent to file a protest against the same.37

The core issue to be resolved is whether LMCEC and its corporate officers may be prosecuted for
violation of Sections 254 (Attempt to Evade or Defeat Tax) and 255 (Willful Failure to Supply Correct
and Accurate Information and Pay Tax).

Petitioner filed the criminal complaint against the private respondents for violation of the following
provisions of the NIRC, as amended:

SEC. 254. Attempt to Evade or Defeat Tax. – Any person who willfully attempts in any manner to
evade or defeat any tax imposed under this Code or the payment thereof shall, in addition to other
penalties provided by law, upon conviction thereof, be punished by a fine of not less than Thirty
thousand pesos (P30,000) but not more than One hundred thousand pesos (P100,000) and suffer
imprisonment of not less than two (2) years but not more than four (4) years: Provided, That the
conviction or acquittal obtained under this Section shall not be a bar to the filing of a civil suit for the
collection of taxes.

60
SEC. 255. Failure to File Return, Supply Correct and Accurate Information, Pay Tax, Withhold and
Remit Tax and Refund Excess Taxes Withheld on Compensation. – Any person required under this
Code or by rules and regulations promulgated thereunder to pay any tax, make a return, keep any
record, or supply any correct and accurate information, who willfully fails to pay such tax, make such
return, keep such record, or supply such correct and accurate information, or withhold or remit taxes
withheld, or refund excess taxes withheld on compensations at the time or times required by law or
rules and regulations shall, in addition to other penalties provided by law, upon conviction thereof, be
punished by a fine of not less than Ten thousand pesos (P10,000) and suffer imprisonment of not less
than one (1) year but not more than ten (10) years.

x x x x (Emphasis supplied.)

Respondent Secretary concurred with the Chief State Prosecutor’s conclusion that there is insufficient
evidence to establish probable cause to charge private respondents under the above provisions, based
on the following findings: (1) the tax deficiencies of LMCEC for taxable years 1997, 1998 and 1999
have all been settled or terminated, as in fact LMCEC was issued a Certificate of Immunity and Letter
of Termination, and availed of the ERAP and VAP programs; (2) there was no prior determination of
the existence of fraud; (3) the assessment notices are unnumbered, hence irregular and suspect; (4) the
books of accounts and other accounting records may be subject to audit examination only once in a
given taxable year and there is no proof that the case falls under the exceptions provided in Section 235
of the NIRC; and (5) petitioner committed forum shopping when it filed the instant case even as the
earlier criminal complaint (I.S. No. 00-956) dismissed by the City Prosecutor of Quezon City was still
pending appeal.

Petitioner argues that with the finality of the assessment due to failure of the private respondents to
challenge the same in accordance with Section 228 of the NIRC, respondent Secretary has no
jurisdiction and authority to inquire into its validity. Respondent taxpayer is thereby allowed to do
indirectly what it cannot do directly – to raise a collateral attack on the assessment when even a direct
challenge of the same is legally barred. The rationale for dismissing the complaint on the ground of
lack of control number in the assessment notice likewise betrays a lack of awareness of tax laws and
jurisprudence, such circumstance not being an element of the offense. Worse, the final, conclusive and
undisputable evidence detailing a crime under our taxation laws is swept under the rug so easily on
mere conspiracy theories imputed on persons who are not even the subject of the complaint.

We grant the petition.

There is no dispute that prior to the filing of the complaint with the DOJ, the report on the tax fraud
investigation conducted on LMCEC disclosed that it made substantial underdeclarations in its income
tax returns for 1997, 1998 and 1999. Pursuant to RR No. 12-99,38 a PAN was sent to and received by
LMCEC on February 22, 2001 wherein it was notified of the proposed assessment of deficiency taxes
amounting to ₱430,958,005.90 (income tax - ₱318,606,380.19 and VAT - ₱112,351,625.71) covering
taxable years 1997, 1998 and 1999.39 In response to said PAN, LMCEC sent a letter-protest to the
TFD, which denied the same on April 12, 2001 for lack of legal and factual basis and also for having
been filed beyond the 15-day reglementary period.40

61
As mentioned in the PAN, the revenue officers were not given the opportunity to examine LMCEC’s
books of accounts and other accounting records because its officers failed to comply with the subpoena
duces tecum earlier issued, to verify its alleged underdeclarations of income reported by the Bureau’s
informant under Section 282 of the NIRC. Hence, a criminal complaint was filed by the Bureau against
private respondents for violation of Section 266 which provides:

SEC. 266. Failure to Obey Summons. – Any person who, being duly summoned to appear to testify, or
to appear and produce books of accounts, records, memoranda, or other papers, or to furnish
information as required under the pertinent provisions of this Code, neglects to appear or to produce
such books of accounts, records, memoranda, or other papers, or to furnish such information, shall,
upon conviction, be punished by a fine of not less than Five thousand pesos (P5,000) but not more than
Ten thousand pesos (P10,000) and suffer imprisonment of not less than one (1) year but not more than
two (2) years.

It is clear that I.S. No. 00-956 involves a separate offense and hence litis pendentia is not present
considering that the outcome of I.S. No. 00-956 is not determinative of the issue as to whether probable
cause exists to charge the private respondents with the crimes of attempt to evade or defeat tax and
willful failure to supply correct and accurate information and pay tax defined and penalized under
Sections 254 and 255, respectively. For the crime of tax evasion in particular, compliance by the
taxpayer with such subpoena, if any had been issued, is irrelevant. As we held in Ungab v. Cusi,
Jr.,41 "[t]he crime is complete when the [taxpayer] has x x x knowingly and willfully filed [a]
fraudulent [return] with intent to evade and defeat x x x the tax." Thus, respondent Secretary erred in
holding that petitioner committed forum shopping when it filed the present criminal complaint during
the pendency of its appeal from the City Prosecutor’s dismissal of I.S. No. 00-956 involving the act of
disobedience to the summons in the course of the preliminary investigation on LMCEC’s correct tax
liabilities for taxable years 1997, 1998 and 1999.

In the Details of Discrepancies attached as Annex B of the PAN,42 private respondents were already
notified that inasmuch as the revenue officers were not given the opportunity to examine LMCEC’s
books of accounts, accounting records and other documents, said revenue officers gathered information
from third parties. Such procedure is authorized under Section 5 of the NIRC, which provides:

SEC. 5. Power of the Commissioner to Obtain Information, and to Summon, Examine, and Take
Testimony of Persons. – In ascertaining the correctness of any return, or in making a return when none
has been made, or in determining the liability of any person for any internal revenue tax, or in
collecting any such liability, or in evaluating tax compliance, the Commissioner is authorized:

(A) To examine any book, paper, record or other data which may be relevant or material to such
inquiry;

(B) To obtain on a regular basis from any person other than the person whose internal revenue
tax liability is subject to audit or investigation, or from any office or officer of the national and
local governments, government agencies and instrumentalities, including the Bangko Sentral ng
Pilipinas and government-owned or -controlled corporations, any information such as, but not
limited to, costs and volume of production, receipts or sales and gross incomes of taxpayers, and

62
the names, addresses, and financial statements of corporations, mutual fund companies,
insurance companies, regional operating headquarters of multinational companies, joint
accounts, associations, joint ventures or consortia and registered partnerships, and their
members;

(C) To summon the person liable for tax or required to file a return, or any officer or employee
of such person, or any person having possession, custody, or care of the books of accounts and
other accounting records containing entries relating to the business of the person liable for tax,
or any other person, to appear before the Commissioner or his duly authorized representative at
a time and place specified in the summons and to produce such books, papers, records, or other
data, and to give testimony;

(D) To take such testimony of the person concerned, under oath, as may be relevant or material
to such inquiry; x x x

x x x x (Emphasis supplied.)

Private respondents’ assertions regarding the qualifications of the "informer" of the Bureau deserve
scant consideration. We have held that the lack of consent of the taxpayer under investigation does not
imply that the BIR obtained the information from third parties illegally or that the information received
is false or malicious. Nor does the lack of consent preclude the BIR from assessing deficiency taxes on
the taxpayer based on the documents.43 In the same vein, herein private respondents cannot be allowed
to escape criminal prosecution under Sections 254 and 255 of the NIRC by mere imputation of a
"fictitious" or disqualified informant under Section 282 simply because other than disclosure of the
official registry number of the third party "informer," the Bureau insisted on maintaining the
confidentiality of the identity and personal circumstances of said "informer."

Subsequently, petitioner sent to LMCEC by constructive service allowed under Section 3 of RR No.
12-99, assessment notice and formal demand informing the said taxpayer of the law and the facts on
which the assessment is made, as required by Section 228 of the NIRC. Respondent Secretary,
however, fully concurred with private respondents’ contention that the assessment notices were invalid
for being unnumbered and the tax liabilities therein stated have already been settled and/or terminated.

We do not agree.

A notice of assessment is:

[A] declaration of deficiency taxes issued to a [t]axpayer who fails to respond to a Pre-Assessment
Notice (PAN) within the prescribed period of time, or whose reply to the PAN was found to be without
merit. The Notice of Assessment shall inform the [t]axpayer of this fact, and that the report of
investigation submitted by the Revenue Officer conducting the audit shall be given due course.

The formal letter of demand calling for payment of the taxpayer’s deficiency tax or taxes shall state
the fact, the law, rules and regulations or jurisprudence on which the assessment is based,
otherwise the formal letter of demand and the notice of assessment shall be void.44

63
As it is, the formality of a control number in the assessment notice is not a requirement for its validity
but rather the contents thereof which should inform the taxpayer of the declaration of deficiency tax
against said taxpayer. Both the formal letter of demand and the notice of assessment shall be void if the
former failed to state the fact, the law, rules and regulations or jurisprudence on which the assessment
is based, which is a mandatory requirement under Section 228 of the NIRC.

Section 228 of the NIRC provides that the taxpayer shall be informed in writing of the law and the facts
on which the assessment is made. Otherwise, the assessment is void. To implement the provisions of
Section 228 of the NIRC, RR No. 12-99 was enacted. Section 3.1.4 of the revenue regulation reads:

3.1.4. Formal Letter of Demand and Assessment Notice. – The formal letter of demand and assessment
notice shall be issued by the Commissioner or his duly authorized representative. The letter of
demand calling for payment of the taxpayer’s deficiency tax or taxes shall state the facts, the law,
rules and regulations, or jurisprudence on which the assessment is based, otherwise, the formal
letter of demand and assessment notice shall be void. The same shall be sent to the taxpayer only by
registered mail or by personal delivery. x x x.45 (Emphasis supplied.)

The Formal Letter of Demand dated August 7, 2002 contains not only a detailed computation of
LMCEC’s tax deficiencies but also details of the specified discrepancies, explaining the legal and
factual bases of the assessment. It also reiterated that in the absence of accounting records and other
documents necessary for the proper determination of the company’s internal revenue tax liabilities, the
investigating revenue officers resorted to the "Best Evidence Obtainable" as provided in Section 6(B)
of the NIRC (third party information) and in accordance with the procedure laid down in RMC No. 23-
2000 dated November 27, 2000. Annex "A" of the Formal Letter of Demand thus stated:

Thus, to verify the validity of the information previously provided by the informant, the assigned
revenue officers resorted to third party information. Pursuant to Section 5(B) of the NIRC of 1997,
access letters requesting for information and the submission of certain documents (i.e., Certificate of
Income Tax Withheld at Source and/or Alphabetical List showing the income payments made to L.M.
Camus Engineering Corporation for the taxable years 1997 to 1999) were sent to the various clients of
the subject corporation, including but not limited to the following:

1. Ayala Land Inc.

2. Filinvest Alabang Inc.

3. D.M. Consunji, Inc.

4. SM Prime Holdings, Inc.

5. Alabang Commercial Corporation

6. Philam Properties Corporation

7. SM Investments, Inc.

64
8. Shoemart, Inc.

9. Philippine Securities Corporation

10. Makati Development Corporation

From the documents gathered and the data obtained therein, the substantial underdeclaration as defined
under Section 248(B) of the NIRC of 1997 by your corporation of its income had been confirmed. x x x
x46 (Emphasis supplied.)

In the same letter, Assistant Commissioner Percival T. Salazar informed private respondents that the
estimated tax liabilities arising from LMCEC’s underdeclaration amounted to ₱186,773,600.84 in
1997, ₱150,069,323.81 in 1998 and ₱163,220,111.13 in 1999. These figures confirmed that the non-
declaration by LMCEC for the taxable years 1997, 1998 and 1999 of an amount exceeding 30%
income47 declared in its return is considered a substantial underdeclaration of income, which
constituted prima facie evidence of false or fraudulent return under Section 248(B)48 of the NIRC, as
amended.49

On the alleged settlement of the assessed tax deficiencies by private respondents, respondent Secretary
found the latter’s claim as meritorious on the basis of the Certificate of Immunity From Audit issued on
December 6, 1999 pursuant to RR No. 2-99 and Letter of Termination dated June 1, 1999 issued by
Revenue Region No. 7 Chief of Assessment Division Ruth Vivian G. Gandia. Petitioner, however,
clarified that the certificate of immunity from audit covered only income tax for the year 1997 and does
not include VAT and withholding taxes, while the Letter of Termination involved tax liabilities for
taxable year 1997 (EWT, VAT and income taxes) but which was submitted for review of higher
authorities as per the Certification of RD No. 40 District Officer Pablo C. Cabreros, Jr.50 For 1999,
private respondents supposedly availed of the VAP pursuant to RR No. 8-2001.

RR No. 2-99 issued on February 7, 1999 explained in its Policy Statement that considering the scarcity
of financial and human resources as well as the time constraints within which the Bureau has to "clean
the Bureau’s backlog of unaudited tax returns in order to keep updated and be focused with the most
current accounts" in preparation for the full implementation of a computerized tax administration, the
said revenue regulation was issued "providing for last priority in audit and investigation of tax returns"
to accomplish the said objective "without, however, compromising the revenue collection that would
have been generated from audit and enforcement activities." The program named as "Economic
Recovery Assistance Payment (ERAP) Program" granted immunity from audit and investigation of
income tax, VAT and percentage tax returns for 1998. It expressly excluded withholding tax returns
(whether for income, VAT, or percentage tax purposes). Since such immunity from audit and
investigation does not preclude the collection of revenues generated from audit and enforcement
activities, it follows that the Bureau is likewise not barred from collecting any tax deficiency
discovered as a result of tax fraud investigations. Respondent Secretary’s opinion that RR No. 2-99
contains the feature of a tax amnesty is thus misplaced.

Tax amnesty is a general pardon to taxpayers who want to start a clean tax slate. It also gives the
government a chance to collect uncollected tax from tax evaders without having to go through the

65
tedious process of a tax case.51Even assuming arguendo that the issuance of RR No. 2-99 is in the
nature of tax amnesty, it bears noting that a tax amnesty, much like a tax exemption, is never favored
nor presumed in law and if granted by statute, the terms of the amnesty like that of a tax exemption
must be construed strictly against the taxpayer and liberally in favor of the taxing authority.52

For the same reason, the availment by LMCEC of VAP under RR No. 8-2001 as amended by RR No.
10-2001, through payment supposedly made in October 29, 2001 before the said program ended on
October 31, 2001, did not amount to settlement of its assessed tax deficiencies for the period 1997 to
1999, nor immunity from prosecution for filing fraudulent return and attempt to evade or defeat tax. As
correctly asserted by petitioner, from the express terms of the aforesaid revenue regulations, LMCEC is
not qualified to avail of the VAP granting taxpayers the privilege of last priority in the audit and
investigation of all internal revenue taxes for the taxable year 2000 and all prior years under certain
conditions, considering that first, it was issued a PAN on February 19, 2001, and second, it was the
subject of investigation as a result of verified information filed by a Tax Informer under Section 282 of
the NIRC duly recorded in the BIR Official Registry as Confidential Information (CI) No. 29-
200053 even prior to the issuance of the PAN.

Section 1 of RR No. 8-2001 provides:

SECTION 1. COVERAGE. – x x x

Any person, natural or juridical, including estates and trusts, liable to pay any of the above-cited
internal revenue taxes for the above specified period/s who, due to inadvertence or otherwise,
erroneously paid his internal revenue tax liabilities or failed to file tax return/pay taxes may avail of the
Voluntary Assessment Program (VAP), except those falling under any of the following instances:

1.1 Those covered by a Preliminary Assessment Notice (PAN), Final Assessment Notice (FAN),
or Collection Letter issued on or before July 31, 2001; or

1.2 Persons under investigation as a result of verified information filed by a Tax Informer under
Section 282 of the Tax Code of 1997, duly processed and recorded in the BIR Official Registry
Book on or before July 31, 2001;

1.3 Tax fraud cases already filed and pending in courts for adjudication; and

x x x x (Emphasis supplied.)

Moreover, private respondents cannot invoke LMCEC’s availment of VAP to foreclose any subsequent
audit of its account books and other accounting records in view of the strong finding of
underdeclaration in LMCEC’s payment of correct income tax liability by more than 30% as supported
by the written report of the TFD detailing the facts and the law on which such finding is based,
pursuant to the tax fraud investigation authorized by petitioner under LA No. 00009361. This
conclusion finds support in Section 2 of RR No. 8-2001 as amended by RR No. 10-2001 provides:

SEC. 2. TAXPAYER’S BENEFIT FROM AVAILMENT OF THE VAP. – A taxpayer who has
availed of the VAP shall not be audited except upon authorization and approval of the Commissioner
66
of Internal Revenue when there is strong evidence or finding of understatement in the payment of
taxpayer’s correct tax liability by more than thirty percent (30%) as supported by a written report of the
appropriate office detailing the facts and the law on which such finding is based: Provided, however,
that any VAP payment should be allowed as tax credit against the deficiency tax due, if any, in case the
concerned taxpayer has been subjected to tax audit.

xxxx

Given the explicit conditions for the grant of immunity from audit under RR No. 2-99, RR No. 8-2001
and RR No. 10-2001, we hold that respondent Secretary gravely erred in declaring that petitioner is
now estopped from assessing any tax deficiency against LMCEC after issuance of the aforementioned
documents of immunity from audit/investigation and settlement of tax liabilities. It is axiomatic that the
State can never be in estoppel, and this is particularly true in matters involving taxation. The errors of
certain administrative officers should never be allowed to jeopardize the government’s financial
position.54

Respondent Secretary’s other ground for assailing the course of action taken by petitioner in
proceeding with the audit and investigation of LMCEC -- the alleged violation of the general rule in
Section 235 of the NIRC allowing the examination and inspection of taxpayer’s books of accounts and
other accounting records only once in a taxable year -- is likewise untenable. As correctly pointed out
by petitioner, the discovery of substantial underdeclarations of income by LMCEC for taxable years
1997, 1998 and 1999 upon verified information provided by an "informer" under Section 282 of the
NIRC, as well as the necessity of obtaining information from third parties to ascertain the correctness
of the return filed or evaluation of tax compliance in collecting taxes (as a result of the disobedience to
the summons issued by the Bureau against the private respondents), are circumstances warranting
exception from the general rule in Section 235.55

As already stated, the substantial underdeclared income in the returns filed by LMCEC for 1997, 1998
and 1999 in amounts equivalent to more than 30% (the computation in the final assessment notice
showed underdeclarations of almost 200%) constitutes prima facie evidence of fraudulent return under
Section 248(B) of the NIRC. Prior to the issuance of the preliminary and final notices of assessment,
the revenue officers conducted a preliminary investigation on the information and documents showing
substantial understatement of LMCEC’s tax liabilities which were provided by the Informer, following
the procedure under RMO No. 15-95.56 Based on the prima facie finding of the existence of fraud,
petitioner issued LA No. 00009361 for the TFD to conduct a formal fraud investigation of
LMCEC.57 Consequently, respondent Secretary’s ruling that the filing of criminal complaint for
violation of Sections 254 and 255 of the NIRC cannot prosper because of lack of prior determination of
the existence of fraud, is bereft of factual basis and contradicted by the evidence on record.

Tax assessments by tax examiners are presumed correct and made in good faith, and all presumptions
are in favor of the correctness of a tax assessment unless proven otherwise.58 We have held that a
taxpayer’s failure to file a petition for review with the Court of Tax Appeals within the statutory period
rendered the disputed assessment final, executory and demandable, thereby precluding it from
interposing the defenses of legality or validity of the assessment and prescription of the Government’s
right to assess.59 Indeed, any objection against the assessment should have been pursued following the
67
avenue paved in Section 229 (now Section 228) of the NIRC on protests on assessments of internal
revenue taxes.60

Records bear out that the assessment notice and Formal Letter of Demand dated August 7, 2002 were
duly served on LMCEC on October 1, 2002. Private respondents did not file a motion for
reconsideration of the said assessment notice and formal demand; neither did they appeal to the Court
of Tax Appeals. Section 228 of the NIRC61 provides the remedy to dispute a tax assessment within a
certain period of time. It states that an assessment may be protested by filing a request for
reconsideration or reinvestigation within 30 days from receipt of the assessment by the taxpayer. No
such administrative protest was filed by private respondents seeking reconsideration of the August 7,
2002 assessment notice and formal letter of demand. Private respondents cannot belatedly assail the
said assessment, which they allowed to lapse into finality, by raising issues as to its validity and
correctness during the preliminary investigation after the BIR has referred the matter for prosecution
under Sections 254 and 255 of the NIRC.

As we held in Marcos II v. Court of Appeals62:

It is not the Department of Justice which is the government agency tasked to determine the amount of
taxes due upon the subject estate, but the Bureau of Internal Revenue, whose determinations and
assessments are presumed correct and made in good faith. The taxpayer has the duty of proving
otherwise. In the absence of proof of any irregularities in the performance of official duties, an
assessment will not be disturbed. Even an assessment based on estimates is prima facie valid and
lawful where it does not appear to have been arrived at arbitrarily or capriciously. The burden of proof
is upon the complaining party to show clearly that the assessment is erroneous. Failure to present proof
of error in the assessment will justify the judicial affirmance of said assessment. x x x.

Moreover, these objections to the assessments should have been raised, considering the ample remedies
afforded the taxpayer by the Tax Code, with the Bureau of Internal Revenue and the Court of Tax
Appeals, as described earlier, and cannot be raised now via Petition for Certiorari, under the pretext of
grave abuse of discretion. The course of action taken by the petitioner reflects his disregard or even
repugnance of the established institutions for governance in the scheme of a well-ordered society. The
subject tax assessments having become final, executory and enforceable, the same can no longer be
contested by means of a disguised protest. In the main, Certiorari may not be used as a substitute for a
lost appeal or remedy. This judicial policy becomes more pronounced in view of the absence of
sufficient attack against the actuations of government. (Emphasis supplied.)

The determination of probable cause is part of the discretion granted to the investigating prosecutor and
ultimately, the Secretary of Justice. However, this Court and the CA possess the power to review
findings of prosecutors in preliminary investigations. Although policy considerations call for the widest
latitude of deference to the prosecutor’s findings, courts should never shirk from exercising their
power, when the circumstances warrant, to determine whether the prosecutor’s findings are supported
by the facts, or by the law. In so doing, courts do not act as prosecutors but as organs of the judiciary,
exercising their mandate under the Constitution, relevant statutes, and remedial rules to settle cases and
controversies.63 Clearly, the power of the Secretary of Justice to review does not preclude this Court
and the CA from intervening and exercising our own powers of review with respect to the DOJ’s
68
findings, such as in the exceptional case in which grave abuse of discretion is committed, as when a
clear sufficiency or insufficiency of evidence to support a finding of probable cause is ignored.64

WHEREFORE, the petition is GRANTED. The Decision dated October 31, 2006 and Resolution dated
March 6, 2007 of the Court of Appeals in CA-G.R. SP No. 93387 are hereby REVERSED and SET
ASIDE. The Secretary of Justice is hereby DIRECTED to order the Chief State Prosecutor to file
before the Regional Trial Court of Quezon City, National Capital Judicial Region, the corresponding
Information against L. M. Camus Engineering Corporation, represented by its President Luis M.
Camus and Comptroller Lino D. Mendoza, for Violation of Sections 254 and 255 of the National
Internal Revenue Code of 1997.

No costs.SO ORDERED.

DIGEST: SEPARATE FILE

G.R. No. 178697 November 17, 2010

COMMISSIONER OF INTERNAL REVENUE, Petitioner,


vs.
SONY PHILIPPINES, INC., Respondent.

DECISION

MENDOZA, J.:

This petition for review on certiorari seeks to set aside the May 17, 2007 Decision and the July 5, 2007
Resolution of the Court of Tax Appeals – En Banc1 (CTA-EB), in C.T.A. EB No. 90, affirming the
October 26, 2004 Decision of the CTA-First Division2 which, in turn, partially granted the petition for
review of respondent Sony Philippines, Inc. (Sony). The CTA-First Division decision cancelled the
deficiency assessment issued by petitioner Commissioner of Internal Revenue (CIR) against Sony for
Value Added Tax (VAT) but upheld the deficiency assessment for expanded withholding tax (EWT) in
the amount of ₱1,035,879.70 and the penalties for late remittance of internal revenue taxes in the
amount of ₱1,269, 593.90.3

THE FACTS:

On November 24, 1998, the CIR issued Letter of Authority No. 000019734 (LOA 19734) authorizing
certain revenue officers to examine Sony’s books of accounts and other accounting records regarding
revenue taxes for "the period 1997 and unverified prior years." On December 6, 1999, a
preliminary assessment for 1997 deficiency taxes and penalties was issued by the CIR which Sony
protested. Thereafter, acting on the protest, the CIR issued final assessment notices, the formal letter of
demand and the details of discrepancies.4 Said details of the deficiency taxes and penalties for late
remittance of internal revenue taxes are as follows:

69
DEFICIENCY VALUE -
ADDED TAX (VAT)
(Assessment No. ST-VAT-97-
0124-2000)
Basic Tax Due P 7,958,700.00
Add: Penalties
Interest up to 3-31-2000 P 3,157,314.41
Compromise 25,000.00 3,182,314.41
Deficiency VAT Due P 11,141,014.41

DEFICIENCY EXPANDED
WITHHOLDING TAX
(EWT)
(Assessment No. ST-EWT-97-
0125-2000)
Basic Tax Due P 1,416,976.90
Add: Penalties
Interest up to 3-31-2000 P 550,485.82
Compromise 25,000.00 575,485.82
Deficiency EWT Due P 1,992,462.72

DEFICIENCY OF VAT ON
ROYALTY PAYMENTS
(Assessment No. ST-LR1-97-
0126-2000)
Basic Tax Due P
Add: Penalties
Surcharge P 359,177.80
Interest up to 3-31-2000 87,580.34
Compromise 16,000.00 462,758.14
Penalties Due P 462,758.14

LATE REMITTANCE OF
FINAL WITHHOLDING
TAX
70
(Assessment No. ST-LR2-97-
0127-2000)
Basic Tax Due P
Add: Penalties
Surcharge P 1,729,690.71
Interest up to 3-31-2000 508,783.07
Compromise 50,000.00 2,288,473.78
Penalties Due P 2,288,473.78

LATE REMITTANCE OF
INCOME PAYMENTS
(Assessment No. ST-LR3-97-
0128-2000)
Basic Tax Due P
Add: Penalties
25 % Surcharge P 8,865.34
Interest up to 3-31-2000 58.29
Compromise 2,000.00 10,923.60
Penalties Due P 10,923.60

GRAND TOTAL P 15,895,632.655

Sony sought re-evaluation of the aforementioned assessment by filing a protest on February 2, 2000.
Sony submitted relevant documents in support of its protest on the 16th of that same month.6

On October 24, 2000, within 30 days after the lapse of 180 days from submission of the said supporting
documents to the CIR, Sony filed a petition for review before the CTA.7

After trial, the CTA-First Division disallowed the deficiency VAT assessment because the subsidized
advertising expense paid by Sony which was duly covered by a VAT invoice resulted in an input VAT
credit. As regards the EWT, the CTA-First Division maintained the deficiency EWT assessment on
Sony’s motor vehicles and on professional fees paid to general professional partnerships. It also
assessed the amounts paid to sales agents as commissions with five percent (5%) EWT pursuant to
Section 1(g) of Revenue Regulations No. 6-85. The CTA-First Division, however, disallowed the EWT
assessment on rental expense since it found that the total rental deposit of ₱10,523,821.99 was incurred
71
from January to March 1998 which was again beyond the coverage of LOA 19734. Except for the
compromise penalties, the CTA-First Division also upheld the penalties for the late payment of VAT
on royalties, for late remittance of final withholding tax on royalty as of December 1997 and for the
late remittance of EWT by some of Sony’s branches.8 In sum, the CTA-First Division partly granted
Sony’s petition by cancelling the deficiency VAT assessment but upheld a modified deficiency EWT
assessment as well as the penalties. Thus, the dispositive portion reads:

WHEREFORE, the petition for review is hereby PARTIALLY GRANTED. Respondent is ORDERED
to CANCEL and WITHDRAW the deficiency assessment for value-added tax for 1997 for lack of
merit. However, the deficiency assessments for expanded withholding tax and penalties for late
remittance of internal revenue taxes are UPHELD.

Accordingly, petitioner is DIRECTED to PAY the respondent the deficiency expanded withholding tax
in the amount of ₱1,035,879.70 and the following penalties for late remittance of internal revenue taxes
in the sum of ₱1,269,593.90:

1. VAT on Royalty P 429,242.07


2. Withholding Tax on Royalty 831,428.20
3. EWT of Petitioner's Branches 8,923.63
Total P 1,269,593.90

Plus 20% delinquency interest from January 17, 2000 until fully paid pursuant to Section 249(C)(3) of
the 1997 Tax Code.

SO ORDERED.9

The CIR sought a reconsideration of the above decision and submitted the following grounds in support
thereof:

A. The Honorable Court committed reversible error in holding that petitioner is not liable for the
deficiency VAT in the amount of ₱11,141,014.41;

B. The Honorable court committed reversible error in holding that the commission expense in
the amount of P2,894,797.00 should be subjected to 5% withholding tax instead of the 10% tax
rate;

C. The Honorable Court committed a reversible error in holding that the withholding tax
assessment with respect to the 5% withholding tax on rental deposit in the amount of
₱10,523,821.99 should be cancelled; and

D. The Honorable Court committed reversible error in holding that the remittance of final
withholding tax on royalties covering the period January to March 1998 was filed on time.10

72
On April 28, 2005, the CTA-First Division denied the motion for reconsideration.1avvphi1 Unfazed,
the CIR filed a petition for review with the CTA-EB raising identical issues:

1. Whether or not respondent (Sony) is liable for the deficiency VAT in the amount of
P11,141,014.41;

2. Whether or not the commission expense in the amount of ₱2,894,797.00 should be subjected
to 10% withholding tax instead of the 5% tax rate;

3. Whether or not the withholding assessment with respect to the 5% withholding tax on rental
deposit in the amount of ₱10,523,821.99 is proper; and

4. Whether or not the remittance of final withholding tax on royalties covering the period
January to March 1998 was filed outside of time.11

Finding no cogent reason to reverse the decision of the CTA-First Division, the CTA-EB dismissed
CIR’s petition on May 17, 2007. CIR’s motion for reconsideration was denied by the CTA-EB on July
5, 2007.

The CIR is now before this Court via this petition for review relying on the very same grounds it raised
before the CTA-First Division and the CTA-EB. The said grounds are reproduced below:

GROUNDS FOR THE ALLOWANCE OF THE PETITION

THE CTA EN BANC ERRED IN RULING THAT RESPONDENT IS NOT LIABLE FOR
DEFICIENCY VAT IN THE AMOUNT OF PHP11,141,014.41.

II

AS TO RESPONDENT’S DEFICIENCY EXPANDED WITHHOLDING TAX IN THE


AMOUNT OF PHP1,992,462.72:

A. THE CTA EN BANC ERRED IN RULING THAT THE COMMISSION


EXPENSE IN THE AMOUNT OF PHP2,894,797.00 SHOULD BE SUBJECTED
TO A WITHHOLDING TAX OF 5% INSTEAD OF THE 10% TAX RATE.

B. THE CTA EN BANC ERRED IN RULING THAT THE ASSESSMENT WITH


RESPECT TO THE 5% WITHHOLDING TAX ON RENTAL DEPOSIT IN THE
AMOUNT OF PHP10,523,821.99 IS NOT PROPER.

III

73
THE CTA EN BANC ERRED IN RULING THAT THE FINAL WITHHOLDING TAX ON
ROYALTIES COVERING THE PERIOD JANUARY TO MARCH 1998 WAS FILED ON
TIME.12

Upon filing of Sony’s comment, the Court ordered the CIR to file its reply thereto. The CIR
subsequently filed a manifestation informing the Court that it would no longer file a reply. Thus, on
December 3, 2008, the Court resolved to give due course to the petition and to decide the case on the
basis of the pleadings filed.13

The Court finds no merit in the petition.

The CIR insists that LOA 19734, although it states "the period 1997 and unverified prior years," should
be understood to mean the fiscal year ending in March 31, 1998.14 The Court cannot agree.

Based on Section 13 of the Tax Code, a Letter of Authority or LOA is the authority given to the
appropriate revenue officer assigned to perform assessment functions. It empowers or enables said
revenue officer to examine the books of account and other accounting records of a taxpayer for the
purpose of collecting the correct amount of tax.15 The very provision of the Tax Code that the CIR
relies on is unequivocal with regard to its power to grant authority to examine and assess a taxpayer.

SEC. 6. Power of the Commissioner to Make Assessments and Prescribe Additional Requirements for
Tax Administration and Enforcement. –

(A)Examination of Returns and Determination of tax Due. – After a return has been filed as required
under the provisions of this Code, the Commissioner or his duly authorized representative may
authorize the examination of any taxpayer and the assessment of the correct amount of tax: Provided,
however, That failure to file a return shall not prevent the Commissioner from authorizing the
examination of any taxpayer. x x x [Emphases supplied]

Clearly, there must be a grant of authority before any revenue officer can conduct an examination or
assessment. Equally important is that the revenue officer so authorized must not go beyond the
authority given. In the absence of such an authority, the assessment or examination is a nullity.

As earlier stated, LOA 19734 covered "the period 1997 and unverified prior years." For said reason, the
CIR acting through its revenue officers went beyond the scope of their authority because the deficiency
VAT assessment they arrived at was based on records from January to March 1998 or using the fiscal
year which ended in March 31, 1998. As pointed out by the CTA-First Division in its April 28, 2005
Resolution, the CIR knew which period should be covered by the investigation. Thus, if CIR wanted or
intended the investigation to include the year 1998, it should have done so by including it in the LOA
or issuing another LOA.

Upon review, the CTA-EB even added that the coverage of LOA 19734, particularly the phrase "and
unverified prior years," violated Section C of Revenue Memorandum Order No. 43-90 dated September
20, 1990, the pertinent portion of which reads:

74
3. A Letter of Authority should cover a taxable period not exceeding one taxable year. The practice
of issuing L/As covering audit of "unverified prior years is hereby prohibited. If the audit of a taxpayer
shall include more than one taxable period, the other periods or years shall be specifically indicated in
the L/A.16 [Emphasis supplied]

On this point alone, the deficiency VAT assessment should have been disallowed. Be that as it may, the
CIR’s argument, that Sony’s advertising expense could not be considered as an input VAT credit
because the same was eventually reimbursed by Sony International Singapore (SIS), is also erroneous.

The CIR contends that since Sony’s advertising expense was reimbursed by SIS, the former never
incurred any advertising expense. As a result, Sony is not entitled to a tax credit. At most, the CIR
continues, the said advertising expense should be for the account of SIS, and not Sony.17

The Court is not persuaded. As aptly found by the CTA-First Division and later affirmed by the CTA-
EB, Sony’s deficiency VAT assessment stemmed from the CIR’s disallowance of the input VAT
credits that should have been realized from the advertising expense of the latter.18 It is evident under
Section 11019 of the 1997 Tax Code that an advertising expense duly covered by a VAT invoice is a
legitimate business expense. This is confirmed by no less than CIR’s own witness, Revenue Officer
Antonio Aluquin.20 There is also no denying that Sony incurred advertising expense. Aluquin testified
that advertising companies issued invoices in the name of Sony and the latter paid for the
same.21 Indubitably, Sony incurred and paid for advertising expense/ services. Where the money came
from is another matter all together but will definitely not change said fact.

The CIR further argues that Sony itself admitted that the reimbursement from SIS was income and,
thus, taxable. In support of this, the CIR cited a portion of Sony’s protest filed before it:

The fact that due to adverse economic conditions, Sony-Singapore has granted to our client a subsidy
equivalent to the latter’s advertising expenses will not affect the validity of the input taxes from such
expenses. Thus, at the most, this is an additional income of our client subject to income tax. We submit
further that our client is not subject to VAT on the subsidy income as this was not derived from the sale
of goods or services.22

Insofar as the above-mentioned subsidy may be considered as income and, therefore, subject to income
tax, the Court agrees. However, the Court does not agree that the same subsidy should be subject to the
10% VAT. To begin with, the said subsidy termed by the CIR as reimbursement was not even
exclusively earmarked for Sony’s advertising expense for it was but an assistance or aid in view of
Sony’s dire or adverse economic conditions, and was only "equivalent to the latter’s (Sony’s)
advertising expenses."

Section 106 of the Tax Code explains when VAT may be imposed or exacted. Thus:

SEC. 106. Value-added Tax on Sale of Goods or Properties. –

(A) Rate and Base of Tax. – There shall be levied, assessed and collected on every sale, barter or
exchange of goods or properties, value-added tax equivalent to ten percent (10%) of the gross selling

75
price or gross value in money of the goods or properties sold, bartered or exchanged, such tax to be
paid by the seller or transferor.

Thus, there must be a sale, barter or exchange of goods or properties before any VAT may be levied.
Certainly, there was no such sale, barter or exchange in the subsidy given by SIS to Sony. It was but a
dole out by SIS and not in payment for goods or properties sold, bartered or exchanged by Sony.

In the case of CIR v. Court of Appeals (CA),23 the Court had the occasion to rule that services rendered
for a fee even on reimbursement-on-cost basis only and without realizing profit are also subject to
VAT. The case, however, is not applicable to the present case. In that case, COMASERCO rendered
service to its affiliates and, in turn, the affiliates paid the former reimbursement-on-cost which means
that it was paid the cost or expense that it incurred although without profit. This is not true in the
present case. Sony did not render any service to SIS at all. The services rendered by the advertising
companies, paid for by Sony using SIS dole-out, were for Sony and not SIS. SIS just gave assistance to
Sony in the amount equivalent to the latter’s advertising expense but never received any goods,
properties or service from Sony.

Regarding the deficiency EWT assessment, more particularly Sony’s commission expense, the CIR
insists that said deficiency EWT assessment is subject to the ten percent (10%) rate instead of the five
percent (5%) citing Revenue Regulation No. 2-98 dated April 17, 1998.24 The said revenue regulation
provides that the 10% rate is applied when the recipient of the commission income is a natural person.
According to the CIR, Sony’s schedule of Selling, General and Administrative expenses shows the
commission expense as "commission/dealer salesman incentive," emphasizing the word salesman.

On the other hand, the application of the five percent (5%) rate by the CTA-First Division is based on
Section 1(g) of Revenue Regulations No. 6-85 which provides:

(g) Amounts paid to certain Brokers and Agents. – On gross payments to customs, insurance, real
estate and commercial brokers and agents of professional entertainers – five per centum (5%).25

In denying the very same argument of the CIR in its motion for reconsideration, the CTA-First
Division, held:

x x x, commission expense is indeed subject to 10% withholding tax but payments made to broker is
subject to 5% withholding tax pursuant to Section 1(g) of Revenue Regulations No. 6-85. While the
commission expense in the schedule of Selling, General and Administrative expenses submitted by
petitioner (SPI) to the BIR is captioned as "commission/dealer salesman incentive" the same does not
justify the automatic imposition of flat 10% rate. As itemized by petitioner, such expense is composed
of "Commission Expense" in the amount of P10,200.00 and ‘Broker Dealer’ of P2,894,797.00.26

The Court agrees with the CTA-EB when it affirmed the CTA-First Division decision. Indeed, the
applicable rule is Revenue Regulations No. 6-85, as amended by Revenue Regulations No. 12-94,
which was the applicable rule during the subject period of examination and assessment as specified in
the LOA. Revenue Regulations No. 2-98, cited by the CIR, was only adopted in April 1998 and,
therefore, cannot be applied in the present case. Besides, the withholding tax on brokers and agents was

76
only increased to 10% much later or by the end of July 2001 under Revenue Regulations No. 6-
2001.27 Until then, the rate was only 5%.

The Court also affirms the findings of both the CTA-First Division and the CTA-EB on the deficiency
EWT assessment on the rental deposit. According to their findings, Sony incurred the subject rental
deposit in the amount of ₱10,523,821.99 only from January to March 1998. As stated earlier, in the
absence of the appropriate LOA specifying the coverage, the CIR’s deficiency EWT assessment from
January to March 1998, is not valid and must be disallowed.

Finally, the Court now proceeds to the third ground relied upon by the CIR.

The CIR initially assessed Sony to be liable for penalties for belated remittance of its FWT on royalties
(i) as of December 1997; and (ii) for the period from January to March 1998. Again, the Court agrees
with the CTA-First Division when it upheld the CIR with respect to the royalties for December 1997
but cancelled that from January to March 1998.

The CIR insists that under Section 328 of Revenue Regulations No. 5-82 and Sections 2.57.4 and
2.58(A)(2)(a)29 of Revenue Regulations No. 2-98, Sony should also be made liable for the FWT on
royalties from January to March of 1998. At the same time, it downplays the relevance of the
Manufacturing License Agreement (MLA) between Sony and Sony-Japan, particularly in the payment
of royalties.

The above revenue regulations provide the manner of withholding remittance as well as the payment of
final tax on royalty. Based on the same, Sony is required to deduct and withhold final taxes on royalty
payments when the royalty is paid or is payable. After which, the corresponding return and remittance
must be made within 10 days after the end of each month. The question now is when does the royalty
become payable?

Under Article X(5) of the MLA between Sony and Sony-Japan, the following terms of royalty
payments were agreed upon:

(5)Within two (2) months following each semi-annual period ending June 30 and December 31, the
LICENSEE shall furnish to the LICENSOR a statement, certified by an officer of the LICENSEE,
showing quantities of the MODELS sold, leased or otherwise disposed of by the LICENSEE during
such respective semi-annual period and amount of royalty due pursuant this ARTICLE X therefore, and
the LICENSEE shall pay the royalty hereunder to the LICENSOR concurrently with the furnishing of
the above statement.30

Withal, Sony was to pay Sony-Japan royalty within two (2) months after every semi-annual period
which ends in June 30 and December 31. However, the CTA-First Division found that there was
accrual of royalty by the end of December 1997 as well as by the end of June 1998. Given this, the
FWTs should have been paid or remitted by Sony to the CIR on January 10, 1998 and July 10, 1998.
Thus, it was correct for the CTA-First Division and the CTA-EB in ruling that the FWT for the royalty
from January to March 1998 was seasonably filed. Although the royalty from January to March 1998
was well within the semi-annual period ending June 30, which meant that the royalty may be payable

77
until August 1998 pursuant to the MLA, the FWT for said royalty had to be paid on or before July 10,
1998 or 10 days from its accrual at the end of June 1998. Thus, when Sony remitted the same on July 8,
1998, it was not yet late.

In view of the foregoing, the Court finds no reason to disturb the findings of the CTA-EB.

WHEREFORE, the petition is DENIED.SO ORDERED.

DIGEST:

FACTS:
Sony Philippines was ordered examined for “the period 1997 and unverified prior years” as indicated in
the Letter of Authority. The audit yielded assessments against Sony Philippines for deficiency VAT
and FWT, viz: (1) late remittance of Final Withholding Tax on royalties for the period January to
March 1998 and (2) deficiency VAT on reimbursable received by Sony Philippines from its offshore
affiliate, Sony International Singapore (SIS).

ISSUES:
(1) Is Petitioner liable for deficiency Value Added Tax?
(2) Was the investigation of its 1998 Final Withholding Tax return valid?

HELD:
(1) NO. Sony Philippines did in fact incur expenses supported by valid VAT invoices when it paid for
certain advertising costs. This is sufficient to accord it the benefit of input VAT credits and where the
money came from to satisfy said advertising billings is another matter but does not alter the VAT
effect. In the same way, Sony Philippines can not be deemed to have received the reimbursable as a fee
for a VAT-taxable activity. The reimbursable was couched as an aid for Sony Philippines by SIS in
view of the company’s “dire or adverse economic conditions”. More importantly, the absence of a sale,
barter or exchange of goods or properties supports the non-VAT nature of the reimbursement. This was
distinguished from the COMASERCO case where even if there was similarly a reimbursement-on-cost
arrangement between affiliates, there was in fact an underlying service. Here, the advertising services
were rendered in favor of Sony Philippines not SIS.

(2) NO. A Letter of Authority should cover a taxable period not exceeding one year and to indicate
that it covers ‘unverified prior years’ should be enough to invalidate it. In addition, even if the Final
Withholding Tax was covered by Sony Philippines’ fiscal year ending March 1998, the same fell
outside of ‘the period 1997’ and was thus not validly covered by the Letter of Authority.

4. PRESCRIPTION OG GOVERNMENT’S RIGHT TO ACCESS


78
G.R. No. 104171 February 24, 1999

COMMISSIONER OF INTERNAL REVENUE, petitioner,


vs.
B.F. GOODRICH PHILS., INC. (now SIME DARBY INTERNATIONAL TIRE CO., INC.) and
THE COURT OF APPEALS, respondents.

PANGANIBAN, J.:

Notwithstanding the expiration of the five-year prescriptive period, may the Bureau of Internal
Revenue (BIR) still assess a taxpayer even after the latter has already paid the tax due, on the ground
that the previous assessment was insufficient or based on a "false" return?

The Case

This is the main question raised before us in this Petition for Review on Certiorari assailing the
Decision 1 dated February 14, 1992, promulgated by the Court of Appeals 2 in CA-GR SP No. 25100.
The assailed Decision reversed the Court of Tax Appeals (CTA) 3 which upheld the BIR
commissioner's assessments made beyond the five-year statute of limitations.

The Facts

The facts undisputed. 4 Private Respondent BF Goodrich Phils., Inc. (now Sime Darby International
Tire Co, Inc.), was an American-owned and controlled corporation previous to July 3, 1974. As a
condition for approving the manufacture by private respondent of tires and other rubber products, the
Central Bank of the Philippines required that it should develop a rubber plantation. In compliance with
this requirement, private respondent purchased from the Philippine government in 1961, under the
Public Land Act and the Parity Amendment to the 1935 Constitution, certain parcels of land located in
Tumajubong, Basilan, and there developed a rubber plantation.

More than a decade later, on August 2, 1973, the justice secretary rendered an opinion stating that,
upon the expiration of the Parity Amendment on July 3, 1974, the ownership rights of Americans over
public agricultural lands, including the right to dispose or sell their real estate, would be lost. On the
basis of this Opinion, private respondent sold to Siltown Realty Philippines, Inc. on January 21, 1974,
its Basilan landholding for P500,000 payable in installments. In accord with the terms of the sale,
Siltown Realty Philippines, Inc. leased the said parcels of land to private respondent for a period of 25
years, with an extension of another 25 years at the latter's option.

Based on the BIR's Letter of Authority No. 10115 dated April 14, 1975, the books and accounts of
private respondent were examined for the purpose of determining its tax liability for taxable year 1974.
The examination resulted in the April 23, 1975 assessment of private respondent for deficiency income
tax in the amount of P6,005.35, which it duly paid.

79
Subsequently, the BIR also issued Letters of Authority Nos. 074420 RR and 074421 RR and
Memorandum Authority Reference No. 749157 for the purpose of examining Siltown's business,
income and tax liabilities. On the basis of this examination, the BIR commissioner issued against
private respondent on October 10, 1980, an assessment for deficiency in donor's tax in the amount of
P1,020,850, in relation to the previously mentioned sale of its Basilan landholdings to Siltown.
Apparently, the BIR deemed the consideration for the sale insufficient, and the difference between the
fair market value and the actual purchase price a taxable donation.

In a letter dated November 24, 1980, private respondent contested this assessment. On April 9, 1981, it
received another assessment dated March 16, 1981, which increased to P 1,092,949 the amount
demanded for the alleged deficiency donor's tax, surcharge, interest and compromise penalty.

Private respondent appealed the correctness and the legality of these last two assessments to the CTA.
After trial in due course, the CTA rendered its Decision dated March 29, 1991, the dispositive portion
of which reads as follows:

WHEREFORE, the decision of the Commissioner of Internal Revenue assessing


petitioner deficiency gift tax is MODIFIED land petitioner is ordered to pay the amount
of P1,311,179.01 plus 10% surcharge and 20% annual interest from March 16, 1981 until
fully paid provided that the maximum amount that may be collected as interest on
delinquency shall in no case exceed an amount corresponding to a period of three years
pursuant to Section 130(b)(l) and (c) of the 1977 Tax Code, as amended by P.D. No.
1705, which took effect on August 1, 1980.

SO ORDERED. 5

Undaunted, private respondent elevated the matter to the Court of Appeals, which reversed the CTA, as
follows:

What is involved here is not a first assessment; nor is it one within the 5-year period
stated in Section 331 above. Since what is involved in this case is a multiple assessment
beyond the five-year period, the assessment must be based on the grounds provided in
Section 337, and not on Section 15 of the 1974 Tax Code. Section 337 utilizes the very
specific terms "fraud, irregularity, and mistake". "Falsity does not appear to be included
in this enumeration. Falsity suffices for an assessment, which is a firstassessment made
within the five-year period. When it is a subsequent assessment made beyond the five-
year period, then, it may be validly justified only by "fraud, irregularity and mistake" on
the part of the
taxpayer.6

Hence, this Petition for Review under Rule 45 of the Rules of Court. 7

The Issues

Before us, petitioner raises the following issues:

80
I

Whether or not petitioner's right to assess herein deficiency donor's tax has indeed
prescribed as ruled by public respondent Court of Appeals

II

Whether or not the herein deficiency donor's tax assessment for 1974 is valid and in
accordance with law

Prescription is the crucial issue in the resolution of this case.

The Court's Ruling

The petition has no merit.

Main Issue: Prescription

The petitioner contends that the Court of Appeals erred in reversing the CTA on the issue of
prescription, because its ruling was based on factual findings that should have been left undisturbed on
appeal, in the absence of any showing that it had been tainted with gross error or grave abuse of
discretion. 8 The Court is not persuaded.

True, the factual findings of the CTA are generally not disturbed on appeal when supported by
substantial evidence and in the absence of gross error or grave abuse of discretion. However, the CTA's
application of the law to the facts of this controversy is an altogether different matter, for it involves a
legal question. There is a question of law when the issue is the application of the law to a given set of
facts. On the other hand, a question of fact involves the truth or falsehood of alleged facts.9 In the
present case, the Court of Appeals ruled not on the truth or falsity of the facts found by the CTA, but on
the latter's application of the law on prescription.

Sec. 331 of the National Internal Revenue Code provides:

Sec. 331. Period of limitation upon assessment and collection. — Except as provided in
the succeeding section, internal-revenue taxes shall be assessed within five years after the
return was filed, and no proceeding in court without assessment for the collection of such
taxes shall be begun after expiration of such period. For the purposes of this section, a
return filed before the last day prescribed by law for the filing thereof shall be considered
as filed on such last day: Provided, That this limitation shall not apply to cases already
investigated prior to the approval of this Code.

Applying this provision of law to the facts at hand, it is clear that the October 16, 1980 and the March
1981 assessments were issued by the BIR beyond the five-year statute of limitations. The Court has
thoroughly studied the records of this case and found no basis to disregard the five-year period of
prescription. As succinctly pronounced by the Court of Appeals:

81
The subsequent assessment made by the respondent Commissioner on October 40, 1980,
modified by that of March 16, 1981, violates the law. Involved in this petition is the
income of the petitioner for the year 1974, the returns for which were required to be filed
on or before April 15 of the succeeding year. The returns for the year 1974 were duly
filed by the petitioner, and assessment of taxes due for such year — including that on the
transfer of properties on June 21, 1974 — was made on April 13, 1975 and
acknowledged by Letter of Confirmation No. 101155 terminating the examination on this
subject. The subsequent assessment of October 10, 1980 modified, by that of March 16,
1981, was made beyond the period expressly set in Section 331 of the National Internal
Revenue Code . . . . 10

Petitioner relies on the CTA ruling, the salient portion of which reads:

Falsity is what we have here, and for that matter, we hasten to add that the second
assessment (March 16, 1981) of the Commissioner was well-advised having been made
in contemplation of his power under Section 15 of the 1974 Code (now Section 16, of
NIRC) to assess the proper tax on the best evidence obtainable "when there is reason to
believe that a report of a taxpayer is false, incomplete or erroneous. More, when there is
falsity with intent to evade tax as in this case, the ordinary period of limitation upon
assessment and collection does not apply so that contrary to the averment of petitioner,
the right to assess respondent has not prescribed.

What is the considered falsity? The transfer through sale of the parcels of land in
Tumajubong, Lamitan, Basilan in favor of Siltown Realty for the sum of P500,000.00
only whereas said lands had been sworn to under Presidential Decree No. 76 (Dec. 6,
1972) as having a value of P2,683,467 (P2,475,467 + P207,700) (see Declaration of Real
Property form, p. 28, and p. 15, no. 5, BIR Record). 11

For the purpose of safeguarding taxpayers from any unreasonable examination, investigation or
assessment, our tax law provides a statute of limitations in the collection of taxes. Thus, the law on
prescription, being a remedial measure, should be liberally construed in order to afford such
protection. 12 As a corollary, the exceptions to the law on prescription should perforce be strictly
construed.

Sec. 15 of the NIRC, on the other hand, provides that "[w]hen a report required by law as a basis for
the assessment of any national internal revenue tax shall not be forthcoming within the time fixed by
law or regulation, or when there is reason to believe that any such report is false, incomplete, or
erroneous, the Commissioner of Internal Revenue shall assess the proper tax on the best evidence
obtainable." Clearly, Section 15 does not provide an exception to the statute of limitations on the
issuance of an assessment, by allowing the initial assessment to be made on the basis of the best
evidence available. Having made its initial assessment in the manner prescribed, the commissioner
could not have been authorized to issue, beyond the five-year prescriptive period, the second and the
third assessments under consideration before us.

82
Nor is petitioner's claim of falsity sufficient to take the questioned assessments out of the ambit of the
statute of limitations. The relevant part of then Section 332 of the NIRC, which enumerates the
exceptions to the period of prescription, provides:

Sec. 332. Exceptions as to period of limitation of assessment and collection of taxes. —


(a) In the case of a false or fraudulent return with intent to evade a tax or of a failure to
file a return, the tax may be assessed, or a proceeding in court for the collection of such
tax may be begun without assessment, at any time within ten years after the discovery of
the falsity, fraud, or omission: . . . .

Petitioner insists that private respondent committed "falsity" when it sold the property for a price lesser
than its declared fair market value. This fact alone did not constitute a false return which contains
wrong information due to mistake, carelessness or ignorance.13 It is possible that real property may be
sold for less than adequate consideration for a bona fide business purpose; in such event, the sale
remains an "arm's length" transaction. In the present case, the private respondent was compelled to sell
the property even at a price less than its market value, because it would have lost all ownership rights
over it upon the expiration of the parity amendment. In other words, private respondent was attempting
to minimize its losses. At the same time, it was able to lease the property for 25 years, renewable for
another 25. This can be regarded as another consideration on the price.

Furthermore, the fact that private respondent sold its real property for a price less than its declared fair
market value did not by itself justify a finding of false return. Indeed, private respondent declared the
sale in its 1974 return submitted to the BIR. 14 Within the five-year prescriptive period, the BIR could
have issued the questioned assessment, because the declared fair market value of said property was of
public record. This it did not do, however, during all those five years. Moreover, the BIR failed to
prove that respondent's 1974 return had been filed fraudulently. Equally significant was its failure to
prove respondent's intent to evade the payment of the correct amount of tax.

Ineludibly, the BIR failed to show that private respondent's 1974 return was filed fraudulently with
intent to evade the payment of the correct amount of tax. 15 Moreover, even though a donor's tax, which
is defined as "a tax on the privilege of transmitting one's property or property rights to another or others
without adequate and full valuable consideration," 16 is different from capital gains tax, a tax on the
gain from the sale of the taxpayer's property forming part of capital assets, 17 the tax return filed by
private respondent to report its income for the year 1974 was sufficient compliance with the legal
requirement to file a return. In other words, the fact that the sale transaction may have partly resulted in
a donation does not change the fact that private respondent already reported its income for 1974 by
filing an income tax return.

Since the BIR failed to demonstrate clearly that private respondent had filed a fraudulent return with
the intent to evade tax, or that it had failed to file a return at all, the period for assessments has
obviously prescribed. Such instances of negligence or oversight on the part of the BIR cannot prejudice
taxpayers, considering that the prescriptive period was precisely intended to give them peace of mind.

Based on the foregoing, a discussion of the validity and legality of the assailed assessments has become
moot and unnecessary.

83
WHEREFORE, the Petition for Review is DENIED and the assailed Decision of the Court of Appeals
is AFFIRMED. No costs. SO ORDERED.

DIGEST:

Facts: Private respondent BF Goodrich Philippines Inc. was an American corporation prior to July 3,
1974. As a condition for approving the manufacture of tires and other rubber products, private
respondent was required by the Central Bank to develop a rubber plantation. In compliance therewith,
private respondent bought from the government certain parcels of land in Tumajubong Basilan, in 1961
under the Public Land Act and the Parity Amendment to the 1935 constitution, and there developed a
rubber plantation.

On August 2, 1973, the Justice Secretary rendered an opinion that ownership rights of Americans over
Public agricultural lands, including the right to dispose or sell their real estate, would be lost upon
expiration on July 3, 1974 of the Parity Amendment. Thus, private respondent sold its Basilan land
holding to Siltown Realty Phil. Inc., (Siltown) for P500,000 on January 21, 1974. Under the terms of
the sale, Siltown would lease the property to private respondent for 25 years with an extension of 25
years at the option of private respondent.

Private respondent books of accounts were examined by BIR for purposes of determining its tax
liability for 1974. This examination resulted in the April 23, 1975 assessment of private respondent for
deficiency income tax which it duly paid. Siltown’s books of accounts were also examined, and on the
basis thereof, on October 10, 1980, the Collector of Internal Revenue assessed deficiency donor’s tax
of P1,020,850 in relation to said sale of the Basilan landholdings.

Private respondent contested this assessment on November 24, 1980. Another assessment dated March
16, 1981, increasing the amount demanded for the alleged deficiency donor’s tax, surcharge, interest
and compromise penalty and was received by private respondent on April 9, 1981. On appeal, CTA
upheld the assessment. On review, CA reversed the decision of the court finding that the assessment
was made beyond the 5-year prescriptive period in Section 331 of the Tax Code.

Issue: Whether or not petitioner’s right to assess has prescribed.

Held: Applying then Sec. 331, NIRC (now Sec. 203, 1997 NIRC which provides a 3-year prescriptive
period for making assessments), it is clean that the October 16, 1980 and March 16, 1981 assessments
were issued by the BIR beyond the 5-year statute of limitations. The court thoroughly studied the

84
records of this case and found no basis to disregard the 5-year period of prescription, expressly set
under Sec. 331 of the Tax Code, the law then in force.

For the purpose of safeguarding taxpayers from any unreasonable examination, investigation or
assessment, our tax law provides a statute of limitations in the collection of taxes. Thus, the law or
prescription, being a remedial measure, should be liberally construed in order to afford such protection.
As a corollary, the exceptions to the law on prescription should perforce be strictly construed.

G.R. No. L-22492 September 5, 1967

BASILAN ESTATES, INC., petitioner,


vs.
THE COMMISSIONER OF INTERNAL REVENUE and THE COURT OF TAX
APPEALS, respondents.

Felix A. Gulfin and Antonio S. Alano for petitioner.


Office of the Solicitor General for respondents.

BENGZON, J.P., J.:

A Philippine corporation engaged in the coconut industry, Basilan Estates, Inc., with principal offices
in Basilan City, filed on March 24, 1954 its income tax returns for 1953 and paid an income tax of
P8,028. On February 26, 1959, the Commissioner of Internal Revenue, per examiners' report of
February 19, 1959, assessed Basilan Estates, Inc., a deficiency income tax of P3,912 for 1953 and
P86,876.85 as 25% surtax on unreasonably accumulated profits as of 1953 pursuant to Section 25 of
the Tax Code. On non-payment of the assessed amount, a warrant of distraint and levy was issued but
the same was not executed because Basilan Estates, Inc. succeeded in getting the Deputy
Commissioner of Internal Revenue to order the Director of the district in Zamboanga City to hold
execution and maintain constructive embargo instead. Because of its refusal to waive the period of
prescription, the corporation's request for reinvestigation was not given due course, and on December
2, 1960, notice was served the corporation that the warrant of distraint and levy would be executed.

On December 20, 1960, Basilan Estates, Inc. filed before the Court of Tax Appeals a petition for
review of the Commissioner's assessment, alleging prescription of the period for assessment and
collection; error in disallowing claimed depreciations, travelling and miscellaneous expenses; and error
in finding the existence of unreasonably accumulated profits and the imposition of 25% surtax thereon.
On October 31, 1963, the Court of Tax Appeals found that there was no prescription and affirmed the
deficiency assessment in toto.

On February 21, 1964, the case was appealed to Us by the taxpayer, upon the following issues:

1. Has the Commissioner's right to collect deficiency income tax prescribed?


85
2. Was the disallowance of items claimed as deductible proper?

3. Have there been unreasonably accumulated profits? If so, should the 25% surtax be imposed on the
balance of the entire surplus from 1947-1953, or only for 1953?

4. Is the petitioner exempt from the penalty tax under Republic Act 1823 amending Section 25 of the
Tax Code?

PRESCRIPTION

There is no dispute that the assessment of the deficiency tax was made on February 26, 1959; but the
petitioner claims that it never received notice of such assessment or if it did, it received the notice
beyond the five-year prescriptive period. To show prescription, the annotation on the notice (Exhibit
10, No. 52, ACR, p. 54-A of the BIR records) "No accompanying letter 11/25/" is advanced as
indicative of the fact that receipt of the notice was after March 24, 1959, the last date of the five-year
period within which to assess deficiency tax, since the original returns were filed on March 24, 1954.

Although the evidence is not clear on this point, We cannot accept this interpretation of the petitioner,
considering the presence of circumstances that lead Us to presume regularity in the performance of
official functions. The notice of assessment shows the assessment to have been made on February 26,
1959, well within the five-year period. On the right side of the notice is also stamped "Feb. 26, 1959"
— denoting the date of release, according to Bureau of Internal Revenue practice. The Commissioner
himself in his letter (Exh. H, p. 84 of BIR records) answering petitioner's request to lift, the warrant of
distraint and levy, asserts that notice had been sent to petitioner. In the letter of the Regional Director
forwarding the case to the Chief of the Investigation Division which the latter received on March 10,
1959 (p. 71 of the BIR records), notice of assessment was said to have been sent to petitioner.
Subsequently, the Chief of the Investigation Division indorsed on March 18, 1959 (p. 24 of the BIR
records) the case to the Chief of the Law Division. There it was alleged that notice was already sent to
petitioner on February 26, 1959. These circumstances pointing to official performance of duty must
necessarily prevail over petitioner's contrary interpretation. Besides, even granting that notice had been
received by the petitioner late, as alleged, under Section 331 of the Tax Code requiring five years
within which to assess deficiency taxes, the assessment is deemed made when notice to this effect is
released, mailed or sent by the Collector to the taxpayer and it is not required that the notice be
received by the taxpayer within the aforementioned five-year period.1

ASSESSMENT

The questioned assessment is as follows:

Net Income per return P40,142.90


Add: Over-claimed
P10,500.49
depreciation
Mis. expenses
6,759.17
disallowed
Officer's
86
travelling 2,300.40 19,560.06
expenses
disallowed
Net Income per
P59,702.96
Investigation
20% tax on P59,702.96 11,940.00
Less: Tax already 8,028.00
assessed
Deficiency income tax P3,912.00
Add: Additional tax of
86,876.75
25% on P347,507.01

Tax Due & Collectible P90,788.75


=========

The Commissioner disallowed:

Over-claimed
P10,500.49
depreciation
Miscellaneous
6,759.17
expenses
Officer's travelling
2,300.40
expenses

DEDUCTIONS

A. Depreciation. — Basilan Estates, Inc. claimed deductions for the depreciation of its assets up to
1949 on the basis of their acquisition cost. As of January 1, 1950 it changed the depreciable value of
said assets by increasing it to conform with the increase in cost for their replacement. Accordingly,
from 1950 to 1953 it deducted from gross income the value of depreciation computed on the
reappraised value.

In 1953, the year involved in this case, taxpayer claimed the following depreciation deduction:

Reappraised assets P47,342.53


New assets consisting of hospital
building and equipment 3,910.45
Total depreciation
P51,252.98

Upon investigation and examination of taxpayer's books and papers, the Commissioner of Internal
Revenue found that the reappraised assets depreciated in 1953 were the same ones upon which
depreciation was claimed in 1952. And for the year 1952, the Commissioner had already determined,
87
with taxpayer's concurrence, the depreciation allowable on said assets to be P36,842.04, computed on
their acquisition cost at rates fixed by the taxpayer. Hence, the Commissioner pegged the deductible
depreciation for 1953 on the same old assets at P36,842.04 and disallowed the excess thereof in the
amount of P10,500.49.

The question for resolution therefore is whether depreciation shall be determined on the acquisition
cost or on the reappraised value of the assets.

Depreciation is the gradual diminution in the useful value of tangible property resulting from wear and
tear and normal obsolescense. The term is also applied to amortization of the value of intangible assets,
the use of which in the trade or business is definitely limited in duration.2 Depreciation commences
with the acquisition of the property and its owner is not bound to see his property gradually waste,
without making provision out of earnings for its replacement. It is entitled to see that from earnings the
value of the property invested is kept unimpaired, so that at the end of any given term of years, the
original investment remains as it was in the beginning. It is not only the right of a company to make
such a provision, but it is its duty to its bond and stockholders, and, in the case of a public service
corporation, at least, its plain duty to the public.3 Accordingly, the law permits the taxpayer to recover
gradually his capital investment in wasting assets free from income tax.4 Precisely, Section 30 (f) (1)
which states:

(1)In general. — A reasonable allowance for deterioration of property arising out of its use or
employment in the business or trade, or out of its not being used: Provided, That when the
allowance authorized under this subsection shall equal the capital invested by the taxpayer . . .
no further allowance shall be made. . . .

allows a deduction from gross income for depreciation but limits the recovery to the capital invested in
the asset being depreciated.

The income tax law does not authorize the depreciation of an asset beyond its acquisition cost. Hence, a
deduction over and above such cost cannot be claimed and allowed. The reason is that deductions from
gross income are privileges,5 not matters of right.6 They are not created by implication but upon clear
expression in the law.7

Moreover, the recovery, free of income tax, of an amount more than the invested capital in an asset will
transgress the underlying purpose of a depreciation allowance. For then what the taxpayer would
recover will be, not only the acquisition cost, but also some profit. Recovery in due time thru
depreciation of investment made is the philosophy behind depreciation allowance; the idea of profit on
the investment made has never been the underlying reason for the allowance of a deduction for
depreciation.

Accordingly, the claim for depreciation beyond P36,842.04 or in the amount of P10,500.49 has no
justification in the law. The determination, therefore, of the Commissioner of Internal Revenue
disallowing said amount, affirmed by the Court of Tax Appeals, is sustained.

B. Expenses. — The next item involves disallowed expenses incurred in 1953, broken as follows:

88
Miscellaneous
P6,759.17
expenses
Officer's travelling
2,300.40
expenses
Total
P9,059.57

These were disallowed on the ground that the nature of these expenses could not be satisfactorily
explained nor could the same be supported by appropriate papers.

Felix Gulfin, petitioner's accountant, explained the P6,759.17 was actual expenses credited to the
account of the president of the corporation incurred in the interest of the corporation during the
president's trip to Manila (pp. 33-34 of TSN of Dec. 5, 1962); he stated that the P2,300.40 was the
president's travelling expenses to and from Manila as to the vouchers and receipts of these, he said the
same were made but got burned during the Basilan fire on March 30, 1962 (p. 40 of same TSN).
Petitioner further argues that when it sent its records to Manila in February, 1959, the papers in support
of these miscellaneous and travelling expenses were not included for the reason that by February 9,
1959, when the Bureau of Internal Revenue decided to investigate, petitioner had no more obligation to
keep the same since five years had lapsed from the time these expenses were incurred (p. 41 of same
TSN). On this ground, the petitioner may be sustained, for under Section 337 of the Tax Code, receipts
and papers supporting such expenses need be kept by the taxpayer for a period of five years from the
last entry. At the time of the investigation, said five years had lapsed. Taxpayer's stand on this issue is
therefore sustained.

UNREASONABLY ACCUMULATED PROFITS

Section 25 of the Tax Code which imposes a surtax on profits unreasonably accumulated, provides:

Sec. 25. Additional tax on corporations improperly accumulating profits or surplus — (a)
Imposition of tax. — If any corporation, except banks, insurance companies, or personal holding
companies, whether domestic or foreign, is formed or availed of for the purpose of preventing
the imposition of the tax upon its shareholders or members or the shareholders or members of
another corporation, through the medium of permitting its gains and profits to accumulate
instead of being divided or distributed, there is levied and assessed against such corporation, for
each taxable year, a tax equal to twenty-five per centum of the undistributed portion of its
accumulated profits or surplus which shall be in addition to the tax imposed by section twenty-
four, and shall be computed, collected and paid in the same manner and subject to the same
provisions of law, including penalties, as that tax.1awphîl.nèt

The Commissioner found that in violation of the abovequoted section, petitioner had unreasonably
accumulated profits as of 1953 in the amount of P347,507.01, based on the following circumstances
(Examiner's Report pp. 62-68 of BIR records):

1. Strong financial position of the petitioner as of December 31, 1953. Assets were P388,617.00
while the liabilities amounted to only P61,117.31 or a ratio of 6:1.
89
2. As of 1953, the corporation had considerable capital adequate to meet the reasonable needs of
the business amounting to P327,499.69 (assets less liabilities).

3. The P200,000 reserved for electrification of drier and mechanization and the P50,000
reserved for malaria control were reverted to its surplus in 1953.

4. Withdrawal by shareholders, of large sums of money as personal loans.

5. Investment of undistributed earnings in assets having no proximate connection with the


business — as hospital building and equipment worth P59,794.72.

6. In 1953, with an increase of surplus amounting to P677,232.01, the capital stock was
increased to P500,000 although there was no need for such increase.

Petitioner tried to show that in considering the surplus, the examiner did not take into account the
possible expenses for cultivation, labor, fertilitation, drainage, irrigation, repair, etc. (pp. 235-237 of
TSN of Dec. 7, 1962). As aptly answered by the examiner himself, however, they were already
included as part of the working capital (pp. 237-238 of TSN of Dec. 7, 1962).

In the unreasonable accumulation of P347,507.01 are included P200,000 for electrification of driers
and mechanization and P50,000 for malaria control which were reserved way back in 1948 (p. 67 of the
BIR records) but reverted to the general fund only in 1953. If there were any plans for these amounts to
be used in further expansion through projects, it did not appear in the records as was properly indicated
in 1948 when such amounts were reserved. Thus, while in 1948 it was already clear that the money was
intended to go to future projects, in 1953 upon reversion to the general fund, no such intention was
shown. Such reversion therefore gave occasion for the Government to consider the same for tax
purposes. The P250,000 reverted to the general fund was sought to be explained as later used
elsewhere: "part of it in the Hilano Industries, Inc. in building the factory site and buildings to house
technical men . . . part of it was spent in the facilities for the waterworks system and for
industrialization of the coconut industry" (p. 117 of TSN of Dec. 6, 1962). This is not sufficient
explanation. Persuasive jurisprudence on the matter such as those in the United States from where our
tax law was derived,8 has it that: "In order to determine whether profits were accumulated for the
reasonable needs of the business or to avoid the surtax upon shareholders, the controlling intention of
the taxpayer is that which is manifested at the time of the accumulation, not subsequently declared
intentions which are merely the products of after-thought."9 The reversion here was made because the
reserved amount was not enough for the projects intended, without any intent to channel the same to
some particular future projects in mind.

Petitioner argues that since it has P560,717.44 as its expenses for the year 1953, a surplus of
P347,507.01 is not unreasonably accumulated. As rightly contended by the Government, there is no
need to have such a large amount at the beginning of the following year because during the year,
current assets are converted into cash and with the income realized from the business as the year goes,
these expenses may well be taken care of (pp. 238 of TSN of Dec. 7, 1962). Thus, it is erroneous to say
that the taxpayer is entitled to retain enough liquid net assets in amounts approximately equal to current
operating needs for the year to cover "cost of goods sold and operating expenses" for "it excludes

90
proper consideration of funds generated by the collection of notes receivable as trade accounts during
the course of the year."10 In fact, just because the fatal accumulations are less than 70% of the annual
operating expenses of the year, it does not mean that the accumulations are reasonable as a matter of
law."11

Petitioner tried to show that investments were made with Basilan Coconut Producers Cooperative
Association and Basilan Hospital (pp. 103-105 of TSN of Dec. 6, 1962) totalling P59,794.72 as of
December 31, 1953. This shows all the more the unreasonable accumulation. As of December 31, 1953
already P59,794.72 was spent — yet as of that date there was still a surplus of P347,507.01.

Petitioner questions why the examiner covered the period from 1948-1953 when the taxable year on
review was 1953. The surplus of P347,507.01 was taken by the examiner from the balance sheet of
petitioner for 1953. To check the figure arrived at, the examiner traced the accumulation process from
1947 until 1953, and petitioner's figure stood out to be correct. There was no error in the process
applied, for previous accumulations should be considered in determining unreasonable accumulations
for the year concerned. "In determining whether accumulations of earnings or profits in a particular
year are within the reasonable needs of a corporation, it is neccessary to take into account prior
accumulations, since accumulations prior to the year involved may have been sufficient to cover the
business needs and additional accumulations during the year involved would not reasonably be
necessary."12

Another factor that stands out to show unreasonable accumulation is the fact that large amounts were
withdrawn by or advanced to the stockholders. For the year 1953 alone these totalled P197,229.26. Yet
the surplus of P347,507.01 was left as of December 31, 1953. We find unacceptable petitioner's
explanation that these were advances made in furtherance of the business purposes of the petitioner. As
correctly held by the Court of Tax Appeals, while certain expenses of the corporation were credited
against these amounts, the unspent balance was retained by the stockholders without refunding them to
petitioner at the end of each year. These advances were in fact indirect loans to the stockholders
indicating the unreasonable accumulation of surplus beyond the needs of the business.

ALLEGED EXEMPTION

Petitioner wishes to avail of the exempting proviso in Sec. 25 of the Internal Revenue Code as
amended by R.A. 1823, approved June 22, 1957, whereby accumulated profits or surplus if invested in
any dollar-producing or dollar-earning industry or in the purchase of bonds issued by the Central Bank,
may not be subject to the 25% surtax. We have but to point out that the unreasonable accumulation was
in 1953. The exemption was by virtue of Republic Act 1823 which amended Sec. 25 only on June 22,
1957 — more than three years after the period covered by the assessment.

In resume, Basilan Estates, Inc. is liable for the payment of deficiency income tax and surtax for the
year 1953 in the amount of P88,977.42, computed as follows:

Net Income per


P40,142.90
return
Add: Over- 10,500.49
91
claimed
depreciation
Net income per
P50,643.39
finding
20% tax on
P10,128.67
P50,643.39
Less: Tax
8,028.00
already assessed
Deficiency
income tax P2,100.67
Add: 25%
surtax on 86,876.75
P347,507.01
Total tax due and
P88,977.42
collectible
===========

WHEREFORE, the judgment appealed from is modified to the extent that petitioner is allowed its
deductions for travelling and miscellaneous expenses, but affirmed insofar as the petitioner is liable for
P2,100.67 as deficiency income tax for 1953 and P86,876.75 as 25% surtax on the unreasonably
accumulated profit of P347,507.01. No costs. So ordered.

DIGEST:

Doctrine:
The income tax law does not authorize the depreciation of an asset beyond its acquisition cost. Hence,
a deduction over and above such cost cannot be claimed and allowed. The reason is that deductions
from gross income are privileges, not matters of right. They are not created by implication but upon
clear expression in the law.

Facts:
Basilan Estates, Inc. claimed deductions for the depreciation of its assets on the basis of their
acquisition cost. As of January 1, 1950 it changed the depreciable value of said assets by increasing it
to conform with the increase in cost for their replacement. Accordingly, from 1950 to 1953 it deducted
from gross income the value of depreciation computed on the reappraised value.

CIR disallowed the deductions claimed by petitioner, consequently assessing the latter of deficiency
income taxes.

92
Issue:
Whether or not the depreciation shall be determined on the acquisition cost rather than the reappraised
value of the assets

Held:
Yes. The following tax law provision allows a deduction from gross income for depreciation but limits
the recovery to the capital invested in the asset being depreciated:

(1)In general. — A reasonable allowance for deterioration of property arising out of its use or
employment in the business or trade, or out of its not being used: Provided, That when the allowance
authorized under this subsection shall equal the capital invested by the taxpayer . . . no further
allowance shall be made. . . .

The income tax law does not authorize the depreciation of an asset beyond its acquisition cost. Hence, a
deduction over and above such cost cannot be claimed and allowed. The reason is that deductions from
gross income are privileges, not matters of right. They are not created by implication but upon clear
expression in the law [Gutierrez v. Collector of Internal Revenue, L-19537, May 20, 1965].

Depreciation is the gradual diminution in the useful value of tangible property resulting from wear and
tear and normal obsolescense. It commences with the acquisition of the property and its owner is not
bound to see his property gradually waste, without making provision out of earnings for its
replacement.

The recovery, free of income tax, of an amount more than the invested capital in an asset will
transgress the underlying purpose of a depreciation allowance. For then what the taxpayer would
recover will be, not only the acquisition cost, but also some profit. Recovery in due time thru
depreciation of investment made is the philosophy behind depreciation allowance; the idea of profit on
the investment made has never been the underlying reason for the allowance of a deduction for
depreciation.

G.R. No. 127777 October 1, 1999

PETRONILA C. TUPAZ, petitioner,


vs.
HONORABLE BENEDICTO B. ULEP Presiding Judge of RTC Quezon City, Branch 105, and
PEOPLE OF THE PHILIPPINES, respondents.

93
PARDO, J.:

The case before us is a special civil action for certiorari with application for temporary restraining
order seeking to enjoin respondent Judge Benedicto B. Ulep of the Regional Trial Court, Quezon City,
Branch 105, from trying Criminal Case No. Q-91-17321, and to nullify respondent judge's order
reviving the information therein against petitioner, for violation of the Tax Code, as the offense
charged has prescribed or would expose petitioner to double jeopardy.1âwphi1.nêt

The facts are as follows:

On June 8, 1990, State Prosecutor (SP) Esteban A. Molon, Jr. filed with the Metropolitan Trial Court
(MeTC), Quezon City, Branch 33, an information against accused Petronila C. Tupaz and her late
husband Jose J. Tupaz, Jr., as corporate officers of El Oro Engravers Corporation, for nonpayment of
deficiency corporate income tax for the year 1979, amounting to P2,369,085.46, in violation of Section
51 (b) in relation to Section 73 of the Tax Code of
1977. 1 On September 11, 1990, the MeTC dismissed the information for lack of jurisdiction. On
November 16, 1990, the trial court denied the prosecution's motion for reconsideration.

On January 10, 1991, SP Molon filed with the Regional Trial Court, Quezon City, two (2)
informations, docketed as Criminal Case Nos. Q-91-17321 2 and Q-9l-17322, 3 against accused and her
late husband, for the same alleged nonpayment of deficiency corporate income tax for the year 1979.
Criminal Case No. Q-91-17321 was raffled to Branch 105, 4 presided over by respondent Judge
Benedicto B. Ulep; Q-91-17322 was raffled to Branch 86, then presided over by Judge Antonio P.
Solano. The identical informations read as follows:

That in Quezon City, Metro Manila and within the jurisdiction of this Honorable Court
and upon verification and audit conducted by the Bureau of Internal Revenue on the 1979
corporate annual income tax return and financial statements of El Oro Engravers Corp.,
with office address at 809 Epifanio delos Santos Avenue, Quezon City, Metro Manila, it
was ascertained that said corporation was found liable to pay the amount of
P2,369,085.46, as deficiency corporate income tax for the year 1979 and that, despite
demand of the payment of the aforesaid deficiency tax by the Bureau of Internal Revenue
and received by said corporation, which demand has already become final, said El Oro
Engravers Corp., through above-named accused, the responsible corporate-officers of
said corporation, failed and refused, despite repeated demands, and still fail and refuse to
pay said tax liability.

CONTRARY TO LAW. 5

On September 25, 1991, both accused posted bail bond in the sum of P1,000.00 each, for their
provisional liberty.

On November 6, 1991, accused filed with the Regional Trial Court, Quezon City, Branch 86, a motion
to dismiss/quash 6 information (Q-91-17322) for the reason that it was exactly the same as the

94
information against the accused pending before RTC, Quezon City, Branch 105 (Q-91-17321).
However, on November 11, 1991, Judge Solano denied the motion. 7

In the meantime, on July 25, 1993, Jose J. Tupaz, Jr. died in Quezon City.

Subsequently, accused Petronila C. Tupaz filed with the Regional Trial Court, Quezon City, Branch
105, a petition for reinvestigation, which Judge Ulep granted in an order dated August 30, 1994. 8

On September 5, 1994, Senior State Prosecutor Bernelito R. Fernandez stated that no new issues were
raised in the request for reinvestigation, and no cogent reasons existed to alter, modify or reverse the
findings of the investigating prosecutor. He considered the reinvestigation as terminated, and
recommended the prompt arraignment and trial of the accused. 9

On September 20, 1994, the trial court (Branch No. 105) arraigned accused Petronila C. Tupaz in
Criminal Case No. Q-91-17321, and she pleaded not guilty to the information therein.

On October 17, 1994, the prosecution filed with the Regional Trial Court, Quezon City, Branch 105, a
motion for leave to file amended information in Criminal Case No. Q91-17321 to allege expressly the
date of the commission of the offense, to wit: on or about August 1984 or subsequently thereafter.
Despite opposition of the accused, on March 2, 1995, the trial court granted the motion and admitted
the amended information. 10 Petitioner was not re-arraigned on the amended information. However, the
amendment was only on a matter of form. 11 Hence, there was no need to re-arraign the accused. 12

On December 5, 1995, accused filed with the Regional Trial Court, Quezon City, Branch 105, a motion
for leave to file and admit motion for reinvestigation. The trial court granted the motion in its order
dated December 13, 1995.

Prior to this, on October 18, 1995, Judge Ulep issued an order directing the prosecution to withdraw the
information in Criminal Case No. Q-91-17322, pending before Regional Trial Court, Quezon City,
Branch 86, after discovering that said information was identical to the one filed with Regional Trial
Court, Quezon City, Branch 105. On April 16, 1996, State Prosecutor Alfredo P. Agcaoili filed with
the trial court a motion to withdraw information in Criminal Case No. Q-91-17321. Prosecutor
Agcaoili thought that accused was charged in Criminal Case No. Q-91-17321, for nonpayment of
deficiency contractor's tax, but found that accused was exempted from paying said tax.

On May 15, 1996, Prosecutor Agcaoili filed with the Regional Trial Court, Quezon City, Branch 86, a
motion for consolidation of Criminal Case No. Q-91-17322 with Criminal Case No. Q-91-17321
pending before the Regional Trial Court, Quezon City, Branch 105. On the same date, the
court 13 granted the motion for consolidation.

On May 20, 1996, Judge Ulep of Regional Trial Court, Quezon City, Branch 105, granted the motion
for withdrawal of the information in Criminal Case No. Q-91-17321 and dismissed the case, as prayed
for by the prosecution.

On May 28, 1996, Prosecutor Agcaoili filed with the Regional Trial Court, Quezon City, Branch 105, a
motion to reinstate information in Criminal Case Q-91-17321, 14 stating that the motion to withdraw
95
information was made through palpable mistake, and was the result of excusable neglect. He thought
that Criminal Case No. Q-91-17321 was identical to Criminal Case No. Q-90-12896, wherein accused
was charged with nonpayment of deficiency contractor's tax, amounting to P346,879.29.

Over the objections of accused, on August 6, 1996, the Regional Trial Court, Quezon City, Branch 105,
granted the motion and ordered information in Criminal Case No. Q-91-17321 reinstated. 15 On
September 24, 1996, accused filed with the trial court a motion for reconsideration. On December 4,
1996, the trial court denied the motion.

Hence, this petition.

On July 9, 1997, we required respondents to comment on the petition within ten (10) days from notice.
On October 10, 1997, the Solicitor General filed his comment. 16

On October 26, 1998, the Court resolved to give due course to the petition and required the parties to
file their respective memoranda within twenty (20) days from notice. The parties have complied.

Petitioner submits that respondent judge committed a grave abuse of discretion in reinstating the
information in Criminal Case No. Q-91-17321 because (a) the offense has prescribed; or (b) it exposes
her to double jeopardy.

As regards the issue of prescription, petitioner contends that: (a) the period of assessment has
prescribed, applying the three (3) year period provided under Batas Pambansa No. 700; (b) the offense
has prescribed since the complaint for preliminary investigation was filed with the Department of
Justice only on June 8, 1989, and the offense was committed in April 1980 when she filed the income
tax return covering taxable year 1979.

Petitioner was charged with nonpayment of deficiency corporate income tax for the year 1979, which
tax return was filed in April 1980. On July 16, 1984, the Bureau of Internal Revenue (BIR) issued a
notice of assessment. Petitioner contends that the July 16, 1984 assessment was made out of time.

Petitioner avers that while Sections 318 and 319 of the NIRC of 1977 provide a five (5) year period of
limitation for the assessment and collection of internal revenue taxes, Batas Pambansa Blg. 700,
enacted on February 22, 1984, amended the two sections and reduced the period to three (3) years. As
provided under B.P. Blg. 700, the BIR has three (3) years to assess the tax liability, counted from the
last day of filing the return, or from the date the return is filed, whichever comes later. Since the tax
return was filed in April 1980, the assessment made on July 16, 1984 was beyond the three (3) year
prescriptive period.

Petitioner submits that B.P. Blg. 700 must be given retroactive effect since it is favorable to the
accused. Petitioner argues that Article 22 of the Revised Penal Code, regarding the allowance of
retroactive application of penal laws when favorable to the accused shall apply in this case.

The Solicitor General, in his comment, maintains that the prescriptive period for assessment and
collection of petitioner's deficiency corporate income tax was five (5) years. The Solicitor General
asserts that the shortened period of three (3) years provided under B.P. Blg. 700 applies to assessments
96
and collections of internal revenue taxes beginning taxable year 1984. Since the deficiency corporate
income tax was for taxable year 1979, then petitioner was still covered by the five (5) year period.
Thus, the July 16, 1984 tax assessment was made within the prescribed period.1âwphi1.nêt

At the outset, it must be stressed that "internal revenue taxes are self-assessing and no further
assessment by the government is required to create the tax liability. An assessment, however, is not
altogether inconsequential; it is relevant in the proper pursuit of judicial and extra judicial remedies to
enforce taxpayer liabilities and certain matters that relate to it, such as the imposition of surcharges and
interest, and in the application of statues of limitations and in the establishment of tax liens." 17

An assessment contains not only a computation of tax liabilities, but also a demand for payment within
a prescribed period. The ultimate purpose of assessment is to ascertain the amount that each taxpayer is
to pay. 18 An assessment is a notice to the effect that the amount therein stated is due as tax and a
demand for payment thereof. 19 Assessments made beyond the prescribed period would not be binding
on the taxpayer. 20

We agree with the Solicitor General that the shortened period of three (3) years prescribed under B.P.
Blg. 700 is not applicable to petitioner. B.P. Blg. 700, effective April 5, 1984, specifically states that
the shortened period of three years shall apply to assessments and collections of internal revenue taxes
beginning taxable year 1984. Assessments made on or after April 5, 1984 are governed by the five-year
period if the taxes assessed cover taxable years prior to January 1,
1984. 21 The deficiency income tax under consideration is for taxable year 1979. Thus, the period of
assessment is still five (5) years, under the old law. The income tax return was filed in April 1980.
Hence, the July 16, 1984 tax assessment was issued within the prescribed period of five (5) years, from
the last day of filing the return, or from the date the return is filed, whichever comes later.

Art. 22 of the Revised Penal Code finds no application in this case for the simple reason that the
provisions on the period of assessment can not be considered as penal in nature.

Petitioner also asserts that the offense has prescribed. Petitioner invokes Section 340 (now 281 of 1997
NIRC) of the Tax Code which provides that violations of any provision of the Code prescribe in five
(5) years. Petitioner asserts that in this case, it began to run in 1979, when she failed to pay the correct
corporate tax due during that taxable year. Hence, when the BIR instituted criminal proceedings on
June 8, 1989, by filing a complaint for violation of the Tax Code with the Department of Justice for
preliminary investigation it was beyond the prescriptive period of five (5) years. At most, the BIR had
until 1984 to institute criminal proceedings.

On the other hand, the Solicitor General avers that the information for violation of the Tax Code was
filed within the prescriptive period of five (5) years provided in Section 340 (now 281 in 1997 NIRC)
of the Code. It is only when the assessment has become final and unappealable that the five (5) year
period commences to run. A notice of assessment was issued on July 16, 1984. When petitioner failed
to question or protest the deficiency assessment thirty (30) days therefrom, or on August 16, 1984, it
became final and unappealable. Consequently, it was from this period that the prescriptive period of
five (5) years commenced. Thus, the complaint filed with the Department of Justice on June 8, 1989
was within the prescribed period.

97
We agree with the Solicitor General that the offense has not prescribed. Petitioner was charged with
failure to pay deficiency income tax after repeated demands by the taxing authority. In Lim,
Sr. v. Court of Appeals, 22 we stated that by its nature the violation could only be committed after
service of notice and demand for payment of the deficiency taxes upon the taxpayer. Hence, it cannot
be said that the offense has been committed as early as 1980, upon filing of the income tax return. This
is so because prior to the finality of the assessment, the taxpayer has not committed any violation for
nonpayment of the tax. The offense was committed only after the finality of the assessment coupled
with taxpayer's willful refusal to pay the taxes within the allotted period. In this case, when the notice
of assessment was issued on July 16, 1984, the taxpayer still had thirty (30) days from receipt thereof
to protest or question the assessment. Otherwise, the assessment would become final and
unappealable. 23 As he did not protest, the assessment became final and unappealable on August 16,
1984. Consequently, when the complaint for preliminary investigation was filed with the Department
of Justice on June 8, 1989, the criminal action was instituted within the five (5) year prescriptive
period.

Petitioner contends that by reinstating the information, the trial court exposed her to double jeopardy.
Neither the prosecution nor the trial court obtained her permission before the case was dismissed. She
was placed in jeopardy for the first time after she pleaded to a valid complaint filed before a competent
court and the case was dismissed without her express consent. When the trial court reinstated the
information charging the same offense, it placed her in double jeopardy.

Petitioner also asserts that the trial court gravely erred when, over her objections, it admitted the
amended information. She submits that the amendment is substantial in nature, and would place her in
double jeopardy.

On the other hand, the Solicitor General contends that reinstating the information does not violate
petitioner's right against double jeopardy. He asserts that petitioner induced the dismissal of the
complaint when she sought the reinvestigation of her tax liabilities. By such inducement, petitioner
waived or was estopped from claiming her right against double jeopardy.

The Solicitor General further contends that, assuming arguendo that the case was dismissed without
petitioner's consent, there was no valid dismissal of the case since Prosecutor Agcaoili was under a
mistaken assumption that it was a charge of nonpayment of contractor's tax.

We sustain petitioner's contention. The reinstatement of the information would expose her to double
jeopardy. An accused is placed in double jeopardy if he is again tried for an offense for which he has
been convicted, acquitted or in another manner in which the indictment against him was dismissed
without his consent. In the instant case, there was a valid complaint filed against petitioner to which she
pleaded not guilty. The court dismissed the case at the instance of the prosecution, without asking for
accused-petitioner's consent. This consent cannot be implied or presumed. 24 Such consent must be
expressed as to have no doubt as to the accused's conformity. 25 As petitioner's consent was not
expressly given, the dismissal of the case must be regarded as final and with prejudice to the re-filing
of the case. 26 Consequently, the trial court committed grave abuse of discretion in reinstating the
information against petitioner in violation of her constitutionally protected right against double
jeopardy.
98
WHEREFORE, we GRANT the petition. We enjoin the lower court, the Regional Trial Court of
Quezon City, Branch 105, from trying Criminal Case No. Q-91-17321 and order its dismissal. Costs de
oficio.SO ORDERED.

DIGEST:

Facts: June 8, 1990, an information against accused Petronila C. Tupaz and her late husband Jose J.
Tupaz, Jr., as corporate officers of El Oro Engravers Corp., was field for non-payment of deficiency
corporate in come tax for the year 1979 in violation of Sec. 51(b) in relation to Sec. 73 of the 1977 Tax
Code. The information was dismissed for the lack of jurisdiction by the MeTC of Q.C.

January 10, 1991, 2 information were filed before the RTC of Q.C. against spouses for the same
alleged non-payment of deficiency corporate income tax for the year 1979.

Prior to this, petitioner was charged with nonpayment of deficiency corporate income tax for the year
1979, which tax return was filed in April 1980. On July 16, 1984, the BIR issued a notice of
assessment. Petitioner contends that the July 16, 1984 assessment was made out of time.

Petitioner avers that while Sec. 318 and 319 of the 1977 NIRC provide a 5-year period of limitation for
the assessment and collection of internal revenue taxes, BP700, enacted on February 22, 1984,
amended the 2 sections and reduced the period to 3 years to assess the tax liability, counted from the
last day of filing the return or from the date the return is filed, whichever comes later. Since the tax
return was filed in April 1980, the assessment made on July 16, 1984 was beyond the 3-year
prescriptive period.

Issue: Whether the government’s right to assess has prescribed.

Held: The shortened period of 3 years prescribed under BP700 is not applicable to petitioner. BP700,
effective April 5, 1984, specifically states that the shortened period of 3 years shall apply to
assessments and collections of internal revenue taxes beginning taxable year 1984. Assessments made
on or after April 5, 1984 are governed by the 5-year period if the taxes assessed cover taxable years
prior to January 1, 1984. The deficiency income tax under consideration is for taxable year 1979. Thus,
the period of assessment is still 5 years, under the old law. The income tax return was filed in April
1980. Hence, the July 16, 1984 tax assessment was issued within the prescribed period of 5 years, from
the last day of filing the return, or from the date the return is filed, whichever comes later.

At the outset, it must be stressed that “internal revenue taxes are self-assessing and no further
assessment by the government is required to create the tax liability. An assessment, however, is not
99
altogether inconsequential; it is relevant in the proper pursuit of judicial and extrajudicial remedies to
enforce taxpayer liabilities and certain matters that relate to it, such as the imposition of surcharges and
interest, and in the application of statute of limitations and in the establishment with tax liens.”

G.R. No. L-19470 January 30, 1965

GONZALO P. NAVA, petitioner,


vs.
COMMISSIONER OF INTERNAL REVENUE, respondent.

E. P. Villar and A. Tordesillas for petitioner.


Office of the Solicitor General for respondent.

REYES, J.B.L., J.:

Gonzalo P. Nava prosecuted this appeal against a decision dated September 25, 1961 by the Court of
Tax Appeals (C.T.A. Case No. 568) holding him liable in the amount of P3,052.00 as deficiency
income tax for the year 1950 as well as from its order dated February 10, 1962 denying a motion to
reconsider said decision.

The undisputed facts are: that on May 15, 1951, Nava filed his income tax return for the year 1950,
and, on the same date, he was assessed by respondent Commissioner (formerly Collector) of Internal
Revenue in the sum of P4,952.00, based solely on said return. Nava paid one-half of the tax due,
leaving a balance of P2,491.00. Subsequently, Nava offered his backpay certificate to pay said balance,
but respondent refused the offer. On July 28, 1953, he requested the respondent to hold in abeyance the
collection of said balance until the question of whether or not he was entitled to pay the same out of his
backpay shall have been decided, but this was also rejected by the latter in a reply letter dated January
5, 1954. This rejection was followed by two more letters or notices demanding payment of the balance
thereof, the last of which was dated February 22, 1955.

On March 30, 1955, after investigation of petitioner's 1950 income tax return, respondent Collector
issued a deficiency income tax assessment notice (Exhibit "4") requiring petitioner to pay not later than
April 30, 1955 the sum of P9,124.50, that included the balance of P2,491.00, still unpaid under the
original assessment, plus a 50% surcharge. Several notices of this revised assessment are alleged to
have been issued to the taxpayer, but Nava claims to have learned of it for the first time on December
19, 1956, more than five years since the original tax return was filed, and testified to that effect in the
court below. In a letter of January 10, 1957, Nava called attention to the fact that more than six years
had elapsed, protested the assessment, and contended that it was a closed issue. The Director insisted
upon his demand that the new assessment be paid (registered letter of Mach 25, 1957, Exhibit "5").
Nava asked for reconsideration, and on June 16, 1958 was informed that reinvestigation would be
granted provided the taxpayer waived the statute of limitations (Exh. "7"), a condition that was rejected
(Exh. "8"). Thereupon, the reconsideration of the assessment was denied by the Collector's letter of
July 22, 1958 (Exh. "9"), and on August 8, 1958 Nava filed a petition for review with the Court of Tax
100
Appeals. The latter reduced the deficiency to P3,052.00, and cancelled the 50% surcharge. The
petitioner appealed to this Court.

The principal issue in this appeal is whether the enforcement of the tax assessment has prescribed. The
Court of Tax Appeals ruled that it had not, stating that:

The duplicate copy of the income tax assessment notice indicates that it was issued on March
30, 1955 (Exh. 4, page 7, B.I.R. records). "Call-up" letters were sent to petitioner reminding him
of the obligation. These call-up letters or notices were recorded in Exh. C for petitioner (Exh. 3
for respondent, page 6, B.I.R. records), to wit:

1st notice 4/10/56


2nd notice 7/3/56
Final 9/25/56

In addition to the written notice sent to petitioner, he was also personally interviewed. A report
on these written notices and personal interviews appears in the memorandum of an agent of the
Bureau of Internal Revenue dated December 10, 1956, the pertinent portion of which reads as
follows.

"Several call-up letters and repeated demands have been made to subject taxpayers but in
spite of the considerable length of time that has elapsed the above accounts still remain
unsettled. The warrant assemblies of the above-stated tax cases were assigned to Agent
A. H. Aguilar and an interview with Mr. G. P. Nava revealed that the latter refused to pay
alleging that these cases come within the purview of the Avelino case, hence, the
B.I.R. has no more right to collect from him." (Exh. D, page 8, B.I.R. records).

Petitioner's claim that he came to know of the assessment only on or about December 19, 1956
can not be given much credence. We are inclined to believe that the assessment notice dated
March 30, 1955 and the several call-up letters sent to him were received by him in due course of
mail but that he ignored them because of his belief that the right of the Government to collect
the tax had prescribed in view of the decision in the Avelino case. This conclusion finds support
in a note sent or delivered by petitioner to an employee of the Bureau who interviewed him,
wherein he stated:

"This is to certify that I have received today, second final notice from the Bureau of
Internal Revenue delivered by Mrs. Canlas. My reply to your said final notice, as per
your request, will be sent to you on or before January 3, 1957, in view of the fact that I
may not be able to contact right away my Accountant." (Exh. E, page 9, B.I.R. records;
Emphasis ours.)

The fact that petitioner admitted receipt of the "second final notice" without protest is an
indication that he received the previous notices

101
Assuming that petitioner received the income tax assessment notice dated March 30, 1955 in
due course of mail, that is, not later than April 10, 1955, the assessment was made within the
five-year period since he filed his income tax return on May 15, 1951, even granting that the
ten-year period applicable to fraud cases does not apply to this case. (The assessment includes
the fraud penalty.) Since the deficiency income tax was assessed on or about April 10, 1955, the
Government is authorized to collect the same by distraint or levy or by judicial action within
five years from that date, or not later than April 10, 1960. Judicial action was instituted in the
Court of First Instance of Manila in Civil Case No. 32796 for collection of said amount,
followed by the institution of the instant appeal in this Court by petitioner himself on August 8,
1958, both within the five-year period. Therefore, we are of the opinion that the right of the
Government to assess and collect said deficiency income tax has not prescribed." (Annex "O",
petition, pp. 134-137, records).

It is to be noted that in its decision the Court of Tax Appeals relied mainly on the duplicate copy of the
deficiency income tax notice found in the Bureau of Internal Revenue file of petitioner Nava (Exhibit
"4", page 7, B.I.R. records). On the corresponding blank space for the date of issue of said duplicate
copy was typed "3/30/55". Petitioner Nava denied having received the original copy of said notice. The
Revenue Commissioner, on the other hand, presented a witness (Mr. Pablo Sangil, an employee [clerk]
of the B.I.R.) who attempted to establish that the original copy thereof was actually issued or sent on
March 30, 1955. This witness, however, disclaimed having personal knowledge of its issuance or
release on said date either by mail or personal delivery because, according to him, he was assigned in
the income tax section of the Bureau of Internal Revenue in October, 1956 only. Sangil also declared
that there is no notation whatsover in said file copy (Exhibit "4"), nor even a slip of paper attached to
the records, to show that the original copy of said exhibit was ever actually issued or sent to the
taxpayer. He even admitted that he had no hand in the preparation or sending of written notices or
demand letters of the Bureau of Internal Revenue to the taxpayers, his duties being merely to keep the
dockets of taxpayers pertaining to income tax, to post and transmit papers to the other branches of the
Bureau for action, and to keep letters of taxpayers, memorandum and other official matters.
Respondent presented another witness, Mr. Eliseo B. Fernandez, whose duties, as record clerk of the
Records Control Section of the Bureau of Internal Revenue since 1957 (already past the limitation
period of this case), are to send mail and to keep a record book of letters which are mailed to the
taxpayers. Insofar as the testimony of this witness is concerned, he only declared as to the fact that
there appears in his record book a note (Exhibit "10") that a letter dated March 15, 1957 was mailed by
special delivery with return card to Gonzalo P. Nava. He admitted, however, that he was not the one
who prepared such entry in the record book. What was the nature of the letter does not appear; at any
rate, it was mailed beyond the 5-year limitation period.

The lower court also relied on the supposed notices noted in ink (followed by an illegible initial) in
Exhibit "3" for respondent (page 6, B.I.R. records), the first of which was purportedly sent on April 10,
1956, the second on July 3, 1956, and the final one on August 25, 1956, as well as on the supposed
"call-up" or demand letters referred to in a memorandum of an agent (Mrs. Canlas) of the Bureau of
Internal Revenue. (Exhibit "D", page 8, B.I.R. records). No witness for the respondent testified to the
issuance or sending of any of these supposed written demand letters or notices, nor was there any
duplicate or even a simple copy thereof found in petitioner Nava's Bureau of Internal Revenue file.
Although witness Sangil testified as to the meaning of the dates noted in Exhibit "3", his testimony
102
cannot be given much credence because those supposed notices were sent on or before August 25, 1956
at the latest, and, as hereinabove pointed out, the witness was assigned in the income tax section of the
Bureau of Internal Revenue since October, 1956 only.

Thus, contrary to the finding of the Court of Tax Appeals, respondent utterly failed to prove by
substantial evidence that the assessment notice dated March 30, 1955 and the other supposed written
demand letters or notices subsequent thereto were in fact issued or sent to taxpayer Nava. The
presumption that a letter duly directed and mailed was received in the regular course of mail (Sec. 5
[v], Rule 131, revised Rules of Court) cannot be applied to the case at bar.

The facts to be proved to raise this presumption are (a) that the letter was properly
addressed with postage prepaid, and (b) that it was mailed. Once these facts are proved, the
presumption is that the letter was received by the addressee as soon as it could have been
transmitted to him in the ordinary course of the mail. But if one of the said facts fails to appear,
the presumption does not lie." (VI, Moran, Comments on the Rules of Court, 1963 ed., 56-57;
citing Enriquez vs. Sun Life Assurance of Canada, 41 Phil. 269) (Emphasis supplied).

Since none of these requirements have been shown, there has been no valid and effective issuance or
release of said deficiency income tax assessment notice dated March 30, 1955 and of the other demand
letters or notices subsequent thereto, the latest of which was purportedly sent on August 25, 1956, and
these dates cannot be reckoned with in computing the period of prescription within which a court
action to collect the same may be brought.

The fact that in Exhibit "E" Nava acknowledged receipt of the second final notice personally delivered
to him is no proof that he received the first notice by mail. There is a difference between receiving a
second final notice and receiving a final notice for the second time.

It being undisputed that an original assessment of Nava's 1950 income tax return was made on May 15,
1951, and no valid and effective notice of the re-assessment having been made against the petitioner
after that date (May 15, 1951), it is evident that the period under Section 331 of the Tax Code within
which to make a re-assessment expired on May 15, 1956. Since the notice of said deficiency income
tax was effectively made on December 19, 1956 at the earliest, the judicial action to collect any
deficiency tax on Nava's 1950 income tax return has already prescribed under Section 332 (c) of the
Tax Code, it having been found by the Tax Appeals court that said return was not false or fraudulent.

While we have held that an assessment is made when sent within the prescribed period, even if
received by the taxpayer after its expiration (Coll. of Int. Rev. vs. Bautista, L-12250 and L-12259, May
27, 1959), this ruling makes it the more imperative that the release, mailing, or sending of the notice be
clearly and satisfactorily proved. Mere notations made without the taxpayer's intervention, notice, or
control, without adequate supporting evidence, cannot suffice; otherwise, the taxpayer would be at the
mercy of the revenue offices, without adequate protection or defense.

Having reached the conclusion that the action to collect said deficiency income tax has already
prescribed, it is unnecessary to discuss the other issues raised by petitioner Nava in the instant
appeal.1äwphï1.ñët

103
WHEREFORE, the decision of the Court of Tax Appeals under review is reversed, without costs.

NO DIGEST

G.R. No. L-38540 April 30, 1987

REPUBLIC OF THE PHILIPPINES, petitioner,


vs.
THE COURT OF APPEALS, and NIELSON & COMPANY, INC., respondents.

The Solicitor General for petitioner.

Quasha, Aspillera, Zafra, Tayag and Ancheta for respondents.

PADILLA, J.:

This is a petition for review on certiorari of the decision of the respondent Court of Appeals 1 in CA
G.R. No. 37417-R, dated 3 April 1974, reversing the decision of the then Court of First Instance of
Manila which ordered private respondent Nielson & Co., Inc. to pay the Government the amount of
P11,496.00 as ad valorem tax, occupation fees, additional residence tax and 25% surcharge for late
payment, for the years 1949 to 1952, and costs of suit, and of the resolution of the respondent Court,
dated 31 May 1974, denying petitioner's motion for reconsideration of said decision of 3 April 1974.

In a demand letter, dated 16 July 1955 (Exhibit A), the Commissioner of Internal Revenue assessed
private respondent deficiency taxes for the years 1949 to 1952, totalling P14,449.00, computed as
follows:

1-1/2% ad valorem tax on P448,000.00..........................P7,320.00

25% surcharge for late payment......................................1,830.00

Occupation fees for the years 1949

to 1952 at P1.00 per ha. per

year on 1, 230 hectares.....................................4,920.00

Additional residence tax on P79,000.00

at P1.00 per every P5,000.00

per year or P75.00 x 4 years................................303.20


104
25% surcharge for late payment.........................................75.00

TOTAL AMOUNT DUE............................ P14,449.00 2

Petitioner reiterated its demand upon private respondent for payment of said amount, per letters dated
24 April 1956 (Exhibit D), 19 September 1956 (Exhibit E) and 9 February 1960 (Exhibit F). Private
respondent did not contest the assessment in the Court of Tax Appeals. On the theory that the
assessment had become final and executory, petitioner filed a complaint for collection of the said
amount against private respondent with the Court of First Instance of Manila, where it was docketed as
Civil Case No. 42911. However, for failure to serve summons upon private respondent, the complaint
was dismissed, without prejudice, in the Court's order dated 30 June 1961. On motion, the order of
dismissal was set aside, at the same time giving petitioner sixty (60) days within which to serve
summons upon private respondent.

For failure anew to serve summons, the Court of First Instance of Manila issued an order dated 4
October 1962 dismissing Civil Case No. 42911 without prejudice. The order of dismissal became final
on 5 November 1962.

On 15 November 1962, the complaint against private respondent for collection of the same tax was
refiled, but the same was erroneously docketed as Civil Case No. 42911, the same case previously
dismissed without prejudice. Without correcting this error, another complaint was filed on 26
November 1963, docketed as Civil Case No. 55817, the subject matter of the present appeal.

As herein earlier stated, the Court a quo rendered a decision against the private respondent. On appeal
to the respondent Court of Appeals, the decision was reversed. Petitioner, Republic of the Philippines,
filed a motion for reconsideration which was likewise denied by said Court in a resolution dated 31
May 1974. Hence, this petition, with the following assignment of errors:

THE COURT OF APPEALS ERRED IN NOT HOLDING THAT THE LETTER OF ASSESSMENT
DATED JULY 16, 1955, EXHIBIT "A," WAS RECEIVED BY PRIVATE RESPONDENT IN THE
ORDINARY COURSE OF THE MAIL PURSUANT TO SECTION 8, RULE 13 OF THE REVISED
RULES OF COURT.

II

THE COURT OF APPEALS ERRED IN NOT HOLDING THAT PRIVATE RESPONDENT


FAILED TO REBUT THE PRESUMPTION THAT THE LETTER ASSESSMENT DATED JULY
16, 1955, HAVING BEEN DULY DIRECTED AND MAILED WAS RECEIVED IN THE
REGULAR COURSE OF THE MAIL AND THAT OFFICIAL DUTY HAS BEEN REGULARLY
PERFORMED.

III

105
THAT, ASSUMING, WITHOUT ADMITTING, THAT THE LETTER DATED JULY 16, 1955
(EXHIBIT "A") CANNOT BE CONSIDERED AS AN ASSESSMENT, ON THE THEORY THAT
THE SAME HAS NOT BEEN RECEIVED BY PRIVATE RESPONDENT, THE COURT OF
APPEALS ERRED IN NOT HOLDING THAT THE LETTER OF THE DEPUTY COLLECTOR
(NOW DEPUTY COMMISSIONER) OF INTERNAL REVENUE DATED SEPTEMBER 19, 1956
(EXHIBIT "E") IS ITSELF AN ASSESSMENT WHICH WAS DULY RECEIVED BY PRIVATE
RESPONDENT.

Relying on the provisions of Section 8, Rule 13 and Section 5, paragraphs m & v. Rule 131 of the
Revised Rules of Court, petitioner claims that the demand letter of 16 July 1955 showed an imprint
indicating that the original thereof was released and mailed on 4 August 1955 by the Chief, Records
Section of the Bureau of Internal Revenue, and that the original letter was not returned to said Bureau;
thus, said demand letter must be considered to have been received by the private
respondent. 3 According to petitioner, if service is made by ordinary mail, unless the actual date of
receipt is shown, service is deemed complete and effective upon the expiration of five (5) days after
mailing. 4 As the letter of demand dated 16 July 1955 was actually mailed to private respondent, there
arises the presumption that the letter was received by private respondent in the absence of evidence to
the contrary. 5 More so, where private respondent did not offer any evidence, except the self-serving
testimony of its witness, that it had not received the original copy of the demand letter dated 16 July
1955. 6

We do not agree with petitioner's above contentions. As correctly observed by the respondent court in
its appealed decision, while the contention of petitioner is correct that a mailed letter is deemed
received by the addressee in the ordinary course of mail, stilt this is merely a disputable presumption,
subject to controversion, and a direct denial of the receipt thereof shifts the burden upon the party
favored by the presumption to prove that the mailed letter was indeed received by the addressee. Thus:

Appellee contends that per Exhibit A, the notice was released and mailed to the appellant
by the BIR on Aug. 4, 1955 under the signature of the Chief, Records Section, Office;
that since the original thereof was not returned to the appellee, the presumption is that the
appellant received the mailed notice. This is correct, but this being merely a mere
disputable presumption, the same is subject to controversion, and a direct denial of the
receipt thereof shifts the burden upon the party favored by the presumption to prove that
the mailed letter was received by the addressee. The appellee, however, argues that since
notice was rc-,Ieased and mailed and the fact of its release was admitted by the appellant
the admission is proof that he received the mailed notice of assessment. We do not think
so. It is true the Court a quo made such a finding of fact, but as pointed out by the
appehant in its brief, and as borne out by the records, no such admission was ever made
by the appellant in the answer or in any other pleading, or in any declaration, oral or
documentary before the trial court. We note that the appellee has not met this challenge,
and after a review of the records, we find appeflant's assertion well-taken. 7

Since petitioner has not adduced proof that private respondent had in fact received the demand letter of
16 July 1955, it can not be assumed that private respondent received said letter. Records, however,
show that petitioner wrote private respondent a follow-up letter dated 19 September 1956, reiterating its
106
demand for the payment of taxes as originally demanded in petitioner's letter dated 16 July 1955. This
follow-up letter is considered a notice of assessment in itself which was duly received by private
respondent in accordance with its own admission. 8 The aforesaid letter reads:

September 19, 1956

Nielson and Company, Inc.

Ayala Boulevard, Manila

Gentlemen:

In reply to you (sic) letter dated June 1, 1956 relative to your pending internal revenue
tax liability involving the amount of P15,649.00 as annual occupation fees, ad valorem
and additional residence taxes, surcharges and penalty, originally demanded of you on
July 16, 1955, I have the honor to inform you that investigation conducted by an agent of
this office show that you and the Hixbar Gold Mining Co., Inc. entered into an agreement
in 1938 whereby you were given full exclusive and irrevocable control of all the
operations, development, processing and marketing of mineral products from the latter's
mines and that au the assessments, taxes and fees of any nature in connection with the
said operation, development, proceeding and marketing of these products shall be paid by
you. In view thereof, and it appearing that the aforesaid tax liabilities accrued when your
contract was in fun force and effect, you are therefore, the party hable for the payment
thereof, notwithstanding the alleged contract subsequently entered into by you and the
Hixbar Gold Mining Co., Inc. on September 9, 1954.

It is therefore, again requested that payment of the aforesaid amount of P15,649.00 be


made to the City Treasurer, Manila within five (5) days from your receipt hereof so that
this case may be closed.

You are further requested to pay the sum of P150.00 as compromise suggested in our
letter to you dated February 24, 1955, it appearing that the same has not as yet been paid
up to the present.

Very respectfully yours,

JOSE ARANAS

Deputy Collector of Internal


Revenue 9

Under Section 7 of Republic Act No. 1125, the assessment is appealable to the Court of Tax Appeals
within thirty (30) days from receipt of the letter. The taxpayer's failure to appeal in due time, as in the
case at bar, makes the assessment in question final, executory and demandable. Thus, private
respondent is now barred from disputing the correctness of the assessment or from invoking any
defense that would reopen the question of its liability on the merits. 10
107
In Mamburao Lumber Co. vs. Republic, 11 this Court further said:

In a suit for collection of internal revenue taxes, as in this case, where the assessment has already
become final and executory, the action to collect is akin to an action to enforce a judgment. No inquiry
can be made therein as to the merits of the original case or the justness of the judgment relied upon. ...

ACCORDINGLY, the appealed decision is hereby reversed. The decision of the Court a quo is hereby
reinstated. No costs.

SO ORDERED.

NO DIGEST

G.R. No. L-18804 May 27, 1965

COMMISSIONER OF INTERNAL REVENUE, petitioner,


vs.
WESTERN PACIFIC CORPORATION, respondent.

Office of the Solicitor General for petitioner.


R. Melo and A. S. Velasquez for respondent.

PAREDES, J.:

On March 2, 1959, the respondent Western Pacific Corporation, was assessed for P3,731.00, as
deficiency income tax for the year 1953. This assessment was brought about by the disallowance of
P8,265.82, listed in respondent's return for 1953, as expense items, and P10,387.50, as written off "bad
debts." The assessment was received by respondent on the same date (March 2, 1959). On March 5,
1959, the Commissioner of Internal Revenue wrote the respondent corporation a letter of demand for
the payment of the amount, including therein a breakdown of said assessment. Under date of June 29,
1959, respondent corporation, thru Ruifino Melo & Company, Consulting and Examining Auditors,
requested for non-assessment, claiming that there has been prescription in making the assessment, that
the expense items and bad debts were allowable deduction. The letter was accompanied by a
Resolution of the corporation, dated February 2, 1954, where it was resolved to write off the debts of
the people appearing in another annex. The Commissioner on July 30, 1959 replied to the request,
denying the same, and demanding the payment of the amount due within thirty (30) days from receipt
of said demand. On September 19, 1958, respondent corporation requested that it be permitted until
September 25, 1959, to submit formal objections to the assessment. The formal objections appearing in
the letter of September 22, 1959, were identical to those of the June 29, 1959 communication, reason
for which the Commissioner did not give any favorable action. The last letter of the Commissioner,
dated October 28, 1959, among others, requested payment of the assessment within ten (10) days from
receipt thereof.

108
On December 18, 1959, respondent Western Pacific Corporation, presented with the Court of Tax
Appeals a petition for Review of assessment made by the Commissioner, on three (3) counts, to wit:

(1) whether or not the making of the assessment had prescribed;

(2) whether expenses incurred in securing IGC Licenses are capital expenditures, and, as such,
not deductible from the income; and

(3) whether the bad debts written off should likewise be deducted.

When the issues were joined, by the filing of the Answer, and after hearing, the CTA rendered
judgment absolving the Western Pacific Corporation from the assessment. It, however, ruled out
prescription, stating that March 2, 1959, was the last day of the five (5) year period within which to
make the assessment. On this point, the CTA ruled:

However, we do not agree with petitioner that the assessment in question was issued beyond the
5-year statutory limitation. February 28, 1959 fell on a Saturday. Pursuant to Republic Act No.
1880, as, implemented by Executive Order No. 25, effective July 1, 1959, all bureaus and
offices of the government, except schools, court, hospitals and health clinics, hold office only
five days a week or from Monday to Friday. Saturday and Sunday, are constituted public
holidays or days of exemption from labor or work as far as government offices, including that of
respondent Commissioner, are concerned. The offices and bureaus concerned are officially
closed on those days. So that on February 28, 1959 and March 1, 1959, which were Saturday
and Sunday, respectively, the office of respondent was officially closed. And where the last day
for doing an act required by law falls on a holiday, the act may be done on the next succeeding
business day. (Section 31, Revised Administrative Code.) Similarly, in computing any period of
time prescribed by statute, the day of the act after which the designated period of time begins to
run is not included. But the last day of the period so computed is to be included, unless it is a
Sunday or a legal holiday, in which event the time shall run until the end of the next day which
is neither a Sunday or a holiday (Section 1, Rule 28, Rules of Court). Consequently, since
February 28, 1959 was a Saturday and the next day, March 1, 1959, a Sunday, respondent had
until the next succeeding business day, March 2, 1959, Monday, within which to issue the
deficiency assessment. The assessment in question having been issued on March 2, 1959, it was,
therefore, seasonably made.

We concur in the above findings and conclusions, convinced as We are, that they are actually and
legally correct..

The above ruling notwithstanding, the Commissioner of Internal Revenue appealed against the
judgment which absolved respondent Western Pacific Corporation from liability, alleging that the CTA
erred:.

(1) In taking cognizance of the case, notwithstanding lack of jurisdiction; and

(2) Granting it had jurisdiction, in considering the expense items and the written off bad debts as
deductible.1äwphï1.ñët
109
Without going into the merits of the decision absolving the respondent corporation of tax liability, We
find that the assessment made by the Commissioner should be maintained, for the simple reason that
when the petition for review was brought to the CTA by the respondent corporation, the said Court no
longer had jurisdiction to entertain the same. The assessment had long become final. A petition for
review should be presented, within the reglementary period, as provided for in Section 11, Republic
Act No. 1125, which is "thirty (30) days from receipt of the assessment." The thirty (30) day period is
jurisdictional (Pangasinan Transportation Co. vs. Blaquera, L-13101, April 29, 1960).

It will be noted that the assessment was received by the respondent corporation on March 2, 1959. It
was only on June 29, 1959, when said corporation formally assailed the assessment, on the grounds of
prescription in making the assessment and the impropriety of the disallowance of the listed deductions.
From March 3 to June 29, 1959, manifestly more than thirty (30) days had lapsed and the assessment
became final, executory and demandable (Ventanilla vs. Bd. of Tax Appeals, et al., L-7384, Dec. 19,
1955). Of course, in the interim, a number of communications were exchanged between the parties, the
latest of which was dated October 28, 1959. Even if this date is considered as the commencement of
the thirty (30) day period, still the petition for review with the CTA was out of time, because it was
only on December 18, 1959, that said petition was presented. Failure to comply with the thirty-day
statutory period would bar appeal and deprive the CTA of its jurisdiction to entertain and determine the
correctness of the assessment (Gibbs & Gibbs vs. Coll. of Int. Rev. & CTA, L-13453, Feb. 29, 1960).

IN VIEW OF THE FOREGOING, the decision of the CTA is hereby set aside for having been
rendered without jurisdiction, the assessment in question having been already final, executory and
demandable before the petition for review was presented; and another entered, ordering respondent
Western Pacific Corporation to pay the assessment made by the Collector of Internal Revenue, and the
further amount of 5% surcharge and 1% monthly interest on the amount assessed, from April 1, 1959
until date of full payment. Costs against the respondent corporation.

DIGEST:

CIR vs. Western Pacific Corporation


G.R. No. L-18804 May 27, 1965

Facts:
Respondent was assessed for P3,731.00, as deficiency income tax for the year 1953 which was brought
about by the disallowance of P8,265.82, listed in respondent's return for 1953, as expense items, and
P10,387.50, as written off "bad debts." The assessment was received by respondent on the same date
(March 2, 1959). On March 5, 1959, the CIR wrote a letter of demand for the payment of the amount,
including a breakdown of said assessment. Under date of June 29, 1959, respondent requested for non-
assessment, claiming that there has been prescription in making the assessment, that the expense items
and bad debts were allowable deduction. The Commissioner on July 30, 1959 denied the same, and
demanded for payment within 30 days from receipt of said demand. On September 19, 1959,
respondent requested that it be permitted until September 25, 1959, to submit formal objections to the
assessment. The formal objections appearing in the letter of September 22, 1959, were identical to
those of the June 29, 1959 communication. The last letter of the Commissioner, dated October 28,
1959, among others, requested payment of the assessment within 10 days from receipt.
110
Respondent appealed to the CTA, which absolved respondent but not based on prescription.

Issue:
Whether or not the action has prescribed

Ruling: YES
The Court that the assessment made by the Commissioner should be maintained, for the simple reason
that when the petition for review was brought to the CTA who no longer had jurisdiction to entertain
the same. The assessment had long become final. A petition for review should be presented, within the
reglementary period, which is 30 days from receipt of the assessment. The 30 day period is
jurisdictional. The assessment was received on March 2, 1959. It was only on June 29, 1959, when said
corporation formally assailed the assessment, on the grounds of prescription in making the assessment
and the impropriety of the disallowance of the listed deductions. From March 3 to June 29, 1959,
manifestly more than the 30 days had lapsed and the assessment became final, executory and
demandable. Failure to comply with the 30 day statutory period would bar appeal and deprive the CTA
of its jurisdiction. The decision of the CTA is set aside for having been rendered without jurisdiction,
the assessment in question having been already final, executory and demandable before the petition for
review was presented.

G.R. No. 162155 August 28, 2007

COMMISSIONER OF INTERNAL REVENUE and ARTURO V. PARCERO in his official


capacity as Revenue District Officer of Revenue District No. 049 (Makati), Petitioners,
vs.
PRIMETOWN PROPERTY GROUP, INC., Respondent.

DECISION

CORONA, J.:

This petition for review on certiorari1 seeks to set aside the August 1, 2003 decision2 of the Court of
Appeals (CA) in CA-G.R. SP No. 64782 and its February 9, 2004 resolution denying reconsideration.3

On March 11, 1999, Gilbert Yap, vice chair of respondent Primetown Property Group, Inc., applied for
the refund or credit of income tax respondent paid in 1997. In Yap's letter to petitioner revenue district
officer Arturo V. Parcero of Revenue District No. 049 (Makati) of the Bureau of Internal Revenue
(BIR),4 he explained that the increase in the cost of labor and materials and difficulty in obtaining
financing for projects and collecting receivables caused the real estate industry to slowdown.5 As a
consequence, while business was good during the first quarter of 1997, respondent suffered losses
amounting to ₱71,879,228 that year.6

According to Yap, because respondent suffered losses, it was not liable for income
taxes.7 Nevertheless, respondent paid its quarterly corporate income tax and remitted creditable
111
withholding tax from real estate sales to the BIR in the total amount of ₱26,318,398.32.8 Therefore,
respondent was entitled to tax refund or tax credit.9

On May 13, 1999, revenue officer Elizabeth Y. Santos required respondent to submit additional
documents to support its claim.10 Respondent complied but its claim was not acted upon. Thus, on
April 14, 2000, it filed a petition for review11 in the Court of Tax Appeals (CTA).

On December 15, 2000, the CTA dismissed the petition as it was filed beyond the two-year prescriptive
period for filing a judicial claim for tax refund or tax credit.12 It invoked Section 229 of the National
Internal Revenue Code (NIRC):

Sec. 229. Recovery of Taxes Erroneously or Illegally Collected. -- No suit or proceeding shall be
maintained in any court for the recovery of any national internal revenue tax hereafter alleged to have
been erroneously or illegally assessed or collected, or of any penalty claimed to have been collected
without authority, or of any sum alleged to have been excessively or in any manner wrongfully
collected, until a claim for refund or credit has been duly filed with the Commissioner; but such suit or
proceeding may be maintained, whether or not such tax, penalty, or sum has been paid under protest or
duress.

In any case, no such suit or proceeding shall be filed after the expiration of two (2) years from the
date of payment of the tax or penalty regardless of any supervening cause that may arise after
payment: Provided, however, That the Commissioner may, even without a claim therefor, refund or
credit any tax, where on the face of the return upon which payment was made, such payment appears
clearly to have been erroneously paid. (emphasis supplied)

The CTA found that respondent filed its final adjusted return on April 14, 1998. Thus, its right to claim
a refund or credit commenced on that date.13

The tax court applied Article 13 of the Civil Code which states:

Art. 13. When the law speaks of years, months, days or nights, it shall be understood that years are of
three hundred sixty-five days each; months, of thirty days; days, of twenty-four hours, and nights
from sunset to sunrise.

If the months are designated by their name, they shall be computed by the number of days which they
respectively have.

In computing a period, the first day shall be excluded, and the last included. (emphasis supplied)

Thus, according to the CTA, the two-year prescriptive period under Section 229 of the NIRC for the
filing of judicial claims was equivalent to 730 days. Because the year 2000 was a leap year,
respondent's petition, which was filed 731 days14 after respondent filed its final adjusted return, was
filed beyond the reglementary period.15

Respondent moved for reconsideration but it was denied.16 Hence, it filed an appeal in the CA.17

112
On August 1, 2003, the CA reversed and set aside the decision of the CTA.18 It ruled that Article 13 of
the Civil Code did not distinguish between a regular year and a leap year. According to the CA:

The rule that a year has 365 days applies, notwithstanding the fact that a particular year is a leap year.19

In other words, even if the year 2000 was a leap year, the periods covered by April 15, 1998 to April
14, 1999 and April 15, 1999 to April 14, 2000 should still be counted as 365 days each or a total of 730
days. A statute which is clear and explicit shall be neither interpreted nor construed.20

Petitioners moved for reconsideration but it was denied.21 Thus, this appeal.

Petitioners contend that tax refunds, being in the nature of an exemption, should be strictly construed
against claimants.22 Section 229 of the NIRC should be strictly applied against respondent inasmuch as
it has been consistently held that the prescriptive period (for the filing of tax refunds and tax credits)
begins to run on the day claimants file their final adjusted returns.23 Hence, the claim should have been
filed on or before April 13, 2000 or within 730 days, reckoned from the time respondent filed its final
adjusted return.

The conclusion of the CA that respondent filed its petition for review in the CTA within the two-year
prescriptive period provided in Section 229 of the NIRC is correct. Its basis, however, is not.

The rule is that the two-year prescriptive period is reckoned from the filing of the final adjusted
return.24 But how should the two-year prescriptive period be computed?

As already quoted, Article 13 of the Civil Code provides that when the law speaks of a year, it is
understood to be equivalent to 365 days. In National Marketing Corporation v. Tecson,25 we ruled that
a year is equivalent to 365 days regardless of whether it is a regular year or a leap year.26

However, in 1987, EO27 292 or the Administrative Code of 1987 was enacted. Section 31, Chapter
VIII, Book I thereof provides:

Sec. 31. Legal Periods. — "Year" shall be understood to be twelve calendar months; "month" of
thirty days, unless it refers to a specific calendar month in which case it shall be computed according to
the number of days the specific month contains; "day", to a day of twenty-four hours and; "night" from
sunrise to sunset. (emphasis supplied)

A calendar month is "a month designated in the calendar without regard to the number of days it may
contain."28 It is the "period of time running from the beginning of a certain numbered day up to, but not
including, the corresponding numbered day of the next month, and if there is not a sufficient number of
days in the next month, then up to and including the last day of that month."29 To illustrate, one
calendar month from December 31, 2007 will be from January 1, 2008 to January 31, 2008; one
calendar month from January 31, 2008 will be from February 1, 2008 until February 29, 2008.30

A law may be repealed expressly (by a categorical declaration that the law is revoked and abrogated by
another) or impliedly (when the provisions of a more recent law cannot be reasonably reconciled with
the previous one).31Section 27, Book VII (Final Provisions) of the Administrative Code of 1987 states:
113
Sec. 27. Repealing clause. — All laws, decrees, orders, rules and regulation, or portions thereof,
inconsistent with this Code are hereby repealed or modified accordingly.

A repealing clause like Sec. 27 above is not an express repealing clause because it fails to identify or
designate the laws to be abolished.32 Thus, the provision above only impliedly repealed all laws
inconsistent with the Administrative Code of 1987.1avvphi1

Implied repeals, however, are not favored. An implied repeal must have been clearly and unmistakably
intended by the legislature. The test is whether the subsequent law encompasses entirely the subject
matter of the former law and they cannot be logically or reasonably reconciled.33

Both Article 13 of the Civil Code and Section 31, Chapter VIII, Book I of the Administrative Code of
1987 deal with the same subject matter — the computation of legal periods. Under the Civil Code, a
year is equivalent to 365 days whether it be a regular year or a leap year. Under the Administrative
Code of 1987, however, a year is composed of 12 calendar months. Needless to state, under the
Administrative Code of 1987, the number of days is irrelevant.

There obviously exists a manifest incompatibility in the manner of computing legal periods under the
Civil Code and the Administrative Code of 1987. For this reason, we hold that Section 31, Chapter
VIII, Book I of the Administrative Code of 1987, being the more recent law, governs the computation
of legal periods. Lex posteriori derogat priori.

Applying Section 31, Chapter VIII, Book I of the Administrative Code of 1987 to this case, the two-
year prescriptive period (reckoned from the time respondent filed its final adjusted return34 on April 14,
1998) consisted of 24 calendar months, computed as follows:

Year 1st calendar April 15, to May 14,


1 month 1998 1998
2nd calendar May 15, to June 14,
month 1998 1998
3rd calendar June 15, to July 14,
month 1998 1998
4th calendar July 15, to August 14,
month 1998 1998
5th calendar August 15, to September
month 1998 14, 1998
6th calendar September to October 14,
month 15, 1998 1998
7th calendar October 15, to November
month 1998 14, 1998
8th calendar November to December

114
month 15, 1998 14, 1998
9th calendar December to January 14,
month 15, 1998 1999
10th calendar January 15, to February 14,
month 1999 1999
11th calendar February 15, to March 14,
month 1999 1999
12th calendar March 15, to April 14,
month 1999 1999
Year 13th calendar April 15, to May 14,
2 month 1999 1999
14th calendar May 15, to June 14,
month 1999 1999
15th calendar June 15, to July 14,
month 1999 1999
16th calendar July 15, to August 14,
month 1999 1999
17th calendar August 15, to September
month 1999 14, 1999
18th calendar September to October 14,
month 15, 1999 1999
19th calendar October 15, to November
month 1999 14, 1999
20th calendar November to December
month 15, 1999 14, 1999
21st calendar December to January 14,
month 15, 1999 2000
22nd calendar January 15, to February 14,
month 2000 2000
23rd calendar February 15, to March 14,
month 2000 2000
24th calendar March 15, to April 14,
month 2000 2000

115
We therefore hold that respondent's petition (filed on April 14, 2000) was filed on the last day of the
24th calendar month from the day respondent filed its final adjusted return. Hence, it was filed within
the reglementary period.

Accordingly, the petition is hereby DENIED. The case is REMANDED to the Court of Tax Appeals
which is ordered to expeditiously proceed to hear C.T.A. Case No. 6113 entitled Primetown Property
Group, Inc. v. Commissioner of Internal Revenue and Arturo V. Parcero.

No costs.SO ORDERED.

DIGEST 1:

On March 11, 1999, Primetown Property Group, Inc., through its vice chair, applied for the refund or
credit of income tax respondent paid in 1997 due to the slowdown of the real estate industry where
respondent suffered losses. With this, it contended that it was not liable for income taxes. Nevertheless,
respondent paid its quarterly corporate income tax and remitted creditable withholding tax from real
estate sales to the BIR therefore, respondent was entitled to tax refund or tax credit.

On May 13, 1999, revenue officer required respondent to submit additional documents to support its
claim. Respondent complied but its claim was not acted upon. Thus, on April 14, 2000, it filed a
petition for review in the Court of Tax Appeals (CTA).

On December 15, 2000, the CTA dismissed the petition as it was filed beyond the two-year prescriptive
period for filing a judicial claim for tax refund or tax credit as it found that respondent filed its final
adjusted return on April 14, 1998. Thus, its right to claim a refund or credit commenced on that date.

Both Article 13 of the Civil Code and Section 31, Chapter VIII, Book I of the Administrative Code of
1987 deal with the same subject matter the computation of legal periods. Under the Civil Code, a year
is equivalent to 365 days whether it be a regular year or a leap year. Under the Administrative Code of
1987, however, a year is composed of 12 calendar months. Needless to state, under the Administrative
Code of 1987, the number of days is irrelevant.

There obviously exists a manifest incompatibility in the manner of computing legal periods under the
Civil Code and the Administrative Code of 1987. For this reason, we hold that Section 31, Chapter
VIII, Book I of the Administrative Code of 1987, being the more recent law, governs the computation
of legal periods.

We therefore hold that respondent's petition (filed on April 14, 2000) was filed on the last day of the
24th calendar month from the day respondent filed its final adjusted return. Hence, it was filed within
the reglementary period

DIGEST 2:

CIR vs. Primetown Property Group, Inc.,


GR 161155 August 28, 2007
116
Facts:
On March 11, 1999, Gilbert Yap, vice chair of Primetown Property Group, Inc., applied for the refund
or credit of income tax respondent paid in 1997. He explained that the increase in the cost of labor and
materials and difficulty in obtaining financing for projects and collecting receivables caused the real
estate industry to slowdown. As a consequence, respondent suffered losses, and it was, therefore, not
liable for income taxes but still paid its quarterly corporate income tax and remitted creditable
withholding tax. Therefore, respondent was entitled to tax refund or tax credit.

On May 13, 1999, revenue officer required respondent to submit additional documents to support its
claim. Respondent complied but its claim was not acted upon. Thus, on April 14, 2000, it filed a
petition for review in the CTA.

On December 15, 2000, the CTA dismissed the petition as it was filed beyond the two-year prescriptive
period and applied Article 13 of the Civil Code, CA reversed and set aside the decision of the CTA.

Issue:
Whether or not Article 13 of the Civil Code should be applied in computing the legal period

Ruling: NO
EO 292 (Administrative Code of 1987) should be applied as it is the more recent law governing the
computation of legal period. Both Article 13 of the Civil Code and sec. 31 of the Administrative Code
deal with the same subject matter. Under the Civil Code a year is equivalent to 365 days regardless of
whether it is a leap year or not. According to the Admin Code, it is composed of 12 calendar months,
regardless of the number of days. In the present case, the petition which was filed on April 14, 2000,
was filed within the reglementary period as it was filed in the last day of the 24 th calendar month from
the day it filed its final return.

COMMISSIONER OF INTERNAL REVENUE, Petitioner, v. GJM PHILIPPINES


MANUFACTURING, INC., Respondent.

DECISION

PERALTA, J.:

For resolution is a Petition for Review under Rule 45 of the Rules of Court which petitioner
Commissioner of Internal Revenue (CIR) filed, praying for the reversal of the Decision1 of the Court of
Tax Appeals (CTA) En Banc dated March 6, 2012 and its Resolution2 dated July 12, 2012 in CTA EB
CASE No. 637. The CTA En Banc affirmed the Decision3 of the CTA First Division dated January 26,
117
2010 and its Resolution4 dated May 4, 2010 in favor of respondent GJM Philippines Manufacturing,
Inc. (GJM).

The facts, as culled from the records, are as follows:

On April 12, 2000, GJM filed its Annual Income Tax Return for the year 1999. Thereafter, its parent
company, Warnaco (ITK) Ltd., underwent bankruptcy proceedings, resulting in the transfer of
ownership over GJM and its global affiliates to Luen Thai Overseas Limited in December 2001. On
August 26, 2002, GJM informed the Revenue District Officer of Trece Martirez, through a letter, that
on April 29, 2002, it would be canceling its registered address in Makati and transferring to Rosario,
Cavite, which is under Revenue District Office (RDO) No. 54. On August 26, 2002, GJM's request for
transfer of its tax registration from RDO No. 48 to RDO No. 54 was confirmed through Transfer
Confirmation Notice No. OCN ITR 000018688.

On October 1 8, 2002, the Bureau of Internal Revenue (BIR) sent a letter of informal conference
informing GJM that the report of investigation on its income and business tax liabilities for 1999 had
been submitted. The report disclosed that GJM was still liable for an income tax deficiency and the
corresponding 20% interest, as well as for the compromise penalty in the total amount of
P1,192,541.51. Said tax deficiency allegedly resulted from certain disallowances/understatements, to
wit: (a) Loading and Shipment/Freight Out in the amount of P2,354,426.00; (b) Packing expense,
P8,859,975.00; (c) Salaries and Wages, P2,717,910.32; (d) Staff Employee Benefits, P1,191,965.87;
and (e) Fringe Benefits Tax, in the amount of P337,814.57. On October 24, 2002, GJM refuted said
findings through its Financial Controller.

On February 12, 2003, the Bureau of Internal Revenue (BIR) issued a Pre-Assessment Notice and
Details of Discrepancies against GJM. On April 14, 2003, it issued an undated Assessment Notice,
indicating a deficiency income tax assessment in the amount of PI,480,099.29. On July 25, 2003, the
BIR issued a Preliminary Collection Letter requesting GJM to pay said income tax deficiency for the
taxable year 1999. Said letter was addressed to GJM's former address in Pio del Pilar, Makati. On
August 18, 2003, although the BIR sent a Final Notice Before Seizure to GJM's address in Cavite, the
latter claimed that it did not receive the same.

On December 8, 2003, GJM received a Warrant of Distraint and/or Levy from the BIR RDO No. 48-
West Makati. The company then filed its Letter Protest on January 7, 2004, which the BIR denied on
January 15, 2004. Hence, GJM filed a Petition for Review before the CTA.

On January 26, 2010, the CTA First Division rendered a Decision in favor of GJM, the dispositive
portion of which reads:

WHEREFORE, the deficiency income tax assessment in the amount of PI,480,099.29, inclusive of
interest, for taxable year 1999, covered by Formal Assessment Notice No. IT-1731 6-99-03-282 and
the Warrant of Distraint and/or Levy dated November 27, 2003, both issued against petitioner by
respondent, are hereby CANCELLED and WITHDRAWN.

Accordingly, respondent is hereby ORDERED to cease and desist from implementing the said
118
assessment and Warrant.

SO ORDERED.5ChanRoblesVirtualawlibrary

When its Motion for Reconsideration was denied, the CIR brought the case to the CTA En Banc.

On March 6, 2012, the CTA En Banc denied the CIR's petition, thus:

WHEREFORE, the Petition for Review is hereby DENIED. Accordingly, the impugned Decision
dated January 26, 2010 and Resolution dated May 4, 2010 are hereby AFFIRMEDin toto.

SO ORDERED.6ChanRoblesVirtualawlibrary

The CIR filed a Motion for Reconsideration but the same was denied for lack of merit. Thus, the
instant petition.

The CIR raised the following issues:

I.

WHETHER OR NOT THE FORMAL ASSESSMENT NOTICE (FAN) FOR DEFICIENCY INCOME
TAX ISSUED TO GJM FOR TAXABLE YEAR 1999 WAS RELEASED, MAILED, AND SENT
WITHIN THE THREE (3)-YEAR PRESCRIPTIVE PERIOD UNDER SECTION 203 OF THE NIRC
OF 1997.

II.

WHETHER OR NOT THE BIR'S RIGHT TO ASSESS GJM FOR DEFICIENCY INCOME TAX
FOR TAXABLE YEAR 1999 HAS ALREADY PRESCRIBED.

The petition lacks merit.

Section 203 of the 1997 National Internal Revenue Code (NIRC), as amended, specifically provides for
the period within which the C1R must make an assessment. It provides:

SEC. 203. Period of Limitation Upon Assessment and Collection. - Except as provided in Section
222, internal revenue taxes shall be assessed within three (3) years after the last day prescribed by law
for the filing of the return, and no proceeding in court without assessment for the collection of such
taxes shall be begun after the expiration of such period: Provided, That in a case where a return is filed
beyond the period prescribed by law, the three (3)-year period shall be counted from the day the return
was tiled. For purposes of this Section, a return filed before the last day prescribed by law for the filing
thereof shall be considered as filed on such last day. (Emphasis supplied)

Thus, the CIR has three (3) years from the date of the actual filing of the return or from the last day
prescribed by law for the filing of the return, whichever is later, to assess internal revenue taxes. Here,

119
GJM filed its Annual Income fax Return for the taxable year 1999 on April 12, 2000. The three (3)-
year prescriptive period, therefore, was only until April 15, 2003. The records reveal that the BIR sent
the FAN through registered mail on April 14, 2003, well-within the required period. The Court has held
that when an assessment is made within the prescriptive period, as in the case at bar, receipt by the
taxpayer may or may not be within said period. But it must be clarified that the rule does not dispense
with the requirement that the taxpayer should actually receive the assessment notice, even beyond the
prescriptive period.7 GJM, however, denies ever having received any FAN.

If the taxpayer denies having received an assessment from the BIR, it then becomes incumbent upon
the latter to prove by competent evidence that such notice was indeed received by the addressee.8 Flere,
the onus probandi has shifted to the BIR to show by contrary evidence that GJM indeed received the
assessment in the due course of mail. It has been settled that while a mailed letter is deemed received
by the addressee in the course of mail, this is merely a disputable presumption subject to controversion,
the direct denial of which shifts the burden to the sender to prove that the mailed letter was, in fact,
received by the addressee.9

To prove the fact of mailing, it is essential to present the registry receipt issued by the Bureau of Posts
or the Registry return card which would have been signed by the taxpayer or its authorized
representative. And if said documents could not be located, the CIR should have, at the very least,
submitted to the Court a certification issued by the Bureau of Posts and any other pertinent document
executed with its intervention. The Court does not put much credence to the self-serving
documentations made by the BIR personnel, especially if they are unsupported by substantial evidence
establishing the feet of mailing. While it is true that an assessment is made when the notice is sent
within the prescribed period, the release, mailing, or sending of the same must still be clearly and
satisfactorily proved. Mere notations made without the taxpayer's intervention, notice or control, and
without adequate supporting evidence cannot suffice. Otherwise, the defenseless taxpayer would be
unreasonably placed at the mercy of the revenue offices.10

The BIR's failure to prove GJM's receipt of the assessment leads to no other conclusion but that no
assessment was issued. Consequently, the government's right to issue an assessment for the said period
has already prescribed. The CIR offered in evidence Transmittal Letter No. 282 dated April 14, 2003
prepared and signed by one Ma. Nieva A. Guerrero, as Chief of the Assessment Division of BIR
Revenue Region No. 8-Makati, to show that the FAN was actually served upon GJM. However, it
never presented Guerrero to testify on said letter, considering that GJM vehemently denied receiving
the subject FAN and the Details of Discrepancies. Also, the CIR presented the Certification signed by
the Postmaster of Rosario, Cavite, Nicarter Looc, which supposedly proves the fact of mailing of the
FAN and Details of Discrepancy. It also adduced evidence of mail envelopes stamped February 17,
2003 and April 14, 2003, which were meant to prove that, on said dates, the Preliminary Assessment
Notice (PAN) and the FAN were delivered, respectively. Said envelopes also indicate that they were
posted from the Makati Central Post Office. However, according to the Postmaster's Certification, of all
the mail matters addressed to GJM which were received by the Cavite Post Office from February 12,
2003 to September 9, 2003, only two (2) came from the Makati Central Post Office. These two (2)
were received by the Cavite Post Office on February 12, 2003 and May 13, 2003. But the registered
mail could not have been the PAN since the latter was mailed only on February 17, 2003, and the FAN,
although mailed on April 14, 2003, was not proven to be the mail received on May 13, 2003. The CIR
120
likewise failed to show that said mail matters received indeed came from it. It could have simply
presented the registry receipt or the registry return card accompanying the envelope purportedly
containing the assessment notice, but it offered no explanation why it failed to do so. Hence, the CTA
aptly ruled that the CIR failed to discharge its duty to present any evidence to show that GJM indeed
received the FAN sent through registered mail on April 14, 2003.

The Court wishes to note and reiterate that it is not a trier of facts. The CIR mainly raised issues on
factual findings which have already been thoroughly discussed below by both the CTA First Division
and the CTA En Banc. Oft-repeated is the rule that the Court will not lightly set aside the conclusions
reached by the CTA which, by the very nature of its function of being dedicated exclusively to the
resolution of tax problems, has accordingly developed an expertise on the subject, unless there has been
an abuse or improvident exercise of authority. This Court recognizes that the CTA's findings can only
be disturbed on appeal if they are not supported by substantial evidence, or there is a showing of gross
error or abuse on the part of the Tax Court. In the absence of any clear and convincing proof to the
contrary, the Court must presume that the CTA rendered a decision which is valid in every respect. It
has been the Court's long-standing policy and practice to respect the conclusions of quasi-judicial
agencies such as the CTA, a highly specialized body specifically created for the purpose of reviewing
tax cases.11

The Court hereby sustains the order of cancellation and withdrawal of the Formal Assessment Notice
No. IT-17316-99-03-282, and the Warrant of Distraint and/or Levy dated November 27, 2003.

WHEREFORE, PREMISES CONSIDERED, the petition is DENIED. The Decision of the Court of
Tax Appeals En Banc dated March 6, 2012 and its Resolution dated July 12, 2012 in CTA EB CASE
No. 637 are hereby AFFIRMED.SO ORDERED.

DIGEST:

G.R. No. 202695 February 29, 2016

Facts: On April 12, 2000, GJM filed its Annual Income Tax Return for the year 1999. CIR found out
that GJM had tax deficiencies due to disallowances/understatements, therefore, CIR had the right to
assess GJM within the 3 year prescriptive period under Sec. 203 of the NIRC or until April 15, 2003.
On February 17, 2003, CIR delivered the Preliminary Assessment Notice (PAN) to GJM.
Subsequently, on April 14, 2003, the Formal Assessment Notice (FAN) were delivered by the CIR.
GJM denied having received any assessment from the BIR, thus, such right of assessment by the latter
has prescribed.

Issue: Whether or not the denial of GJM having received the Formal Assessment Notice (FAN) made
such right of assessment by the CIR prescribe;

Held: Yes, it has been settled that while a mailed letter is deemed received by the addressee in the
course of mail, this is merely a disputable presumption subject to controversion, the direct denial of
which shifts the burden to the sender to prove that the mailed letter was, in fact, received by the
121
addressee. In the case at bar, CIR was not able to prove that GJM has received the FAN sent by them
ergo their right to assess has prescribed.

G.R. No. L-18956 April 27, 1972

REPUBLIC OF THE PHILIPPINES, plaintiff-appellee,


vs.
MARSMAN DEVELOPMENT COMPANY and/or F.H. BURGESS, in his capacity as
Liquidator of the Marsman Development Company, defendants-appellants.

BARREDO, J.:p

Appeal from the decision of the Court of First Instance of Manila, the Honorable Conrado, M.
Vasquez, presiding, sentencing defendants-appellants to pay the amounts of P44,134.35, P6,603.20 and
P456.12, plus legal interest from August 26, 1959, on the first item, and, from September 5, 1958, on
the later two, representing sales taxes and forest charges, together with surcharges and penalties.

As found by His Honor, the factual setting of the decision is as follows:

Defendant corporation was a timber licensee holding Timber Licensee Agreement No.
37-A, with concessions in the Municipality of Basud and Mondazo, Camarines Norte.
Sometime before October 15, 1953 an investigation was conducted on the business
operation and activities of the corporation leading to the discovery that certain taxes were
due (from) it on logs produced from its concession. On October 15, 1953, the Deputy
Collector of Internal Revenue demanded the payment of P13,136.00 representing forest
charges due from May 18, 1950 to September 30, 1953, and a surcharge of 25% (Exh.
M). On September 13, 1954, after further investigation another assessment was sent to
the defendant corporation by the Bureau of Internal Revenue demanding from it the total
sum of P45,541.66 representing deficiency sales tax, forest charges, surcharges and
penalties (Exh. A). On November 8, 1954 another assessment was addressed to the
defendant corporation for the payment of P456.12 as 25% surcharge for discharging
lumber without permit (Exh. P). The three assessments totalling P59,133.78 are the
subject matter of the instant case for collection.

xxx xxx xxx

The-contention of the defendant that the assessment in question have not yet become
final and executory is not borne out by the record. The Bureau of Internal Revenue made
its first demand for the payment of P13,136.00 as forest charges and surcharges in the
letter dated October 15, 1953 (Exh. M). After further investigation, a second assessment
in the total amount of P45,541.66 was demanded from the defendant corporation
representing sales tax and surcharges, and is contained in the letter dated September 13,
122
1954 (Exh. A). The third assessment for the payment of P456.12 representing 25%
surcharge for discharging lumber without permit was made on November 8, 1954 (Exh.
B).

The first acknowledgment by the defendant corporation of its receipt of assessment


contained in the letter of September 13, 1954, Exh. A, was the letter of the defendant
corporation under the signature of its counsel, Atty. Pedro L. Moya dated December 28,
1954, wherein it is requested that said defendant be furnished with an itemized statement
of the said taxes and wherein notice is served of its intention to question the validity and
the legality of the assessments and to appear before the Conference Staff of the Bureau of
Internal Revenue in connection with the said tax (Exhibit B). In reply to the letter,
Exhibit B, the Bureau of Internal Revenue wrote Atty. Moya a letter dated February 11,
1955 informing him that before the case may be acted upon by the Conference Staff, it
was necessary that the defendant corporation comply within 10 days from date of said
letter, with the provisions of Dept. Order No. 213 dated November 2, 1954 which
required, among others, that requests for reinvestigation or reexamination of tax
assessments shall be made in writing under oath of the taxpayer concerned, specifying the
ground or grounds relied upon for the revision of the assessment and accompanied by
such documents and other documents relied upon in support of the request; and that, as a
general rule, the revision will be granted only upon payment of one-half of the total
assessments and upon filing of a bond to guarantee the payment of the balance of the tax
(Exhibit C). Acknowledgment of Exhibit C was made by Atty. Moya in the latter's letter
of February 23, 1955 wherein, for the reasons therein stated, he requested exemption
from the requirements contained in the letter Exhibit C (Exhibit D). In Reply to Exhibit
D, the Collector of Internal Revenue wrote Atty. Moya on May 3, 1955 informing him
that his request to exempt his client from the requirements contained in the letter dated
February 11, 1955, cannot be favorably considered and that in order that the Conference
Staff may be directed to hear the case on the merits, the said requirements must be
complied with within five days from receipt of said letter; otherwise, the "assessment will
be considered final" (Exhibit E). A follow-up letter dated June 4, 1955, was addressed to
Atty. Moya after discovering that the requirements mentioned in the letters dated
February 11, 1955 and March 3, 1955 have not been complied with inspite of the
considerable length of time that had already elapsed (Exhibit F). In the last paragraph of
the said letter, Exhibit F, the defendant corporation was warned that unless the
aforementioned requirements are complied with within five (5) days from receipt, the
"case will be considered abandoned and appropriate action will be taken in accordance
with law". Again on November 14, 1955, after discovering that the letters dated February
11, 1955, March 3, 1955 and June 4, 1955 have remained unheeded by the defendant
corporation, the latter was given another chance of complying with the requirements
mentioned within five days from receipt of said letter otherwise, the Bureau of Internal
Revenue "will be constrained to enforce the immediate collection of the deficiency
percentage tax and forest charges due" (Exhibit G).

On April 27, 1956, the Bureau of Internal Revenue issued "final tax notices" to the
defendant corporation. Although the letters containing the "final tax notices" were not
123
presented in evidence, the defendants admit having received the same, as shown by the
contents of defendant corporation's letters dated May 10, 1956, Exhibit H, and August 7,
1956, Exhibit J. In said Exhibit H defendant corporation again protested the assessment
of P45,541.66 and reiterated its request for specification of the items disputing the
assessment in question. It further requests for a period of 30 days from the receipt of the
specifications within which to consider its tax liability, further reserving its right to
contest the legality or validity of the assessment or any particular items thereof within the
said period of 30 days. Defendant corporation also protested the sending of final notices
and requested that they be countermanded or withheld. Finding no merit in the protests of
the defendant corporation, a warrant of distraint and levy was issued against it by the
Bureau of Internal Revenue on July 3, 1956 (Exhibit O).

On August 3, 1956, defendant corporation again wrote the Collector of Internal Revenue
acknowledging the receipt of the warrant of distraint and levy served upon it and
reiterating its request for a specification of the different items of the assessment, subject
to the right to contest the legality and validity of the same within 30 days after receipt of
said specifications (Exh. J). The record does not show what action was taken on the
request contained in said letter on August 3, 1956. The next communication appearing in
the record is that of the Commissioner of Internal Revenue dated July 30, 1959,
addressed to the defendant corporation demanding on the letter the payment of the
assessment of P45,541.66 which has remained unpaid, and informing the said corporation
that if they do not settle said tax obligation within five days from receipt thereof, the
Bureau of Internal Revenue will be constrained to file an action in Court for the
collection thereof without further notice (Exhibit I). Defendant corporation replied to
Exhibit I in a letter dated August 17, 1959 stating that it needed more time to go over the
records and vouchers, and requesting for an extension of 10 days (Exhibit E). In another
letter of same date, the defendant corporation reiterated its exception to the validity and
legality of the assessment against it in the sum of P45,541.66 and its request for a detailed
statement of the transactions involved (Exhibit L). [Record on Appeal pp. 188-189, 190-
195.]

According to the Record on Appeal, and as additionally stated also by the trial court, the original
complaint filed on September 5, 1958 prayed for the payment of only P13,695.96, and it was only in an
amended complaint filed on August 26, 1959 and admitted on September 23, 1959 that, for the first
time, the amount of P59,133.78 was judicially demanded to be paid.

Upon these facts, appellants now complain that

THE LOWER COURT ERRED IN DECLARING THAT THE NOTICES OF THE


COMMISSIONER OF INTERNAL REVENUE DATED APRIL 27, 1956 WERE THE
"ASSESSMENTS" THAT BECAME FINAL AND EXECUTORY.

II

124
THE LOWER COURT ERRED IN DECLARING THAT THE GOVERNMENT'S
RIGHT TO ASSESS AND COLLECT THE TAXES FOR THE YEARS 1947 TO
SEPTEMBER 23, 1949 HAS NOT PRESCRIBED.

III

THE LOWER COURT LIKEWISE ERRED IN DECLARING THAT THE


GOVERNMENT'S RIGHT TO COLLECT THE SUM OF P45,541.66 HAS NOT
PRESCRIBED.

IV

THE LOWER COURT FURTHER ERRED IN NOT DECLARING THAT SUIT


AGAINST F.H. BURGESS IN HIS CAPACITY AS LIQUIDATOR OF MARSMAN
DEVELOPMENT COMPANY HAS PRESCRIBED AND IN ORDERING HIM TO
PAY THE SUMS CONTAINED IN ITS DECISION.

The Court does not agree.

Anent the first assignment of error, it may be stated that regardless of what might have been alleged in
appellee's pleadings and memoranda, the facts proven by evidence, which are not alleged to have been
objected to as varying supposed judicial admissions, unmistakably show that when Atty. Pedro L.
Moya acknowledged receipt on December 28, 1954, on behalf of appellant corporation, of the Bureau
of Internal Revenue's assessments of September 13, 1954 and November 8, 1954, requesting at the
same time for a reinvestigation before the Conference Staff, he was informed that his request for
investigation would not be given due course unless his client priorly complied within ten (10) days
from Februaxy 11, 1955, the date of the letter of the Bureau, with the provisions of Department Order
No. 213, dated November 2, 1954, which required inter alia, that requests for reinvestigation or
reexamination of tax assessments should be made in writing and under oath of the taxpayer concerned,
specifying the ground or grounds relied upon for the requested revision and accompanied by the
documents relied upon, in support of the request, as well as by the payment of one-half of the total
assessments, plus a bond to guarantee payment of the balance, but appellants failed to comply with said
conditions: that in reply to Atty. Moya's request for exemption from the Department order, on March 3,
1955 (not May), the attorney was advised that his request was denied and that if the corporation failed
to comply therewith within five (5) days from receipt of the letter, "the assessment (would) be
considered final"; that on June 4, 1955, said Atty. Moya was reminded in writing that the previous
demands had not been properly attended to, with the warning that should appellants further fail to
comply with the requirements in the letter of February 11, 1955, within five (5) days from receipt
thereof, the "case (would) be considered abandoned and appropriate action (would) be taken in
accordance with law"; that even as late as November 14, 1955, the corporation was again advised to
comply with the earlier communications of February 11, 1955, March 3, 1955 and June 4, 1955, within
five days, otherwise, the Bureau of Internal Revenue would "be constrained to enforce immediate
collection of the deficiency percentage tax and forest charges due"; that as nothing was done by it to
comply with this last letter, the Bureau of Internal Revenue issued, on April 27, 1956, "final tax
notices" to it, and all that the latter did after receipt thereof was to reiterate, by its letters of May 19,

125
1956 and August 7, 1956, its request for specification of the items involved in the assessment and for
another period of 30 days within which to consider its tax liabilities, reserving once more its right to
contest the legality or validity of the assessment and to protest the issuance of the "final tax notices";
that evidently tired of awaiting compliance by the said appellant, the Bureau of Internal Revenue issued
on July 3, 1956 a warrant of distraint and levy against it, which it acknowledged on August 3, 1956,
only to reiterate again its position previously stated of asking for specification and reserving its right to
contest the validity of the assessment; that, finally, on July 30, 1959, after three years, the
Commissioner of Internal Revenue made extrajudicial demand for payment of the amounts in question
within five (5) days, and since no payment came, and instead, defendants asked for more time to go
over the records and, under separate cover, questioned for the nth time, the validity of the assessment,
the present action was filed.

Under these circumstances, it is plain that His Honor committed no error in holding that the period to
question the tax assessments herein involved had already expired when the Commissioner of Internal
Revenue initiated this suit against defendants. Defendant corporation aknowledged receipt of the said
assessments way back on December 28, 1954, and, in fact, it requested for a reinvestigation before the
Conference Staff, but when the Bureau demanded compliance with the prerequisites aforementioned of
such reinvestigation, the corporation failed to comply. The corporation did ask for exemption, but when
this request was denied, again there was no compliance. In view of such non-compliance, in its letter of
March 3, 1955, the Bureau unequivocally warned the corporation that should it fail further to comply,
within five days from receipt thereof, the "assessments (would) be considered final". still no
compliance came. Subsequent follow-up letters brought no better results.

As it appears, therefore, appellant corporation, by its own omission, made it impossible for the Bureau
of Internal Revenue to act on its motion for reconsideration. Not that it would have otherwise mattered,
for it has been held that the mere filing of such a motion does not suspend the running of the period for
the collection of the tax,1 which implies that any assessment made by the Bureau is supposed to be final
and executory, insofar as the taxpayer is concerned, unless revised by the Bureau in accordance with
law and regulations, but it is to be emphasized that a taxpayer cannot delay the collection of taxes by
the simple expedient of barely asking for clarification or reconsideration, very often unnecessary and
unwarranted, without doing anything to comply with the statutory and reglementary requirements for
the reconsideration of the assessment made against him. In any event, since appellant corporation did
nothing from December, 1954 when it acknowledged receipt of the assessment now impugned to
appeal the same, if such an appeal was possible, to the Court of Tax Appeals, even after it was warned
by the Bureau of Internal Revenue that its failure to comply with the requirements for reconsideration
within five (5) days would result in its being "considered" final, We find no merit in appellants' posture
that the assessments here in question has not yet become final and executory. Consequently, overruling
of appellants' first assignment of error is clearly in order.

In their second assignment of error, appellants raise the issue of prescription. They point out that the
Collector of Internal Revenue had only five years within which to assess the percentage and forest
charges herein involved. Since it does not appear, however, that appellant corporation had filed any
return in relation to the taxes herein involved, and it was incumbent upon appellants to show that such a
return had been submitted,2 We find the following holding of His Honor to be fully in accordance with
law:
126
Defendants' contention that the right to assess the percentage and forest charges for the
period from 1947 to September 23, 1949 had already prescribed is based on the provision
of Section 231 of the Revenue Code which requires the Collector of Internal Revenue to
assess the tax within the period of five years. The Court agrees with the plaintiff that said
Section 231 is not applicable in this case inasmuch as defendant corporation did not file
returns for the taxes in question. The pertinent provision applicable herein is Section 332
(a) which provides that "in case of a false or fraudulent return or of a failure to file a
return, the tax may be assessed ... at anytime within ten years after the discovery of the
falsity, fraud or omission." The assessments made on October 15, 1953, September 13,
1954, and November 3, 1954 were all within the aforecited 10-year period for the
assessment of the tax.

Even if the Court were to consider, as appellants suggest, the fact brought out in their brief but not
found by the trial court that what are being sought to be collected are deficiency taxes, thereby
implying a return must have been filed, nothing can he gained by appellants, for in order that the filing
of a return may serve as the starting point of the period for the making of an assessment, the return
must be as substantive complete as to include the needed details on which the full assessment may be
made, and appellants have not shown that such was the nature of the return they would infer had been
filed by the corporation.3

Appellants' third assignment of error does not require any extended discussion. The argument
thereunder that the judicial action for the recovery of the bigger amount of P45,541.66 was not filed
within five (5) years from September 13, 1954, the date of the earliest assessment, has neither factual
nor legal basis. As aptly explained by his Honor, such argument proceeds from the erroneous premises
that because the amended complaint in which the said amount was first alleged and demanded was
formally admitted by the court only on September 23, 1959 and that the filing of said amended
complaint on August 26, 1959 is immaterial. While in the procedural sense, especially in relation to the
possible necessity of and time for the filing of responsive and other corresponding pleadings, an
amended complaint is deemed filed only as of the date of its admission, nothing in Breslin v. Luzon, 84
Phil. 625, relied upon by appellant, was intended to modify the self-evident proposition that for
practical reasons and to avoid the complications that may rise from undue delays in the admission
thereof, such an amended complaint must be considered as filed, for the purposes of such a substantive
matter as prescription, on the date it is actually filed with the court, regardless of when it is ultimately
formally admitted by the court. After all, the only purpose of requiring leave of and formal admission
by the court of an amended pleading after issues have already been joined as to the original ones is to
prevent the injection of other issues which might either to be considered as barred already or made the
subject of another proceeding, if they are not anyway indispensable for the resolution of the original
ones and no unnecessary multiplicity of suits would result; so, when the court ultimately admits the
amendment, the legal effect, for substantive purposes, of such admission retroacts as a rule to the date
of its actual filing.

Appellants' last assignment of error was disposed of by the trial court this wise:

The defendants further contend that the present action is already barred under section 77
of the Corporation Law, Act No. 1459, as amended, which allows the corporate existence
127
of a corporation to continue only for three years after its dissolution, for the purpose of
presenting or defending suits by or against it, and to settle and close its affairs. They point
out that inasmuch as the Marsman Development Co. was extra-judicially dissolved on
April 23, 1954, a fact admitted in the amended complaint, the filing of both the original
complaint on September 8, 1958 and the amended complaint on August 26, 1956 was
beyond the aforesaid three-year period.

The record shows that the filing of the amended complaint was intended, among others,
to include as a party defendant, in an alternative capacity, Mr. F.H. Burgess, who is the
liquidator of the Marsman Development Co. Although it is an admitted fact that the
defendant corporation was extrajudicially dissolved on April 23, 1954, there is no claim
that the affairs of said corporation had already been finally liquidated or settled.
Evidently, Mr. F.H. Burgess is still continuing in his aforesaid capacity as liquidator of
the Marsman Development Co. While section 77 of the Corporation Law provides for a
three-year period for the continuation of the corporate existence of the corporation for
purposes of liquidation, there is nothing in said provision which bars an action for the
recovery of the debts of the corporation against the liquidator thereof, after the lapse of
the said three-year period.

We agree with His Honor. The stress given by appellants to the extinction of the corporate and juridical
personality as such of appellant corporation by virtue of its extra-judicial dissolution which admittedly
took place on April 23, 1954 is misdirected. While Section 77 of the Corporation Law does provide
that:

Every corporation whose charter expires by its own limitation or is annulled by forfeiture
or otherwise, or whose corporate existence for other purposes is terminated in any other
manner, shall nevertheless be continued as a body corporate for three years after the time
when it would have been so dissolved, for the purpose of prosecuting and defending suits
by or against it and of enabling it gradually to settle and close its affairs, to dispose of and
convey its property and to divide its capital stock, but not for the purpose of continuing
the business for which it was established.

the next provision, Section 78, adds for clarification:

At any time during said three years said corporation is authorized and empowered to
convey all of its property to trustees for the benefit of members, stock-holders, creditors,
and others interested. From and after any such conveyance by the corporation of its
property in trust for the benefit of its members, stockholders, creditors, and others in
interest, all interest which the corporation had in the property terminates, the legal
interest vests in the trustee, and the beneficial interest in the members, stockholders,
creditors, or other persons in interest.

It is to be recalled that the assessments against appellant corporation for deficiency taxes due for its
operations since 1947 were made by the Bureau of Internal Revenue on October 15, 1953, September
13, 1954 and November 8, 1954, such that the first was before its dissolution and the last two not later

128
than six months after such dissolution. Thus, in whatever way the matter may be viewed, the
Government became the creditor of the corporation before the completion of its dissolution by the
liquidation of its assets. Appellant F.H. Burgess, whom it chose as liquidator, became in law the trustee
of all its assets for the benefit of all persons enumerated in Section 78, including its creditors, among
whom is the Government, for the taxes herein involved. To assume otherwise would render the extra-
judicial dissolution illegal and void, since, according to Section 62 of the Corporation Law, such kind
of dissolution is permitted only when it "does not affect the rights of any creditor having a claim
against the corporation." It is immaterial that the present action was filed after the expiration of three
years after April 23, 1954, for at the very least, and assuming that judicial enforcement of taxes may
not be initiated after said three years despite the fact that the actual liquidation has not been terminated
and the one in charge thereof is still holding the assets of the corporation, obviously for the benefit of
all the creditors thereof, the assessment aforementioned, made within the three years, definitely
established the Government as a creditor of the corporation for whom the liquidator is supposed to hold
assets of the corporation. And since the suit at bar is only for the collection of taxes finally assessed
against the corporation within the three years invoked by appellants, their fourth assignment of error
cannot be sustained. As to the allegation that appellant Burgess has not in fact received any property or
asset of the corporation, that is a matter that can well be taken care of in the execution of the judgment
which may be rendered herein, albeit it seems some kind of fraud would be perceptible, if the
corporation had been dissolved without leaving any assets whatsoever with the liquidator.

ACCORDINGLY, the judgment of the trial court is affirmed with costs against the appellants.

DIGEST:

Republic of the Philippines vs. Marsman Development Company


G.R. No. L-18956 April 27, 1972

Facts:
On October 15, 1953 the Deputy Collector of Internal Revenue demanded respondent (timber licensee)
to pay forest charges due from May 18, 1950 to September 30, 1953, and a surcharge of 25%. On
September 13, 1954, after further investigation another assessment was sent to the defendant
demanding sum of P45,541.66 representing deficiency sales tax, forest charges, surcharges and
penalties. On November 8, 1954 another assessment was addressed for the payment of P456.12 as 25%
surcharge for discharging lumber without permit. The three assessments totalling P59,133.78 are the
subject matter. Defendant Corporation also protested, among others, the sending of final notices and
requested that they be countermanded or withheld. Finding no merit in the protests of the defendant
corporation, a warrant of distraint and levy was issued against it by the BIR on July 3, 1956. According
to the Record on Appeal, and as additionally stated also by the trial court, the original complaint filed
on September 5, 1958 prayed for the payment of only P13,695.96, and it was only in an amended
complaint filed on August 26, 1959 and admitted on September 23, 1959 that, for the first time, the
amount of P59,133.78 was judicially demanded to be paid.

Issue:
Whether or not the action was filed within the reglementary

129
Ruling: YES
The facts proven show that when Atty. Moya acknowledged receipt on December 28, 1954, on behalf
of appellant corporation, of the Bureau of Internal Revenue's assessments of September 1954 and
November 1954, requesting for a reinvestigation, he was informed that his request for investigation
would not be given due course unless his client complied within 10 days from February 11, 1955, the
date of the letter of the Bureau, with the provisions of Department Order No. 213, dated November 2,
1954, which required that requests for reinvestigation or reexamination of tax assessments should be
made in writing and under oath of the taxpayer concerned, specifying the grounds relied upon and
accompanied by the documents relied upon as well as by the payment of one-half of the total
assessments, plus a bond to guarantee payment of the balance, but appellants failed to comply.
Respondent, by its own omission, made it impossible for the BIR to act on its motion for
reconsideration. Mere filing of such a motion does not suspend the running of the period for the
collection of the tax, which implies that any assessment made by the Bureau is supposed to be final and
executory, insofar as the taxpayer is concerned.

G.R. No. L-19727 May 20, 1965

THE COMMISSIONER OF INTERNAL REVENUE, petitioner,


vs.
PHOENIX ASSURANCE CO., LTD., respondent.

-----------------------------

G.R. No. L-19903 May 20, 1965

PHOENIX ASSURANCE, CO., LTD., petitioner,


vs.
COMMISSIONER OF INTERNAL REVENUE, respondent.

Office of the Solicitor General for petitioner-respondent Commissioner of Internal Revenue.


Sycip, Salazar, Luna & Associates and A. S. Monzon, B. V. Abela & J. M. Castillo for respondent-
petitioner Phoenix Assurance Co., Ltd.

BENGZON, J.P., J.:

From a judgment of the Court of Tax Appeals in C.T.A. Cases Nos. 305 and 543, consolidated and
jointly heard therein, these two appeals were taken. Since they involve the same facts and interrelated
issues, the appeals are herein decided together.

Phoenix Assurance Co., Ltd., a foreign insurance corporation organized under the laws of Great
Britain, is licensed to do business in the Philippines with head office in London. Through its head
office, it entered in London into worldwide reinsurance treaties with various foreign insurance
130
companies. It agree to cede a portion of premiums received on original insurances underwritten by its
head office, subsidiaries, and branch offices throughout the world, in consideration for assumption by
the foreign insurance companies of an equivalent portion of the liability from such original
insurances.1äwphï1.ñët

Pursuant to such reinsurance treaties, Phoenix Assurance Co., Ltd., ceded portions of the premiums it
earned from its underwriting business in the Philippines, as follows:

Year Amount Ceded


1952 P316,526.75
1953 P246,082.04
1954 P203,384.69

upon which the Commissioner of Internal Revenue, by letter of May 6, 1958, assessed the following
withholding tax:

Year Withholding Tax


1952 P 75,966.42
1953 59,059.68
1954 48,812.32

Total P183,838.42
=============

On April 1, 1951, Phoenix Assurance Co., Ltd. filed its Philippine income tax return for 1950, claiming
therein, among others, a deduction of P37,147.04 as net addition to marine insurance reserve equivalent
to 40% of the gross marine insurance premiums received during the year. The Commissioner of
Internal Revenue disallowed P11,772.57 of such claim for deduction and subsequently assessed against
Phoenix Assurance Co., Ltd. the sum of P1,884.00 as deficiency income tax. The disallowance resulted
from the fixing by the Commissioner of the net addition to the marine insurance reserve at 100% of the
marine insurance premiums received during the last three months of the year. The Commissioner
assumed that "ninety and third, days are approximately the length of time required before shipments
reach their destination or before claims are received by the insurance companies."

On April 1, 1953, Phoenix Assurance Co., Ltd. filed its Philippine income tax return for 1952,
declaring therein a deduction from gross income of P35,912.25 as part of the head office expenses
incurred for its Philippine business, computed at 5% on its gross Philippine income.

On August 30, 1955 it amended its income tax return for 1952 by excluding from its gross income the
amount of P316,526.75 representing reinsurance premiums ceded to foreign reinsurers and further
eliminating deductions corresponding to the coded premiums. The amended return showed an income
131
tax due in the amount of P2,502.00. The Commissioner of Internal Revenue disallowed P15,826.35 of
the claimed deduction for head office expenses and assessed a deficiency tax of P5,667.00 on July 24,
1958.

On April 30, 1954, Phoenix Assurance Co., Ltd. filed its Philippine income tax return for 1953 and
claimed therein a deduction from gross income of P33,070.88 as head office expenses allocable to its
Philippine business, equivalent to 5%, of its gross Philippine income. On August 30, 1955 it amended
its 1953 income tax return to exclude from its gross income the amount of P246,082.04 representing
reinsurance premiums ceded to foreign reinsurers. At the same time, it requested the refund of
P23,409.00 as overpaid income tax for 1953. To avoid the prescriptive period provided for in Section
306 of the Tax Code, it filed a petition for review on April 11, 1956 in the Court of Tax Appeals
praying for such refund. After verification of the amended income tax return the Commissioner of
Internal Revenue disallowed P12,304.10 of the deduction representing head office expenses allocable
to Philippine business thereby reducing the refundable amount to P20,180.00.

On April 29, 1955, Phoenix Assurance Co., Ltd. filed its Philippine income tax return for 1954
claiming therein, among others, a deduction from gross income of P99,624.75 as head office expenses
allocable to its Philippine business, computed at 5% of its gross Philippine income. It also excluded
from its gross income the amount of P203,384.69 representing reinsurance premiums ceded to foreign
reinsurers not doing business in the Philippines.

On August 1, 1958 the Bureau of Internal Revenue released the following assessment for deficiency
income tax for the years 1952 and 1954 against Phoenix Assurance Co., Ltd.:

1952
Net income per audited
P 12,511.61
return
Unallowable deduction & additional
income:
Overclaimed Head Office expenses:
Amount
P
claimed . . . . .
35,912.25
.......
Amount
allowed . . . . . 20,085.90 P 15,826.35
.......

Net income per


P 28,337.96
investigation

Tax due thereon P 5,667.00


===========

132
1954
Net income per audited P160,320.21
Unallowable deduction & additional
income:
Overclaimed Head Office expenses:
Amount
claimed . . . . . P29,624.73
.......
Amount
allowed . . . . . 19,455.50 10,16.23
.......

Net income per


P170,489.41
investigation

Tax due thereon P 39,737.00


Less: amount already
36,890.00
assessed

DEFICIENCY TAX DUE P 2,847.00


===========

The above assessment resulted from the disallowance of a portion of the deduction claimed by Phoenix
Assurance Co., Ltd. as head office expenses allocable to its business in the Philippines fixed by the
Commissioner at 5% of the net Philippine income instead of 5% of the gross Philippine income as
claimed in the returns.

Phoenix Assurance Co., Ltd. protested against the aforesaid assessments for withholding tax and
deficiency income tax. However, the Commissioner of Internal Revenue denied such protest.
Subsequently, Phoenix Assurance Co., Ltd. appealed to the Court of Tax Appeals. In a decision dated
February 14, 1962, the Court of Tax Appeals allowed in full the decision claimed by Phoenix
Assurance Co., Ltd. for 1950 as net addition to marine insurance reserve; determined the allowable
head office expenses allocable to Philippine business to be 5% of the net income in the Philippines;
declared the right of the Commissioner of Internal Revenue to assess deficiency income tax for 1952 to
have prescribed; absolved Phoenix Assurance Co., Ltd. from payment of the statutory penalties for
non-filing of withholding tax return; and, rendered the following judgment:

WHEREFORE, petitioner Phoenix Assurance Company, Ltd. is hereby ordered to pay the
Commissioner of Internal Revenue the respective amounts of P75,966.42, P59,059.68 and
P48,812.32, as withholding tax for the years 1952, 1953 and 1954, and P2,847.00 as income tax
for 1954, or the total sum of P186,685.42 within thirty (30) days from the date this decision
133
becomes final. Upon the other hand, the respondent Commissioner is ordered to refund to
petitioner the sum of P20,180.00 as overpaid income tax for 1953, which sum is to be deducted
from the total sum of P186,685.42 due as taxes.

If any amount of the tax is not paid within the time prescribed above, there shall be collected a
surcharge of 5% of the tax unpaid, plus interest at the rate of 1% a month from the date of
delinquency to the date of payment, provided that the maximum amount that may be collected as
interest shall not exceed the amount corresponding to a period of three (3) years. Without
pronouncement as to costs.

Phoenix Assurance Co., Ltd. and the Commissioner of Internal Revenue have appealed to this Court
raising the following issues: (1) Whether or not reinsurance premiums ceded to foreign reinsurers not
doing business in the Philippines pursuant to reinsurance contracts executed abroad are subject to
withholding tax; (2) Whether or not the right of the Commissioner of Internal Revenue to assess
deficiency income tax for the year 1952 against Phoenix Assurance Co., Ltd., has prescribed; (3)
Whether or not the deduction of claimed by the Phoenix Assurance Co., Ltd.as net addition to reserve
for the year 1950 is excessive; (4) Whether or not the deductions claimed by Phoenix Assurance Co.,
Ltd. for head office expenses allocable to Philippine business for the years 1952, 1953 and 1954 are
excessive.

The question of whether or not reinsurance premiums ceded to foreign reinsurers not doing business in
the Philippines pursuant to contracts executed abroad are income from sources within the Philippines
subject to withholding tax under Sections 53 and 54 of the Tax Code has already been resolved in the
affirmative in British Traders' Insurance Co., Ltd.v. Commisioner of Internal Revenue, L-20501, April
30, 1965. 1

We come to the issue of prescription. Phoenix Assurance Co., Ltd. filed its income tax return for 1952
on April 1, 1953 showing a loss of P199,583.93. It amended said return on August 30, 1955 reporting a
tax liability of P2,502.00. On July 24, 1958, after examination of the amended return, the
Commissioner of Internal Revenue assessed deficiency income tax in the sum of P5,667.00. The Court
of Tax Appeals found the right of the Commissioner of Internal Revenue barred by prescription, the
same having been exercised more than five years from the date the original return was filed. On the
other hand, the Commissioner of Internal Revenue insists that his right to issue the assessment has not
prescribed inasmuch as the same was availed of before the 5-year period provided for in Section 331 of
the Tax Code expired, counting the running of the period from August 30, 1955, the date when the
amended return was filed.

Section 331 of the Tax Code, which limits the right of the Commissioner of Internal Revenue to assess
income tax within five years from the Filipino of the income tax return, states:

SEC. 331. Period of limitation upon assessment and collection. — Except as provided in the
succeeding section internal revenue taxes shall be assessed within five years after the return was
filed, and no proceeding in court without assessment for the collection of such taxes shall be
begun after the expiration of such period. For the purposes of this section, a return filed before
the last day prescribed by law for the filing thereof shall be considered as filed on such last

134
day: Provided, That this limitation shall not apply to cases already investigated prior to the
approval of this Code.

The question is: Should the running of the prescriptive period commence from the filing of the original
or amended return?

The Court of Tax Appears that the original return was a complete return containing "information on
various items of income and deduction from which respondent may intelligently compute and
determine the tax liability of petitioner, hence, the prescriptive period should be counted from the filing
of said original return. On the other hand, the Commissioner of Internal Revenue maintains that:

"... the deficiency income tax in question could not possibly be determined, or assessed, on the
basis of the original return filed on April 1, 1953, for considering that the declared loss
amounted to P199,583.93, the mere disallowance of part of the head office expenses could not
probably result in said loss being completely wiped out and Phoenix being liable to deficiency
tax. Not until the amended return was filed on August 30, 1955 could the Commissioner assess
the deficiency income tax in question."

Accordingly, he would wish to press for the counting of the prescriptive period from the filing of the
amended return.

To our mind, the Commissioner's view should be sustained. The changes and alterations embodied in
the amended income tax return consisted of the exclusion of reinsurance premiums received from
domestic insurance companies by Phoenix Assurance Co., Ltd.'s London head office, reinsurance
premiums ceded to foreign reinsurers not doing business in the Philippines and various items of
deduction attributable to such excluded reinsurance premiums thereby substantially modifying the
original return. Furthermore, although the deduction for head office expenses allocable to Philippine
business, whose disallowance gave rise to the deficiency tax, was claimed also in the original return,
the Commissioner could not have possibly determined a deficiency tax thereunder because Phoenix
Assurance Co., Ltd. declared a loss of P199,583.93 therein which would have more than offset such
disallowance of P15,826.35. Considering that the deficiency assessment was based on the amended
return which, as aforestated, is substantially different from the original return, the period of limitation
of the right to issue the same should be counted from the filing of the amended income tax return. From
August 30, 1955, when the amended return was filed, to July 24, 1958, when the deficiency assessment
was issued, less than five years elapsed. The right of the Commissioner to assess the deficiency tax on
such amended return has not prescribed.

To strengthen our opinion, we believe that to hold otherwise, we would be paving the way for
taxpayers to evade the payment of taxes by simply reporting in their original return heavy losses and
amending the same more than five years later when the Commissioner of Internal Revenue has lost his
authority to assess the proper tax thereunder. The object of the Tax Code is to impose taxes for the
needs of the Government, not to enhance tax avoidance to its prejudice.

We next consider Phoenix Assurance Co., Ltd.'s claim for deduction of P37,147.04 for 1950
representing net addition to reserve computed at 40% of the marine insurance premiums received

135
during the year. Treating said said deduction to be excessive, the Commissioner of Internal Revenue
reduced the same to P25,374.47 which is equivalent to 100% of all marine insurance premiums
received during the last months of the year.

Paragraph (a) of Section 32 of the Tax Code states:

SEC. 32. Special provisions regarding income and deductions of insurance companies, whether
domestic or foreign. — (a) Special deductions allowed to insurance companies. — In the case
of insurance companies, except domestic life insurance companies and foreign life insurance
companies doing business in the Philippines, the net additions, if any, required by law to be
made within the year to reserve funds and the sums other than dividends paid within the year on
policy and annuity contracts may be deducted from their gross income: Provided, however, That
the released reserve be treated as income for the year of release.

Section 186 of the Insurance Law requires the setting up of reserves for liability on marine insurance:

SEC. 186. ... Provided, That for marine risks the insuring company shall be required to charge as
the liability for reinsurance fifty per centum of the premiums written in the policies upon yearly
risks, and the full premiums written in the policies upon all other marine risks not
terminated (Emphasis supplied.)

The reserve required for marine insurance is determined on two bases: 50% of premiums under policies
on yearly risks and 100% of premiums under policies of marine risks not terminated during the year.
Section 32 (a) of the Tax Code quoted above allows the full amount of such reserve to be deducted
from gross income.

It may be noteworthy to observe that the formulas for determining the marine reserve employed by
Phoenix Assurance Co., Ltd. and the Commissioner of Internal Revenue — 40% of premiums received
during the year and 100% of premiums received during the last three months of the year, respectively
— do not comply with Section 186. Said determination runs short of the requirement. For purposes of
the Insurance Law, this Court therefore cannot countenance the same. The reserve called for in Section
186 is a safeguard to the general public and should be strictly followed not only because it is an express
provision but also as a matter of public policy. However, for income tax purposes a taxpayer is free to
deduct from its gross income a lesser amount, or not to claim any deduction at all. What is prohibited
by the income tax law is to claim a deduction beyond the amount authorized therein.

Phoenix Assurance Co., Ltd.'s claim for deduction of P37,147.04 being less than the amount required
in Section 186 of the Insurance Law, the same cannot be and is not excessive, and should therefore be
fully allowed. *

We come now to the controversy on the taxpayer's claim for deduction on head office expenses
incurred during 1952, 1953, and 1954 allocable to its Philippine business computed at 5% of its gross
income in the Philippines The Commissioner of Internal Revenue redetermined such deduction at 5%
on Phoenix Assurance Co., Ltd's net incomethereby partially disallowing the latter's claim. The parties
are agreed as to the percentage — 5% — but differ as to the basis of computation. Phoenix Assurance

136
Co. Lt. insists that the 5% head office expenses be determined from the gross income, while the
Commissioner wants the computation to be made on the net income. What, therefore, needs to be
resolved is: Should the 5% be computed on the gross or net income?

The record shows that the gross income of Phoenix Assurance Co., Ltd. consists of income from its
Philippine business as well as reinsurance premiums received for its head office in London and
reinsurance premiums ceded to foreign reinsurance. Since the items of income not belonging to its
Philippine business are not taxable to its Philippine branch, they should be excluded in determining the
head office expenses allowable to said Philippine branch. This conclusion finds support in paragraph 2,
subsection (a), Section 30 of the Tax Code, quoted hereunder:

(2) Expenses allowable to non-resident alien individuals and foreign corporations. In the case
of a non-resident alien individual or a foreign corporation, the expenses deductible are the,
necessary expenses paid or incurred in carrying on any business or trade conducted within the
Philippines exclusively. (Emphasis supplied.)

Consequently, the deficiency assessments for 1952, 1953 and 1954, resulting from partial disallowance
of deduction representing head office expenses, are sustained.

Finally, the Commissioner of Internal Revenue assails the dispositive portion of the Tax Court's
decision limiting the maximum amount of interest collectible for deliquency of an amount
corresponding to a period of three years. He contends that since such limitation was incorporated into
Section 51 of the Tax Code by Republic Act 2343 which took effect only on June 20, 1959, it must not
be applied retroactively on withholding tax for the years 1952, 1953 and 1954.

The imposition of interest on unpaid taxes is one of the statutory penalties for tax delinquency, from
the payments of which the Court of Tax Appeals absolved the Phoenix Assurance Co., Ltd. on the
equitable ground that the latter's failure to pay the withholding tax was due to the Commissioner's
opinion that no withholding tax was due. Consequently, the taxpayer could be held liable for the
payment of statutory penalties only upon its failure to comply with the Tax Court's judgment rendered
on February 14. 1962, after Republic Act 2343 took effect. This part of the ruling of the lower court
ought not to be disturbed.

WHEREFORE, the decision appealed from is modified, Phoenix Assurance Co., Ltd. is hereby ordered
to pay the Commissioner, of Internal Revenue the amount of P75,966.42, P59,059.68 and P48,812.32
as withholding tax for the years 1952, 1953 and 1954, respectively, and the sums of P5,667.00 and
P2,847.00 as income tax for 1952 and 1954 or a total of P192,352.42. The Commissioner of Internal
Revenue is ordered to refund to Phoenix Assurance Co., Ltd. the amount of P20,180.00 as overpaid
income tax for 1953, which should be deducted from the amount of P192,352.42.

If the amount of P192,352.42 or a portion thereof is not paid within thirty (30) days from the date this
judgment becomes final, there should be collected a surcharge and interest as provided for in Section
51(c) (2) of the Tax Code. No costs. It is so ordered.

NO DIGEST

137
G.R. No. L-20601 February 28, 1966

BUTUAN SAWMILL, INC., petitioner,


vs.
HON. COURT OF TAX APPEALS, ET AL., respondents.

David G. Nitafan for the petitioner.


Office of the Solicitor General for the respondents.

REYES, J.B.L., J.:

Appeal from a decision of the Court of Tax Appeals, in its CTA Case No. 965, ordering petitioner
herein, Butuan Sawmill, Inc., to pay respondent Commissioner of Internal Revenue the sum of
P36,107.74 as deficiency sales tax and surcharge due on its sales of logs to buyers in Japan from
January 31, 1951 to June 8, 1953.

The facts, as found and stated by the lower court in its decision, are in full accord with the evidences
presented therein; hence, we quote them hereunder:

. . . that during the period from January 31, 1951 to June 8, 1953, it sold logs to Japanese firms
at prices FOB Vessel Magallanes, Agusan (in some cases FOB Vessel, Nasipit, also in Agusan);
that the FOB prices included costs of loading, wharfage stevedoring and other costs in the
Philippines; that the quality, quantity and measurement specifications of the logs were certified
by the Bureau of Forestry; that the freight was paid by the Japanese buyers; and the payments of
the logs were effected by means of irrevocable letters of credit in favor of petitioner and payable
through the Philippine National Bank or any other bank named by it.

Upon investigation by the Bureau of Internal Revenue, it was ascertained that no sales tax return
was filed by the petitioner and neither did it pay the corresponding tax on the sales. On the basis
of agent Antonio Mole's report dated September 17, 1957, respondent, on August 27, 1958,
determined against petitioner the sum of P40,004.01 representing sales tax, surcharge and
compromise penalty on its sales [tax, surcharge and compromise penalty on its sales] of logs
from January 1951 to June 1953 pursuant to Sections 183, 186 and 209 of the National Internal
Revenue Code (Exhibit "E", p. 14, CTA rec. & p. 14, BIR rec.). And in consequence of a
reinvestigation, respondent, on November 6, 1958, amended the amount of the previous
assessment to P38,917.74 (Exh. "F", p. 52, BIR rec.). Subsequent requests for reconsideration of
the amended assessment having been denied (Exh. "G", p. 55, BIR rec.; Exh. "H", pp. 75-76,
BIR rec.: Exh. "I", pp. 79-80, BIR rec.; Exh. "J", p. 81, BIR rec.), petitioner filed the instant
petition for review on November 7, 1960.

On the bases of the above-quoted findings and circumstances, the lower court upheld the legality and
correctness of the amended assessment of the sales tax and surcharge, ruling that the sales in question,
in the light of our previous decisions1, were domestic or "local" sales, and, therefore, subject to sales
tax under the provision of section 186 of the Tax Code, as amended by Republic Acts Nos. 558 and
138
594; and that the assessment thereof was made well within the ten-year period prescribed by Section
332(a) of the same Code, since petitioner herein omitted to file its sales tax returns for the years 1951,
1952 and 1953, and this omission was discovered only on September 17, 1957. The imposition of the
compromise penalty was, however, eliminated therefrom for want of agreement between the taxpayer
and the Collector (now Commissioner) of Internal Revenue. A motion to reconsider said decision
having been denied, petitioner herein interposed the present appeal before this Court.

The issues presented in this appeal are: whether or not petitioner herein is liable to pay the 5% sales tax
as then prescribed by Section 186 of the Tax Code on its sales of logs to the Japanese buyers; and
whether or not the assessment thereof was made within the prescriptive period provided by law
therefor.1äwphï1.ñët

On the first issue, petitioner herein insists that the circumstances enumerated in the above finding,
which this Court had, in previous decisions (Cf. footnote [1]), considered as determinative of the place
of transfer of ownership of the logs sold, for purposes of taxation, are not in themselves evidentiary
indications to show that the parties intended the title of the logs to pass to the Japanese buyers in Japan.
Thus, it points out that the "FOB" feature of the sales contract was made only to fix its price and not to
fix the place of delivery; that the requirement of certification of quality, quantity, and measurement
specifications of the logs by local authorities was done to comply with local laws, rules, and
regulations, and was not a part of the sales arrangement; that the payment of freight by the Japanese
buyers is not an uncommon feature of "FOB" shipments; and that the payment of prices by means of
irrevocable letters of credit is but a common established business practice to secure payment of the
price to the seller. It also insists that, even assuming that the "FOB" feature of the disputed sales
determines the situs of transfer of ownership, the same is merely a prima facie presumption which
yields to contrary proof such as that the logs were made deliverable to the "order of the shipper" and
the logs were shipped at the risk of the shipper, which circumstances, if considered, would negate the
above implications. Hence, petitioner herein contends that the disputed sales were consummated in
Japan, and, therefore, not subject to the taxing jurisdiction of our Government.

The above contentions of petitioner are devoid of merit. In a decided case with practically identical set
of facts obtaining in the case at bar, this Court declared:

. . . it is admitted that the agreed price was "F.O.B. Agusan", thus indicating, although prima
facie, that the parties intended the title to pass to the buyer upon delivery of the logs in Agusan;
on board the vessels that took the goods to Japan. Moreover, said prima facie proof was
bolstered up by the following circumstances, namely:

1. Irrevocable letters of credit were opened by the Japanese buyers in favor of the petitioners.

2. Payment of freight charges of every shipment by the Japanese buyers.

3. The Japanese buyers chartered the ships that carried the logs they purchased from the
Philippines to Japan.

139
4. The Japanese buyers insured the shipment of logs and collected the insurance coverage in
case of loss in transit.

5. The petitioner collected the purchase price of every shipment of logs by surrendering the
covering letter of credit, bill of lading, which was indorsed in blank, tally sheet, invoice and
export entry, to the corresponding bank in Manila of the Japanese agent bank with whom the
Japanese buyers opened letters of credit.

6. In case of natural defects in logs shipped to the buyers discovered in Japan, instead of
returning such defective logs, accepted them, but were granted a corresponding credit based on
the contract price.

7. The logs purchased by the Japanese buyers were measured by a representative of the Director
of Forestry and such measurement was final, thereby making the Government of the Philippines
a sort of agent of the Japanese buyers.

Upon the foregoing facts and authority of Bislig (Bay) Lumber Co., Inc. vs. Collector of Internal
Revenue, G.R. No. L-13186 (January 28, 1961), Misamis Lumber Co., Inc. vs. Collector of Internal
Revenue (56 Off. Gaz. 517) andWestern Mindanao Lumber Development Co., Inc. vs. Court of Tax
Appeals, et al. (G.R. No. L-11710, June 30, 1958), it is clear that said export sales had been
consummated in the Philippines and were, accordingly, subject to sales tax therein." (Taligaman
Lumber Co., Inc. vs. Collector of Internal Revenue, G.R. No. L-15716, March 31, 1962).

With respect to petitioner's contention that there are proofs to rebut the prima facie finding and
circumstances that the disputed sales were consummated here in the Philippines, we find that the
allegation is not borne out by the law or the evidence.

That the specification in the bill of lading to the effect that the goods are deliverable to the order of the
seller or his agent does not necessarily negate the passing of title to the goods upon delivery to the
carrier is clear from the second part of paragraph 2 of Article 1503 of the Civil Code of the Philippines
(which appellant's counsel improperly omits from his citation):

Where goods are shipped, and by the bill of lading the goods are deliverable to the seller or his
agent, or to the order of the seller or of his agent, the seller thereby reserves the ownership in the
goods. But, if except for the form of the bill of lading, the ownership would have passed to the
buyer on shipment of the goods, the sellers's property in the goods shall be deemed to be only
for the purpose of securing performance by the buyer of his obligations under the contract.

Moreover, it has been "a settled rule that in petitions to review decisions of the Court of Tax Appeals,
only questions of law may be raised and may be passed upon by this Court" (Gutierrez vs. Court of Tax
Appeals & Collector of Internal Revenue vs. Gutierrez, G.R. Nos. L-7938 & L-9771, May 21, 1957,
cited in Sanchez vs. Commissioner of Customs, G.R. No. L-8556, September 30, 1957); and it having
been found that there is no proof to substantiate the foregoing contention of petitioner, the same should
also be ruled as devoid of merit.

140
On the second issue, petitioner avers that the filing of its income tax returns, wherein the proceeds of
the disputed sales were declared, is substantial compliance with the requirement of filing a sales tax
return, and, if there should be deemed a return filed, Section 331, and not Section 332(a), of the Tax
Code providing for a five-year prescriptive period within which to make an assessment and collection
of the tax in question from the time the return was deemed filed, should be applied to the case at bar.
Since petitioner filed its income tax returns for the years 1951, 1952 and 1953, and the assessment was
made in 1957 only, it further contends that the assessment of the sales tax corresponding to the years
1951 and 1952 has already prescribed for having been made outside the five-year period prescribed in
Section 331 of the Tax Code and should, therefore, be deducted from the assessment of the deficiency
sales tax made by respondent.

The above contention has already been raised and rejected as not meritorious in a previous case
decided by this Court. Thus, we held that an income tax return cannot be considered as a return for
compensating tax for purposes of computing the period of prescription under Section 331 of the Tax
Code, and that the taxpayer must file a return for the particular tax required by law in order to avail
himself of the benefits of Section 331 of the Tax Code; otherwise, if he does not file a return, an
assessment may be made within the time stated in Section 332(a) of the same Code (Bisaya Land
Transportation Co., Inc. vs. Collector of Internal Revenue & Collector of Internal Revenue vs. Bisaya
Land Transportation Co., Inc., G.R. Nos. L-12100 & L-11812, May 29, 1959). The principle
enunciated in this last cited case is applicable by analogy to the case at bar.

It being undisputed that petitioner failed to file a return for the disputed sales corresponding to the
years 1951, 1952 and 1953, and this omission was discovered only on September 17, 1957, and that
under Section 332(a) of the Tax Code assessment thereof may be made within ten (10) years from and
after the discovery of the omission to file the return, it is evident that the lower court correctly held that
the assessment and collection of the sales tax in question has not yet prescribed.

Wherefore, the decision appealed from should be, as it is hereby affirmed, with costs against petitioner.

DIGEST 1:

FACTS:
The Petitioner was granted a legislative franchise under RA 399 for an electric light, heat, and power
system in Butuan and Cabadbaran, Agusan, together with the issuance of a certificate of public
convenience and necessity by the Public Service Commission. However, the City of Butuan issued
Ordinances numbered 11, 131 and 148 imposing a 2% tax on the gross sales or receipts of any business
operated in the city. ButuanSawmill, Inc. questioned the validity of the taxing ordinance which is
deemed to have impaired the obligation of contract thereby depriving the Petitioner of property without
due process of law. On the other hand, Respondent maintained that it was vested with the “power to
provide for the levy and collection of taxes for general and special purposes” as stipulated in its charter
which was granted in 1950.

ISSUE:
W/N the inclusion of the franchise business of Petitioners falls within the coverage of the taxing
ordinances pursuant to the city’s power of taxation.
141
HELD:
No. the inclusion of the franchise business of the Butuan Sawmill, Inc. by the City of Butuan is beyond
the broad power of taxation of the city under its charter. Neither could the latter’s power therein
granted be taken as an authority delegated to the city to amend or alter the franchise, considering the
absence of an express or specific grant of power to do so. Where there are two statutes, the earlier
special and the latter general – and the terms of the general are broad enough to include the matter
provided for in the special – the fact that one is special and the other is general creates a presumption
that the special is to be considered as a remaining exception to the general as a general law of the land,
while the other as the law of a particular case.

DIGEST 2:

Butuan Sawmill, Inc. v. Court of Tax Appeals


G.R. No L-20601 February 28, 1966

Facts:
The BIR, upon investigation, ascertained that no sales tax return was filed by the petitioner and neither
did it pay the corresponding tax on the logs it sold to Japanese firms. Respondent, on August 27, 1958,
determined against petitioner the sum of P40,004.01 representing sales tax, surcharge and compromise
penalty on its sales from January 1951 to June 1953 pursuant to Sections 183, 186 and 209 of the
National Internal Revenue Code. In consequence of a reinvestigation, respondent, on November 6,
1958, amended the amount of the previous assessment to P38,917.74 Subsequent requests for
reconsideration of the amended assessment having been denied prompting petitioner to file the instant
petition for review on November 7, 1960. The lower court upheld the legality and correctness of the
amended assessment of the sales tax and surcharge, ruling that the sales in question were domestic or
"local" sales, and, therefore, subject to sales tax under the section 186 of the Tax Code, and that the
assessment thereof was made well within the ten-year period prescribed by Section 332(a) of the same
Code, since petitioner omitted to file its sales tax returns for the years 1951, 1952 and 1953, and this
omission was discovered only on September 17, 1957.

Issue:
Whether or not the assessment was made within the prescriptive period

Ruling: YES
The petitioner failed to file a return for the disputed sales corresponding to the years 1951, 1952 and
1953, and this omission was discovered only on September 17, 1957, and that under Section 332(a) of
the Tax Code assessment thereof may be made within 10 years from and after the discovery of the
omission to file the return. According to a previous case decided by the court, an income tax return
cannot be considered as a return for compensating tax for purposes of computing the period of
prescription under Section 331 of the Tax Code, and that the taxpayer must file a return for the
particular tax required by law in order to avail himself of the benefits of Section 331 of the Tax Code;
otherwise, if he does not file a return, an assessment may be made within the time stated in Section
332(a) of the same Code.

142
G.R. No. L-29485 March 31, 1976

COMMISSIONER OF INTERNAL REVENUE, petitioner,


vs.
AYALA SECURITIES CORPORATION and THE HONORABLE COURT OF TAX
APPEALS, respondents.

Solicitor General Felix V. Makasiar, Assistant Solicitor General Isidro C. Borromeo, Solicitor Lolita
O. Gal-lang and Special Attorney Salvador D. David for petitioner.

B. V. Abela M. C. Gutierrez, J. U. Ong and F.J. Malate, Jr. for respondents.

ESGUERRA, J.:

Appeal from the decision of the Court of Tax Appeals dated June 20, 1968, in its CTA Case No. 1346,
cancelling and declaring of no force and effect the assessment made by the petitioner, Commissioner of
Internal Revenue, against the accumulated surplus of the respondent, Ayala Securities Corporation.

The factual background of the case is as follows:

On November 29, 1955, respondent Ayala Securities Corporation, a domestic corporation organized
and existing under the laws of the Philippines, filed its income tax returns with the office of the
petitioner for its fiscal year which ended on September 30, 1955. Attached to its income tax return was
the audited financial statements of the respondent corporation as of September 30, 1955, showing a
surplus of P2,758,442.37. The income tax due on the return of the respondent corporation was duly
paid for within the time prescribed by law.

In a letter dated February 21, 1961, petitioner advised the respondent corporation of the assessment of
P758.687.04 on its accumulated surplus reflected on its income tax return for the fiscal year which
ended September 30, 1955 (Exit. D). The respondent corporation, on the other hand, in a letter dated
April 19, 1961, protested against the assessment on its retained and accumulated surplus pertaining to
the taxable year 1955 and sought reconsideration thereof for the reasons (1) that the accumulation of
the surplus was for a bona fide business purpose and not to avoid the imposition of income tax on the
individual shareholders, and (2) that the said assessment was issued beyond the five-year prescriptive
period (Exh. E).

On May 30, 1961, petitioner wrote respondent corporation's auditing and accounting firm with the
"advise that your request for reconsideration will be the subject matter of further reinvestigation and a
thorough analysis of the issues involved conditioned, however, upon the execution of your client of the
enclosed form for waiver of the defense of prescription". (Exh. F) However, respondent corporation did

143
not execute the requested waiver of the statute of limitations, considering its claim that the assessment
in question had already prescribed.

On February 21, 1963, respondent corporation received a letter dated February 18, 1963, from the
Chief, Manila Examiners, of the Office of the herein petitioner, calling the attention of the respondent
corporation to its outstanding and unpaid tax in the amount of P708,687.04 and thereby requesting for
the payment of the said amount within five (5) days from receipt of the said letter (Exh. G). Believing
the aforesaid letter to be a denial of its protest, the herein respondent corporation filed with the Court of
Tax Appeals a Petition for Review of the assessment, docketed as CTA Case No. 1346.

Respondent corporation in its Petition for Review alleges that the assessment made by petitioner
Commissioner of Internal Revenue is illegal and invalid considering that (1) the assessment in
question, having been issued only on February 21, 1961, and received by the respondent corporation on
March 22, 1961, the same was issued beyond the five-year period from the date of the filing of
respondent corporations income tax return November 29, 1955, and, therefore, petitioner's right to
make the assessment has already prescribed, pursuant to the provision of Section 331 of the National
Internal Revenue Code; and (2) the respondent corporation's accumulation of surplus for the taxable
year 1955 was not improper, considering that the retention of such surplus was intended for legitimate
business purposes and was not availed of by the corporation to prevent the imposition of the income tax
upon its shareholders.

Petitioner in his answer alleged that the assessment made by his office on the accumulated surplus of
the corporation as reflected on its income tax return for the taxable year 1955 has not as yet prescribed
and, further, that the respondent corporation's accumulation of surplus for the taxable year 1955 was
improper as the retention of such surplus was availed of by the corporation to prevent the imposition of
the income tax upon the individual shareholders or members of the said corporation.

After trial the Court of Tax Appeals rendered its decision of June 20, 1968, the dispositive portion of
which is as follows:

WHEREFORE, the decision of the respondent Commissioner of Internal Revenue


assessing petitioner the amount of P758,687.04 as 25 surtax and interest is reversed.
Accordingly, said assessment of respondent for 1955 is hereby cancelled and declared of
no force and effect. Without pronouncement as to costs.

From this decision, the Commissioner of Internal Revenue interposed this appeal.

Petitioner maintains that respondent Court of Tax Appeals erred in holding that the letter dated
February 18, 1963, (Exh. G) is a denial of the private respondent corporation's protest against the
assessment, and as such, is a decision contemplated under the provisions of Sections 7 and 11 of
Republic Act No. 1125. Petitioner contends that the letter dated February 18, 1963, is merely an
ordinary office letter designed to remind delinquent taxpayers of their obligations to pay their taxes to
the Government and, certainly, not a decision on a disputed or protested assessment contemplated
under Section 7(1) of R.A. 1125.

144
Petitioner likewise maintains that the respondent Court of Tax Appeals erred in holding that the
assessment of P758,687.04 as surtax on private respondent corporation's unreasonably accumulated
profits or surplus had already prescribed. Petitioner further contends that the applicable provision of
law to this case is Section 332 (a) of the National Internal Revenue Code which provides for a ten (10)
year prescriptive period of assessment, and not Section 331 thereof as held by the Tax Court which
provides a period of limitation of assessment for five (5) years only after the filing of the return.
Petitioner's theory, therefore, is to the effect that since the Corporate income tax return in question was
filed on, November 29, 1955, and the assessment thereto was issued on February 21, 1961, said
assessment is not barred by prescription as the same was made very well within the ten (10) year period
allowed by law.

Petitioner also maintains that the respondent Court of Tax Appeals erred in not deciding the issue as to
whether or not the accumulated profits or surplus is indispensable to the business operations of the
private respondent corporation. It is the contention of the petitioner that the accumulation of profits or
surplus was resorted to by the respondent corporation in order to avoid the payment of taxes by its
stockholders or members, and was not availed of in order to meet the reasonable needs of its business
operations.

The legal issues for resolution by this Court in this case are: (1) Whether or not the instant case falls
within the jurisdiction of the respondent Court of Tax Appeals; (2) Whether or not the applicable
provision of law to this case is Section 331 of the National Internal Revenue Code, which provides for
a five-year period of prescription of assessment from the filing of the return, or Section 332(a) of the
same Code which provides for a ten-year period of limitation for the same purpose; and (3) Whether or
not the respondent Court of Tax Appeals committed a reversible error in not making any ruling on the
reasonableness or unreasonableness of the accumulated profits or surplus in question of the private
respondent corporation.

It is to be noted that the respondent Court of Tax Appeals is a court of special appellate jurisdiction
created under R. A. No. 1125. Thus under Section 7 (1), R. A. 1125, the Court of Tax Appeals
exercises exclusive appellate jurisdiction to review by appeal "decisions of the Collector of Internal
Revenue in cases involving disputed assessments, refunds of internal revenue taxes, fees or other
charges, penalties imposed in relation thereto, or other matters arising under the National Internal
Revenue Code or other law or part of law administered by the Bureau of Internal Revenue".

The letter of February 18, 1963 (Exh. G), in the view of the Court, is tantamount to a denial of the
reconsideration or protest of the respondent corporation on the assessment made by the petitioner,
considering that the said letter is in itself a reiteration of the demand by the Bureau of Internal Revenue
for the settlement of the assessment already made, and for the immediate payment of the sum of P758,
687.04 in spite of the vehement protest of the respondent corporation on April 21, 1961. This certainly
is a clear indication of the firm stand of petitioner against the reconsideration of the disputed
assessment in view of the continued refusal of the respondent corporation to execute the waiver of the
period of limitation upon the assessment in question.

145
This being so, the said letter amounts to a decision on a disputed or protested assessment and,
therefore, the court a quo did not err in taking cognizance of this case.

II

On the issue of whether Sec. 331 or See. 332(a) of the National Internal Revenue Code should apply to
this case, there is no iota of evidence presented by the petitioner as to any fraud or falsity on the return
with intent to evade payment of tax, not even in the income tax assessment (Exh. 5) nor in the letter-
decision of February 18, 1963 (Exh. G), nor in his answer to the petition for review. Petitioner merely
relies on the provisions of Sec 25 of the National Internal Revenue Code, violation of which, according
to Petitioner, presupposes the existence of fraud. But this is begging the question and We do not
subscribe to the view of the petitioner.

Fraud is a question of fact and the circumstances constituting fraud must be alleged and proved in the
court below. The finding of the trial court as to its existence and non- existence is final and cannot be
reviewed here unless clearly shown to be erroneous (Republic of the Philippines vs. Ker & Company,
Ltd., L-21609, Sept. 29, 1966, 18 SCRA 207; Commissioner of Internal Revenue vs. Lilia Yusay
Gonzales and the Court of Tax Appeals,
L-19495, Nov. 24, 1966, 18 SCRA 757). Fraud is never lightly to be presumed because it is serious
charge (Yutivo Sons Hardware Company vs. Court of Tax Appeals and Collector of Internal Revenue,
L-13203, January 28,1961, 1 SCRA 160).

The applicable provision of law in this case is Section 331 of the National Internal Revenue Code, to
wit:

SEC. 331. Period of limitation upon assessment and collection. — Except as provided in
the succeeding section, internal revenue taxes shall be assessed within five years after the
return was filed, and no proceeding in court without assessment for the collection of such
taxes shall be begun after the expiration of such period. For the purposes of this section, a
return filed before the last day prescribed by law for the filing thereof shall be considered
as filed on such last day: Provided, That this limitation shall not apply to cases already
investigated prior to the approval of this Code.

Under Section 46(d) of the National Internal Revenue Code, the Ayala Securities Corporation
designated September 30, 1955, as the last day of the closing of its fiscal year, and under Section 46(b)
the income tax returns for the said corporation shall be filed on or before the fifteenth (15th) day of the
fourth (4th) month following the close of its fiscal year. The Ayala Securities Corporation could,
therefore, file its income tax returns on or before January 15, 1956. The assessment by the
Commissioner of Internal Revenue shall be made within five (5) years from January 15, 1956, or not
later than January 15, 1961, in accordance with Section 331 of the National Internal Revenue Code
herein above-quoted. As the assessment issued on February 21, 1961, which was received by the Ayala
Securities Corporation on March 22, 1961, was made beyond the five-year period prescribed under
Section 331 of said Code, the same was made after the prescriptive period had expired and, therefore,
was no longer binding on the Ayala Securities Corporation.

146
This Court is of the opinion that the respondent court committed no reversible error in not making any
ruling on the reasonableness or unreasonableness of the accumulated profits or surplus of the
respondent corporation. For this reason, We are of the view that after reaching the conclusion that the
right of the Commissioner of Internal Revenue to assess the 25% surtax had already prescribed under
Section 331 of the National Internal Revenue Code, to delve further into the reasonableness or
unreasonableness of the accumulated profits or surplus of the respondent corporation for the fiscal year
ending September 30, 1955, will only be an exercise in futility.

WHEREFORE, the decision appealed from is hereby affirmed in toto.

Without special pronouncement as to costs.

SO ORDERED.

DIGEST:

Commissioner of Internal Revenue v Ayala Securities Corporation


Facts:
Ayala Securities Corp. (Ayala) failed to file returns of their accumulated surplus so Ayala was charged
with 25% surtax by the Commissioner of internal Revenue. The CTA (Court of Tax Appeals) reversed
the Commissioner’s decision and held that the assessment made against Ayala was beyond the 5-yr
prescriptive period as provided in section 331 of the National Internal Revenue Code. Commissioner
now files a motion for reconsideration of this decision. Ayala invokes the defense of prescription
against the right of the Commissioner to assess the surtax.

Issue:
Whether or not the right to assess and collect the 25% surtax has prescribed after five years.

Held:
No. There is no such time limit on the right of the Commissioner to assess the 25% surtax since there is
no express statutory provision limiting such right or providing for its prescription. Hence, the collection
of surtax is imprescriptible. The underlying purpose of the surtax is to avoid a situation where the
corporation unduly retains its surplus earnings instead of declaring and paying dividends to its
shareholders. SC reverses the ruling of the CTA.

G.R. No. 162852 December 16, 2004

PHILIPPINE JOURNALISTS, INC., petitioner,


vs.
COMMISSIONER OF INTERNAL REVENUE, respondent.

YNARES-SANTIAGO, J.:

147
This is a petition for review filed by Philippine Journalists, Incorporated (PJI) assailing the Decision1 of
the Court of Appeals dated August 5, 2003,2 which ordered petitioner to pay the assessed tax liability of
P111,291,214.46 and the Resolution3 dated March 31, 2004 which denied the Motion for
Reconsideration.

The case arose from the Annual Income Tax Return filed by petitioner for the calendar year ended
December 31, 1994 which presented a net income of P30,877,387.00 and the tax due of
P10,807,086.00. After deducting tax credits for the year, petitioner paid the amount of P10,247,384.00.

On August 10, 1995, Revenue District Office No. 33 of the Bureau of Internal Revenue (BIR) issued
Letter of Authority No. 871204 for Revenue Officer Federico de Vera, Jr. and Group Supervisor
Vivencio Gapasin to examine petitioner’s books of account and other accounting records for internal
revenue taxes for the period January 1, 1994 to December 31, 1994.

From the examination, the petitioner was told that there were deficiency taxes, inclusive of surcharges,
interest and compromise penalty in the following amounts:

Value Added P 229,527.90


Tax
Income Tax 125,002,892.95
Withholding 2,748,012.35
Tax
Total P
127,980,433.20

In a letter dated August 29, 1997, Revenue District Officer Jaime Concepcion invited petitioner to send
a representative to an informal conference on September 15, 1997 for an opportunity to object and
present documentary evidence relative to the proposed assessment. On September 22, 1997,
petitioner’s Comptroller, Lorenza Tolentino, executed a "Waiver of the Statute of Limitation Under the
National Internal Revenue Code (NIRC)".5 The document "waive[d] the running of the prescriptive
period provided by Sections 223 and 224 and other relevant provisions of the NIRC and consent[ed] to
the assessment and collection of taxes which may be found due after the examination at any time after
the lapse of the period of limitations fixed by said Sections 223 and 224 and other relevant provisions
of the NIRC, until the completion of the investigation".6

On July 2, 1998, Revenue Officer De Vera submitted his audit report recommending the issuance of an
assessment and finding that petitioner had deficiency taxes in the total amount of P136,952,408.97. On
October 5, 1998, the Assessment Division of the BIR issued Pre-Assessment Notices which informed
petitioner of the results of the investigation. Thus, BIR Revenue Region No. 6, Assessment
Division/Billing Section, issued Assessment/Demand No. 33-1-000757-947 on December 9, 1998
stating the following deficiency taxes, inclusive of interest and compromise penalty:

Income Tax P108,743,694.88


148
Value 184,299.20
Added Tax
Expanded 2,363,220.38
Withholding
Tax
Total P111,291,214.46

On March 16, 1999, a Preliminary Collection Letter was sent by Deputy Commissioner Romeo S.
Panganiban to the petitioner to pay the assessment within ten (10) days from receipt of the letter. On
November 10, 1999, a Final Notice Before Seizure8 was issued by the same deputy commissioner
giving the petitioner ten (10) days from receipt to pay. Petitioner received a copy of the final notice on
November 24, 1999. By letters dated November 26, 1999, petitioner asked to be clarified how the tax
liability of P111,291,214.46 was reached and requested an extension of thirty (30) days from receipt of
the clarification within which to reply.9

The BIR received a follow-up letter from the petitioner asserting that its (PJI) records do not show
receipt of Tax Assessment/Demand No. 33-1-000757-94.10 Petitioner also contested that the assessment
had no factual and legal basis. On March 28, 2000, a Warrant of Distraint and/or Levy No. 33-06-
04611 signed by Deputy Commissioner Romeo Panganiban for the BIR was received by the petitioner.

Petitioner filed a Petition for Review12 with the Court of Tax Appeals (CTA) which was amended on
May 12, 2000. Petitioner complains: (a) that no assessment or demand was received from the BIR; (b)
that the warrant of distraint and/or levy was without factual and legal bases as its issuance was
premature; (c) that the assessment, having been made beyond the 3-year prescriptive period, is null and
void; (d) that the issuance of the warrant without being given the opportunity to dispute the same
violates its right to due process; and (e) that the grave prejudice that will be sustained if the warrant is
enforced is enough basis for the issuance of the writ of preliminary injunction.

On May 14, 2002, the CTA rendered its decision,13 to wit:

As to whether or not the assessment notices were received by the petitioner, this Court rules in
the affirmative.

To disprove petitioner’s allegation of non-receipt of the aforesaid assessment notices,


respondent presented a certification issued by the Post Master of the Central Post Office, Manila
to the effect that Registered Letter No. 76134 sent by the BIR, Region No. 6, Manila on
December 15, 1998 addressed to Phil. Journalists, Inc. at Journal Bldg., Railroad St., Manila
was duly delivered to and received by a certain Alfonso Sanchez, Jr. (Authorized
Representative) on January 8, 1999. Respondent also showed proof that in claiming Registered
Letter No. 76134, Mr. Sanchez presented three identification cards, one of which is his company
ID with herein petitioner.

149
However, as to whether or not the Waiver of the Statute of Limitations is valid and binding on
the petitioner is another question. Since the subject assessments were issued beyond the three-
year prescriptive period, it becomes imperative on our part to rule first on the validity of the
waiver allegedly executed on September 22, 1997, for if this court finds the same to be
ineffective, then the assessments must necessarily fail.

After carefully examining the questioned Waiver of the Statute of Limitations, this Court
considers the same to be without any binding effect on the petitioner for the following reasons:

The waiver is an unlimited waiver. It does not contain a definite expiration date. Under RMO
No. 20-90, the phrase indicating the expiry date of the period agreed upon to assess/collect the
tax after the regular three-year period of prescription should be filled up…

Secondly, the waiver failed to state the date of acceptance by the Bureau which under the
aforequoted RMO should likewise be indicated…

Finally, petitioner was not furnished a copy of the waiver. It is to be noted that under RMO No.
20-90, the waiver must be executed in three (3) copies, the second copy of which is for the
taxpayer. It is likewise required that the fact of receipt by the taxpayer of his/her file copy be
indicated in the original copy. Again, respondent failed to comply.

It bears stressing that RMO No. 20-90 is directed to all concerned internal revenue officers. The
said RMO even provides that the procedures found therein should be strictly followed, under
pain of being administratively dealt with should non-compliance result to prescription of the
right to assess/collect…

Thus, finding the waiver executed by the petitioner on September 22, 1997 to be suffering from
legal infirmities, rendering the same invalid and ineffective, the Court finds
Assessment/Demand No. 33-1-000757-94 issued on December 5, 1998 to be time-barred.
Consequently, the Warrant of Distraint and/or Levy issued pursuant thereto is considered null
and void.

WHEREFORE, in view of all the foregoing, the instant Petition for Review is
hereby GRANTED. Accordingly, the deficiency income, value-added and expanded
withholding tax assessments issued by the respondent against the petitioner on December 9,
1998, in the total amount of P111,291,214.46 for the year 1994 are hereby
declared CANCELLED, WITHDRAWN and WITH NO FORCE AND EFFECT. Likewise,
Warrant of Distraint and/or Levy No. 33-06-046 is hereby declared NULL and VOID.

SO ORDERED.14
150
After the motion for reconsideration of the Commissioner of Internal Revenue was denied by the CTA
in a Resolution dated August 2, 2002, an appeal was filed with the Court of Appeals on August 12,
2002.

In its decision dated August 5, 2003, the Court of Appeals disagreed with the ruling of the CTA, to wit:

… The petition for review filed on 26 April 2000 with CTA was neither timely filed nor the
proper remedy. Only decisions of the BIR, denying the request for reconsideration or
reinvestigation may be appealed to the CTA. Mere assessment notices which have become final
after the lapse of the thirty (30)-day reglementary period are not appealable. Thus, the CTA
should not have entertained the petition at all.

… [T]he CTA found the waiver executed by Phil. Journalists to be invalid for the following
reasons: (1) it does not indicate a definite expiration date; (2) it does not state the date of
acceptance by the BIR; and (3) Phil. Journalist, the taxpayer, was not furnished a copy of the
waiver. These grounds are merely formal in nature. The date of acceptance by the BIR does not
categorically appear in the document but it states at the bottom page that the BIR "accepted and
agreed to:"…, followed by the signature of the BIR’s authorized representative. Although the
date of acceptance was not stated, the document was dated 22 September 1997. This date could
reasonably be understood as the same date of acceptance by the BIR since a different date was
not otherwise indicated. As to the allegation that Phil. Journalists was not furnished a copy of
the waiver, this requirement appears ridiculous. Phil. Journalists, through its comptroller,
Lorenza Tolentino, signed the waiver. Why would it need a copy of the document it knowingly
executed when the reason why copies are furnished to a party is to notify it of the existence of a
document, event or proceeding? …

As regards the need for a definite expiration date, this is the biggest flaw of the decision. The
period of prescription for the assessment of taxes may be extended provided that the extension
be made in writing and that it be made prior to the expiration of the period of prescription. These
are the requirements for a valid extension of the prescriptive period. To these requirements
provided by law, the memorandum order adds that the length of the extension be specified by
indicating its expiration date. This requirement could be reasonably construed from the rule on
extension of the prescriptive period. But this requirement does not apply in the instant case
because what we have here is not an extension of the prescriptive period but a waiver thereof.
These are two (2) very different things. What Phil. Journalists executed was a renunciation of its
right to invoke the defense of prescription. This is a valid waiver. When one waives the
prescriptive period, it is no longer necessary to indicate the length of the extension of the
prescriptive period since the person waiving may no longer use this defense.

WHEREFORE, the 02 August 2002 resolution and 14 May 2002 decision of the CTA are hereby
SET ASIDE. Respondent Phil. Journalists is ordered [to] pay its assessed tax liability of
P111,291,214.46.

151
SO ORDERED.15

Petitioner’s Motion for Reconsideration was denied in a Resolution dated March 31, 2004. Hence, this
appeal on the following assignment of errors:

I.

The Honorable Court of Appeals committed grave error in ruling that it is outside the
jurisdiction of the Court of Tax Appeals to entertain the Petition for Review filed by the herein
Petitioner at the CTA despite the fact that such case inevitably rests upon the validity of the
issuance by the BIR of warrants of distraint and levy contrary to the provisions of Section 7(1)
of Republic Act No. 1125.

II.

The Honorable Court of Appeals gravely erred when it ruled that failure to comply with the
provisions of Revenue Memorandum Order (RMO) No. 20-90 is merely a formal defect that
does not invalidate the waiver of the statute of limitations without stating the legal justification
for such conclusion. Such ruling totally disregarded the mandatory requirements of Section
222(b) of the Tax Code and its implementing regulation, RMO No. 20-90 which are substantive
in nature. The RMO provides that violation thereof subjects the erring officer to administrative
sanction. This directive shows that the RMO is not merely cover forms.

III.

The Honorable Court of Appeals gravely erred when it ruled that the assessment notices became
final and unappealable. The assessment issued is void and legally non-existent because the BIR
has no power to issue an assessment beyond the three-year prescriptive period where there is no
valid and binding waiver of the statute of limitation.

IV.

The Honorable Court of Appeals gravely erred when it held that the assessment in question has
became final and executory due to the failure of the Petitioner to protest the same. Respondent
had no power to issue an assessment beyond the three year period under the mandatory
provisions of Section 203 of the NIRC. Such assessment should be held void and non-existent,
otherwise, Section 203, an expression of a public policy, would be rendered useless and
nugatory. Besides, such right to assess cannot be validly granted after three years since it would
arise from a violation of the mandatory provisions of Section 203 and would go against the
vested right of the Petitioner to claim prescription of assessment.

V.

The Honorable Court of Appeals committed grave error when it HELD valid a defective waiver
by considering the latter a waiver of the right to invoke the defense of prescription rather than an
extension of the three year period of prescription (to make an assessment) as provided under
152
Section 222 in relation to Section 203 of the Tax Code, an interpretation that is contrary to law,
existing jurisprudence and outside of the purpose and intent for which they were enacted.16

We find merit in the appeal.

The first assigned error relates to the jurisdiction of the CTA over the issues in this case. The Court of
Appeals ruled that only decisions of the BIR denying a request for reconsideration or reinvestigation
may be appealed to the CTA. Since the petitioner did not file a request for reinvestigation or
reconsideration within thirty (30) days, the assessment notices became final and unappealable. The
petitioner now argue that the case was brought to the CTA because the warrant of distraint or levy was
illegally issued and that no assessment was issued because it was based on an invalid waiver of the
statutes of limitations.

We agree with petitioner. Section 7(1) of Republic Act No. 1125, the Act Creating the Court of Tax
Appeals, provides for the jurisdiction of that special court:

SEC. 7. Jurisdiction. – The Court of Tax Appeals shall exercise exclusive appellate jurisdiction
to review by appeal, as herein provided –

(1) Decisions of the Commissioner of Internal Revenue in cases involving disputed assessments,
refunds of internal revenue taxes, fees or other charges, penalties imposed in relation thereto,
or other matters arising under the National Internal Revenue Code or other laws or part of
law administered by the Bureau of Internal Revenue; (Emphasis supplied).

The appellate jurisdiction of the CTA is not limited to cases which involve decisions of the
Commissioner of Internal Revenue on matters relating to assessments or refunds. The second part of
the provision covers other cases that arise out of the NIRC or related laws administered by the Bureau
of Internal Revenue. The wording of the provision is clear and simple. It gives the CTA the jurisdiction
to determine if the warrant of distraint and levy issued by the BIR is valid and to rule if the Waiver of
Statute of Limitations was validly effected.

This is not the first case where the CTA validly ruled on issues that did not relate directly to a disputed
assessment or a claim for refund. In Pantoja v. David,17 we upheld the jurisdiction of the CTA to act on
a petition to invalidate and annul the distraint orders of the Commissioner of Internal Revenue. Also,
in Commissioner of Internal Revenue v. Court of Appeals,18 the decision of the CTA declaring several
waivers executed by the taxpayer as null and void, thus invalidating the assessments issued by the BIR,
was upheld by this Court.

The second and fifth assigned errors both focus on Revenue Memorandum Circular No. 20-90 (RMO
No. 20-90) on the requisites of a valid waiver of the statute of limitations. The Court of Appeals held
that the requirements and procedures laid down in the RMO are only formal in nature and did not
invalidate the waiver that was signed even if the requirements were not strictly observed.

The NIRC, under Sections 203 and 222,19 provides for a statute of limitations on the assessment and
collection of internal revenue taxes in order to safeguard the interest of the taxpayer against
unreasonable investigation.20Unreasonable investigation contemplates cases where the period for
153
assessment extends indefinitely because this deprives the taxpayer of the assurance that it will no
longer be subjected to further investigation for taxes after the expiration of a reasonable period of time.
As was held in Republic of the Phils. v. Ablaza:21

The law prescribing a limitation of actions for the collection of the income tax is beneficial both
to the Government and to its citizens; to the Government because tax officers would be obliged
to act promptly in the making of assessment, and to citizens because after the lapse of the period
of prescription citizens would have a feeling of security against unscrupulous tax agents who
will always find an excuse to inspect the books of taxpayers, not to determine the latter’s real
liability, but to take advantage of every opportunity to molest peaceful, law-abiding citizens.
Without such a legal defense taxpayers would furthermore be under obligation to always keep
their books and keep them open for inspection subject to harassment by unscrupulous tax
agents. The law on prescription being a remedial measure should be interpreted in a way
conducive to bringing about the beneficent purpose of affording protection to the taxpayer
within the contemplation of the Commission which recommend the approval of the
law. (Emphasis supplied)

RMO No. 20-90 implements these provisions of the NIRC relating to the period of prescription for the
assessment and collection of taxes. A cursory reading of the Order supports petitioner’s argument that
the RMO must be strictly followed, thus:

In the execution of said waiver, the following procedures should be followed:

1. The waiver must be in the form identified hereof. This form may be reproduced by the Office
concerned but there should be no deviation from such form. The phrase "but not after
__________ 19___" should be filled up…

2. …

Soon after the waiver is signed by the taxpayer, the Commissioner of Internal Revenue or the
revenue official authorized by him, as hereinafter provided, shall sign the waiver
indicating that the Bureau has accepted and agreed to the waiver. The date of such
acceptance by the Bureau should be indicated…

3. The following revenue officials are authorized to sign the waiver.

A. In the National Office

3. Commissioner For tax cases


involving more
than P1M

B. In the Regional Offices


154
1. The Revenue District Officer with respect to tax cases still pending
investigation and the period to assess is about to prescribe regardless of amount.

5. The foregoing procedures shall be strictly followed. Any revenue official


found not to have complied with this Order resulting in prescription of the
right to assess/collect shall be administratively dealt with. (Emphasis
supplied)22

A waiver of the statute of limitations under the NIRC, to a certain extent, is a derogation of the
taxpayers’ right to security against prolonged and unscrupulous investigations and must therefore be
carefully and strictly construed.23The waiver of the statute of limitations is not a waiver of the right to
invoke the defense of prescription as erroneously held by the Court of Appeals. It is an agreement
between the taxpayer and the BIR that the period to issue an assessment and collect the taxes due is
extended to a date certain. The waiver does not mean that the taxpayer relinquishes the right to invoke
prescription unequivocally particularly where the language of the document is equivocal. For the
purpose of safeguarding taxpayers from any unreasonable examination, investigation or assessment,
our tax law provides a statute of limitations in the collection of taxes. Thus, the law on prescription,
being a remedial measure, should be liberally construed in order to afford such protection. As a
corollary, the exceptions to the law on prescription should perforce be strictly construed.24 RMO No.
20-90 explains the rationale of a waiver:

... The phrase "but not after _________ 19___" should be filled up. This indicates the expiry
date of the period agreed upon to assess/collect the tax after the regular three-year period of
prescription. The period agreed upon shall constitute the time within which to effect the
assessment/collection of the tax in addition to the ordinary prescriptive period. (Emphasis
supplied)

As found by the CTA, the Waiver of Statute of Limitations, signed by petitioner’s comptroller on
September 22, 1997 is not valid and binding because it does not conform with the provisions of RMO
No. 20-90. It did not specify a definite agreed date between the BIR and petitioner, within which the
former may assess and collect revenue taxes. Thus, petitioner’s waiver became unlimited in time,
violating Section 222(b) of the NIRC.

The waiver is also defective from the government side because it was signed only by a revenue district
officer, not the Commissioner, as mandated by the NIRC and RMO No. 20-90. The waiver is not a
unilateral act by the taxpayer or the BIR, but is a bilateral agreement between two parties to extend the
period to a date certain. The conformity of the BIR must be made by either the Commissioner or the
Revenue District Officer. This case involves taxes amounting to more than One Million Pesos
(P1,000,000.00) and executed almost seven months before the expiration of the three-year prescription
period. For this, RMO No. 20-90 requires the Commissioner of Internal Revenue to sign for the BIR.

155
The case of Commissioner of Internal Revenue v. Court of Appeals,25 dealt with waivers that were not
signed by the Commissioner but were argued to have been given implied consent by the BIR. We
invalidated the subject waivers and ruled:

Petitioner’s submission is inaccurate…

The Court of Appeals itself also passed upon the validity of the waivers executed by Carnation,
observing thus:

We cannot go along with the petitioner’s theory. Section 319 of the Tax Code earlier
quoted is clear and explicit that the waiver of the five-year26 prescriptive period must be
in writing and signed by both the BIR Commissioner and the taxpayer.

Here, the three waivers signed by Carnation do not bear the written consent of the BIR
Commissioner as required by law.

We agree with the CTA in holding "these ‘waivers’ to be invalid and without any binding
effect on petitioner (Carnation) for the reason that there was no consent by the respondent
(Commissioner of Internal Revenue)."

For sure, no such written agreement concerning the said three waivers exists between the
petitioner and private respondent Carnation.

What is more, the waivers in question reveal that they are in no wise unequivocal, and therefore
necessitates for its binding effect the concurrence of the Commissioner of Internal
Revenue…. On this basis neither implied consent can be presumed nor can it be contended
that the waiver required under Sec. 319 of the Tax Code is one which is unilateral nor can
it be said that concurrence to such an agreement is a mere formality because it is the very
signatures of both the Commissioner of Internal Revenue and the taxpayer which give
birth to such a valid agreement.27 (Emphasis supplied)

The other defect noted in this case is the date of acceptance which makes it difficult to fix with
certainty if the waiver was actually agreed before the expiration of the three-year prescriptive period.
The Court of Appeals held that the date of the execution of the waiver on September 22, 1997 could
reasonably be understood as the same date of acceptance by the BIR. Petitioner points out however that
Revenue District Officer Sarmiento could not have accepted the waiver yet because she was not the
Revenue District Officer of RDO No. 33 on such date. Ms. Sarmiento’s transfer and assignment to
RDO No. 33 was only signed by the BIR Commissioner on January 16, 1998 as shown by the Revenue
Travel Assignment Order No. 14-98.28 The Court of Tax Appeals noted in its decision that it is unlikely

156
as well that Ms. Sarmiento made the acceptance on January 16, 1998 because "Revenue Officials
normally have to conduct first an inventory of their pending papers and property responsibilities."29

Finally, the records show that petitioner was not furnished a copy of the waiver. Under RMO No. 20-
90, the waiver must be executed in three copies with the second copy for the taxpayer. The Court of
Appeals did not think this was important because the petitioner need not have a copy of the document it
knowingly executed. It stated that the reason copies are furnished is for a party to be notified of the
existence of a document, event or proceeding.

The flaw in the appellate court’s reasoning stems from its assumption that the waiver is a unilateral act
of the taxpayer when it is in fact and in law an agreement between the taxpayer and the BIR. When the
petitioner’s comptroller signed the waiver on September 22, 1997, it was not yet complete and final
because the BIR had not assented. There is compliance with the provision of RMO No. 20-90 only
after the taxpayer received a copy of the waiver accepted by the BIR. The requirement to furnish the
taxpayer with a copy of the waiver is not only to give notice of the existence of the document but of the
acceptance by the BIR and the perfection of the agreement.

The waiver document is incomplete and defective and thus the three-year prescriptive period was not
tolled or extended and continued to run until April 17, 1998. Consequently, the Assessment/Demand
No. 33-1-000757-94 issued on December 9, 1998 was invalid because it was issued beyond the three
(3) year period. In the same manner, Warrant of Distraint and/or Levy No. 33-06-046 which petitioner
received on March 28, 2000 is also null and void for having been issued pursuant to an invalid
assessment.

WHEREFORE, premises considered, the instant petition for review is GRANTED. The Decision of
the Court of Appeals dated August 5, 2003 and its Resolution dated March 31, 2004 are REVERSED
and SET ASIDE. The Decision of the Court of Tax Appeals in CTA Case No. 6108 dated May 14,
2002, declaring Warrant of Distraint and/or Levy No. 33-06-046 null and void, is REINSTATED.

SO ORDERED.

DIGEST:

Philippine Journalist, Inc. v. CIR


G.R. No. 162852; December 16, 2004

Facts: In 1995, the Bureau of Internal Revenue (BIR) issued Letter of Authority for two Revenue
Officers to examine petitioner’s books of account and other accounting records for internal revenue
taxes for the period January 1, 1994 to December 31, 1994.
In 1997, petitioner’s Comptroller, executed a "Waiver of the Statute of Limitation Under the National
Internal Revenue Code (NIRC)". The document "waive[d] the running of the prescriptive period
provided by Sections 223 and 224 and other relevant provisions of the NIRC and consent[ed] to the
assessment and collection of taxes which may be found due after the examination at any time after the
lapse of the period of limitations fixed by said Sections 223 and 224 and other relevant provisions of
the NIRC, until the completion of the investigation.”
157
In 1998, Revenue Officer submitted his audit report recommending the issuance of an assessment and
finding that petitioner had deficiency taxes. Subsequently, the Assessment Division of the BIR issued
Pre-Assessment Notices which informed petitioner of the results of the investigation. Thus, BIR issued
Assessment/Demand stating the deficiency taxes, inclusive of interest and compromise penalty
On March 16, 1999, a Preliminary Collection Letter was sent by Deputy Commissioner Romeo S.
Panganiban to the petitioner to pay the assessment within ten (10) days from receipt of the letter. On
November 10, 1999, a Final Notice Before Seizure was issued by the same deputy commissioner
giving the petitioner ten (10) days from receipt to pay. Petitioner received a copy of the final notice on
November 24, 1999. By letters dated November 26, 1999, petitioner asked to be clarified how the tax
liability of P111,291,214.46 was reached and requested an extension of thirty (30) days from receipt of
the clarification within which to reply.
The BIR received a follow-up letter from the petitioner asserting that its (PJI) records do not show
receipt of Tax Assessment/Demand. Petitioner also contested that the assessment had no factual and
legal basis. On March 28, 2000, a Warrant of Distraint and/or Levy was received by the petitioner.
Petitioner filed a Petition for Review with the Court of Tax Appeals (CTA) which was amended on
May 12, 2000. Petitioner complains: (a) that no assessment or demand was received from the BIR; (b)
that the warrant of distraint and/or levy was without factual and legal bases as its issuance was
premature; (c) that the assessment, having been made beyond the 3-year prescriptive period, is null and
void; (d) that the issuance of the warrant without being given the opportunity to dispute the same
violates its right to due process; and (e) that the grave prejudice that will be sustained if the warrant is
enforced is enough basis for the issuance of the writ of preliminary injunction.
CTA ruled in favor of PJI. It declared that the deficiency income, value-added and expanded
withholding tax assessments issued by the respondent against the petitioner on December 9, 1998, in
the total amount of P111,291,214.46 for the year 1994 ANCELLED, WITHDRAWN andWITH NO
FORCE AND EFFECT. Likewise, it declared that the Warrant of Distraint and/or Levy No. 33-06-
046 NULL and VOID.
On appeal CA ruled that Mere assessment notices which have become final after the lapse of the thirty
(30)-day reglementary period are not appealable. Thus, the CTA should not have entertained the
petition at all. Also, it ruled that there is a valid waiver thus the running of the prescriptive period is
tolled.
Issues: (1) whether or not CTA has jurisdiction over the issues in this case. (2) Whether or not the
Waiver of the Statute of Limitations is valid and binding on the petitioner
Held: (1) No. The appellate jurisdiction of the CTA is not limited to cases which involve decisions of
the Commissioner of Internal Revenue on matters relating to assessments or refunds. The second part
of the provision covers other cases that arise out of the NIRC or related laws administered by the
Bureau of Internal Revenue. The wording of the provision is clear and simple. It gives the CTA the
jurisdiction to determine if the warrant of distraint and levy issued by the BIR is valid and to rule if the
Waiver of Statute of Limitations was validly effected.
(2) No. As found by the CTA, the Waiver of Statute of Limitations, signed by petitioner’s comptroller
on September 22, 1997 is not valid and binding because it does not conform with the provisions of
RMO No. 20-90. It did not specify a definite agreed date between the BIR and petitioner, within which
the former may assess and collect revenue taxes. Thus, petitioner’s waiver became unlimited in time,
violating Section 222(b) of the NIRC.
The waiver document is being incomplete and defective, the three-year prescriptive period was not
tolled or extended and continued to run until April 17, 1998. Consequently, the Assessment/Demand
158
No. 33-1-000757-94 issued on December 9, 1998 was invalid because it was issued beyond the three
(3) year period. In the same manner, Warrant of Distraint and/or Levy No. 33-06-046 which petitioner
received on March 28, 2000 is also null and void for having been issued pursuant to an invalid
assessment.

159

Potrebbero piacerti anche